You are on page 1of 272

Solutions to Exercises

Chapter 1

SINGLE CORRECT CHOICE TYPE QUESTIONS


1. The value of sin 36° is

( 5 + 1) 10 + 2 5 5 +1
(A) (B)
8 4
10 + 2 5 ( 5 − 1) 10 + 2 5
(C) (D)
8 8
Solution: We have

sin 36° =
( 5 −1 )( 10 + 2 5 )
8
Let A =18°,5 A =90° , then
2 A + 3 A =90°
⇒ 2 A= 90° − 3 A
⇒ sin (2 A=
) sin (90 − 3 A=
) cos (3 A)
⇒ 2sin ( A) ⋅ cos ( A)= 4 cos3 ( A) − 3cos ( A)
⇒ cos ( A)[2sin ( A) − 4 cos 2 ( A) + 3] =
0
⇒ cos ( A){2sin ( A) − 4[1 − sin 2 ( A)] + 3} = 0
⇒ cos( A) [4sin ( A) + 2sin ( A) − 1] =
2
0
Here A = 18° and we know that cos (18°) ≠ 0, that is, cos ( A) ≠ 0 , so
4sin 2 ( A) + 2sin ( A) − 1 =0
− 2 ± 4 + 16 − 2 ± 2 5 −1 ± 5
=
sin ( A) = =
2(4) 8 A
As A = 18°, so, sin (A) > 0
5−1
⇒ sin ( A) =
4
10 + 2 5
⇒ cos ( A) =1 − s in2 ( A) =
4

( 5 −1 10 + 2 5 ) (
)(= )(
5 −1 10 + 2 5 )
=
⇒ sin (2 A) 2 = sin (36)
4 4 8

Answer: (D)
2sin θ 1 − cos θ + sin θ
2. If a = , then is equal to
1 + cos θ + sin θ 1 + sin θ

(A) 1 − a (B) 1 + a (C) a (D) 1 / a


Solution: We have
2sin (θ ) 4sin (θ / 2) ⋅ cos (θ / 2) 2sin (θ / 2)
= a = =
1 + cos (θ ) + sin (θ ) 2sin (θ / 2) ⋅ cos θ / 2 + 2 cos (θ / 2) [sin (θ / 2) + cos (θ / 2)]
2

Solution Manual for Mathematics For JEE (Main & Advanced), Trigonometry Vector
Algebra Probability, Vol 2,
Copyright©2017 Wiley India Pvt. Ltd. All rights reserved
Now
1 − cos (θ ) + sin (θ ) 2sin 2 (θ / 2) + 2sin (θ / 2) ⋅ cos (θ / 2) 2sin (θ / 2)[sin (θ / 2) + cos (θ / 2)]
= = = a
1 + sin (θ ) [sin (θ / 2) + cos (θ / 2)]2 [sin (θ / 2) + cos (θ / 2)]2

Answer: (C)
sec8θ − 1
3. =
sec 4θ − 1

tan 8θ cot 8θ
(A) (B)
tan 2θ cot 2θ
tan 8θ cot 8θ
(C) (D)
tan 4θ cot 4θ
Solution: We have
sec (8θ ) −1 1 − cos (8θ ) cos (4θ ) 2sin 2 (4θ ) ⋅ cos (4θ )
= ⋅ =
sec (4θ ) −1 1 − cos (4θ ) cos (8θ ) 2sin 2 (2θ ) ⋅ cos (8θ )
sin (8θ ) ⋅ sin (4θ ) tan (8θ ) ⋅ 2sin (2θ ) ⋅ cos (2θ ) tan (8θ )
= = =
cos (8θ ) ⋅ sin 2 (2θ ) 2sin (2θ ) ⋅ sin (2θ ) tan (2θ )

Answer: (A)

4. If 0 < θ < π / 2 and sin θ = 3/ 5, then tan 2θ =

25 24 25 23
(A) (B) (C) (D)
24 7 7 7
Solution: We have
3
sin (θ ) =
5
2
3 4
cos (θ ) =1 − sin 2 (θ ) =−
1   =
5 5
sin (θ ) 3
tan (θ ) =
cos (θ ) 4
2 tan (θ ) 2 (3) (4) 24
=
tan (2θ ) = =
1 − tan 2 (θ ) 16 − 9 7
Answer: (B)

5. − sin 2 48° + cos 2 12° =

5 −1 5 −1
(A) (B)
8 4
5 +1 5 +1
(C) (D)
4 8
Solution: We have
cos 2 (12°) − sin 2 (48)°

Solution Manual for Mathematics For JEE (Main & Advanced), Trigonometry Vector
Algebra Probability, Vol 2,
Copyright©2017 Wiley India Pvt. Ltd. All rights reserved
cos (12° + 48°) ⋅ cos (12° − 48°)
cos (60°) ⋅ cos (−36°)
1
⋅ cos (36°)
2
1
= 1 − 2sin 2 (18°) 
2
  5 −1  
2
1 
= 1− 2 ⋅ 
2  4  
   

1  (3 − 2 5)  2 5 + 1
= 1 − =
2 4  8

Answer: (C)

n
6. Suppose that sin 3 x sin 3x = ∑ Am cos mx, where A0 , A1 , , Am are constants and An ≠ 0,
m= 0
then the value of n is

(A) 8 (B) 6 (C) 4 (D) 9


Solution: We have
sin 3 x ⋅ sin 3x
(3sin x − sin 3x)
= ⋅ sin 3x
4
3 1
= sin x ⋅ sin 3x − ⋅ sin 2 3x
4 4
1
= (6sin x sin 3x − 2sin 2 3x)
8
1
= [3(cos 2 x − cos 4 x) − (1 − cos 6 x)]
8
1
= (−1 + 3cos 2 x − 3cos 4 x + cos 6 x)
8
n
= ∑ Am cos mx
m =0
1 3
⇒ A0 = − , A1 = 0, A2 =
8 8
−3
=A3 0,= A4 =, A5 0
8
1
A6 =
8

Answer: (B)

Solution Manual for Mathematics For JEE (Main & Advanced), Trigonometry Vector
Algebra Probability, Vol 2,
Copyright©2017 Wiley India Pvt. Ltd. All rights reserved
7. If a, b, c and d are smallest positive angles in the ascending order such that the sine of each
angle is equal to a positive constant λ, then 4sin (a / 2) + 3sin(b / 2) + 2sin(c / 2) + sin(d / 2) is
equal to

(A) 2 1 + λ (B) 1 + λ
(C) 1 + λ (D) 3 1 + λ
Solution: Since 0 < a < b < c < d
and =
sin =
a sin =
b sin =
c sin d λ
Therefore,

b= π − a
c 2π + a
=
d 3π − a
=
Now,
a b c d
+ 3sin + 2sin + sin
4sin
2 2 2 2
a π − a   2π + a   3π − a 
= 4sin + 3sin   + 2sin   + sin  
2  2   2   2 
a a a a
= 4sin + 3cos − 2sin − cos
2 2 2 2
a a
= 2sin + 2 cos
2 2
 a a
=2  sin + cos  =2 1 + sin a =2 1 + λ
 2 2
Answer: (A)

8. If cos(α
= + β ) 4 / 5, sin(α
= − β ) 5 /13 and 0 < α , β < π / 4 then tan2a is equal to

46 56 56 65
(A) (B) (C) (D)
33 23 33 33
Solution: We have
2
4 4 3
cos (α + β ) = ,sin (α + β ) = 1 −   =
5 5 5
2
5  5  12
sin (α − β ) = , cos (α − β ) = 1 −   =
13  13  13
Now,
48
cos(α + β ) ⋅ cos (α −=
β ) cos 2 (α ) − sin 2=
(β )
65
15
and, sin (α + β ) ⋅ sin (α −=
β ) sin 2 (α ) − sin 2 =
(β )
65
− + −
33
cos (α ) − sin (α ) =
2 2
65

Solution Manual for Mathematics For JEE (Main & Advanced), Trigonometry Vector
Algebra Probability, Vol 2,
Copyright©2017 Wiley India Pvt. Ltd. All rights reserved
33
cos (2a ) =
65
2
 33  56
sin (2a ) =1−   =
 65  65
56
tan (2a ) =
33
Answer: (C)

9. If tan A= (1 − cos B) / sin B, then tan 2A

B B
(A) cot B (B) tan (C) cot (D) tan B
2 2
Solution: We have
1 − cos ( B) 2sin 2 ( B / 2)
=
tan ( A) = = tan ( B / 2)
sin ( B) 2sin ( B / 2) ⋅ cos ( B / 2)
2 tan ( A) 2 tan B / 2
=
tan (2 A) = = tan ( B)
1 − tan ( A) 1 − tan 2 ( B / 2)
2

Answer: (D)

π cos 3π 5π 7π 9π
10. cos + + cos + cos + cos =
11 11 11 11 11

1
(A) 0 (B) 1 (C) (D) 1
2
Solution: We have
π   3π   5π   7π   9π 
S =cos   + cos   + cos   + cos   + cos  
 11   11   11   11   11 
  π   π   π   3π   5π   7π   9π  
 2sin=
   S 2sin   cos   + cos   + cos   + cos   + cos  
  11    11    11   11   11   11   11  
2π 4π 2π 6π 4π 8π 6π 10π 8π
= sin + sin − sin + sin − sin + sin − sin + sin − sin
11 11 11 11 11 11 11 11 11
10π  π π
= sin = sin  π − = sin
11  11  11
sin (π / 11) 1
=S =
2sin (π / 11) 2
Answer: (C)

11. If cos 2 B =
cos( A + C ) / cos( A − C ), then tan A, tan B, tan C are in

(A) GP (B) HP (C) AP (D) AGP

Solution: It is given that

Solution Manual for Mathematics For JEE (Main & Advanced), Trigonometry Vector
Algebra Probability, Vol 2,
Copyright©2017 Wiley India Pvt. Ltd. All rights reserved
cos ( A + C )
cos (2 B) =
cos ( A − C )
1 − cos (2 B) cos ( A − C ) − cos ( A + C )
=
1 + cos (2 B) cos ( A − C ) + cos ( A + C )
2sin ( A) ⋅ sin (C )
2sin 2 ( B)
=
2 cos ( B) 2 cos ( A) ⋅ cos (C )
2

tan=
2
( B) tan ( A) ⋅ tan (C )
Hence, tan (A), tan (B) and tan (C) are in G.P.

Answer: (A)

0 and π / 2 < α < π , then sin2α =


12. If α is a root of the equation 25cos 2 θ + 5cos θ − 12 =

8 24 −24 −8
(A) (B) (C) (D)
13 25 25 13
Solution: We have
25cos 2 (θ ) + 5cos (θ ) − 12 =
0
− 5 ± (5)2 − 4.25 ⋅ (−12) − 5 ± 35
=cos (θ ) =
2 (25) 50
π
as < ‘α ’, < π
2
− 40 4
cos (α ) = =−
50 5
2
 −4  3
sin (α ) =
1−   =
 5  5
 3  −4  −24
sin (2α ) 2=
=   
 5  5  25

Answer: (C)

13. sin 2 A + sin 2 ( A − B) − 2sin A cos B sin( A − B) =

(A) cos 2 B (B) sin 2 B (C) cos 2 A (D) sin 2 A

Solution: It is given that


sin 2 ( A) + sin ( A − B)[sin ( A − B) − 2sin A ⋅ cos B]
sin 2 ( A) + sin ( A − B)[sin ( A − B) − sin ( A + B) − sin ( A − B)]
sin 2 ( A) − sin ( A − B) ⋅ sin ( A + B)
sin 2 ( A) − sin 2 ( A) + sin 2 ( B) =
sin 2 ( B)

Answer: (B)

cos /(1 − n cos 2 ), then tan(θ + a ) =


14. If tan θ n sin aaa
=

Solution Manual for Mathematics For JEE (Main & Advanced), Trigonometry Vector
Algebra Probability, Vol 2,
Copyright©2017 Wiley India Pvt. Ltd. All rights reserved
1 1
(A) tan a (B) cot α
1− n 1− n
(C) (1 + n) tan a (D) (1 + n) cot α
Solution: We have
n ⋅ sin (aa
) ⋅ cos ( )
tan (θ ) =
1 − n cos 2 (a )
n ⋅ sin (aa
) ⋅ cos ( )
=
cos 2 (a )
1 − n cos 2 (a )
=
cos 2 (a )
n tan (aa) n tan ( )
=tan (θ ) =
1 + tan (aa
2
) − n (1 − n) + tan 2 ( )
Now,
tan (θ ) + tan (a )
tan (θ + a ) =
tan (θ ) ⋅ tan (a )

[n tan (aaa
) / (1 − n) + tan 2 ( )] + tan ( )
=
1 − [n tan (aaa
) / (1 − n) + tan 2 ( )] ⋅ tan ( )
tan 3 (aaa
) + tan ( ) tan ( )
= =
(1 − n) (1 + tan 2 a ) (1 − n)

Answer: (A)

15
=
15. Let z cos θ + i sin θ. Then, at θ = 2°, the value of ∑ Re( z 2n −1 ) is
n =1

3 3
(A) cosec2° (B) sec 2°
2 4
3 3
(C) sec 2° (D) cosec2°
2 4
Solution: It is given that
2 n −1
z= (cos θ + i sin θ ) 2 n −1
Using De Moivre’s theorem
z 2 n −=
1
cos(2n − 1)θ + i sin(2n − 1)θ
z 2 n −1 ) cos(2n − 1)θ
Re( =
15
∑ Re( z 2n−1 ) =cosθ + cos 3θ + cos 5θ +  to 15 terms
n =1
cos(θ + 14θ ) ⋅ sin(15θ )
=
sin θ
sin15θ ⋅ cos15θ
=
sin θ
Put θ = 2°, then

Solution Manual for Mathematics For JEE (Main & Advanced), Trigonometry Vector
Algebra Probability, Vol 2,
Copyright©2017 Wiley India Pvt. Ltd. All rights reserved
15
3
∑ Re(
= z 2 n −1 )
4
cosec 2°
n =1

Answer: (D)

16. If α and β are solutions of the equation a tan θ + b sec θ =


c, then tan(a + β ) =

2ca 2ab 2bc a2 + c2


(A) (B) (C) (D)
a2 − c2 a 2 − b2 b2 − c2 a2 − c2
Solution: We have

[b sec (θ )]2= [c − a tan(θ )]2

b 2 [1 + tan 2 (θ )] =
= c 2 + a 2 tan 2 (θ ) − 2ac tan (θ )

= (a 2 − b 2 ) tan 2 (θ ) − 2ac tan (θ ) + (c 2 − b 2 ) = 0

2ac
tan ab
+ tan =
a − b2
2

 c2 − b2 
tan ab
⋅ tan =
 2 2 
 a −b 

tan (a ) + tan ( β ) 2ac


tan (a + β )
= =
1 − tan (a ) ⋅ tan ( β ) a 2 − c 2

Answer: (A)

17. In a ∆ABC, if p = sin A sin B sin C and q = cos A cos B cos C , then ∑ tan B tan C =

1+ p 1+ p 1+ q 1+ q
(A) (B) (C) (D)
p q q p
Solution: We have
cos ( A + B + C ) = cos A ⋅ cos B ⋅ cos C − cos A ⋅ sin B ⋅ sin C − sin A ⋅ cos B ⋅ sin C − sin A ⋅ sin B ⋅ cos C

Dividing both sides by


=q cos ( A) cos( B) ⋅ cos(C ) ,
we get
cos ( A + B + C )
=
1 − tan B ⋅ tan C − tan A ⋅ tan C − tan A ⋅ tan B
cos( A) ⋅ cos( B) ⋅ cos(C )
−1
= 1 − Σ tan B ⋅ tan C
q

1+ q
Σ tan B ⋅ tan C =
q

Solution Manual for Mathematics For JEE (Main & Advanced), Trigonometry Vector
Algebra Probability, Vol 2,
Copyright©2017 Wiley India Pvt. Ltd. All rights reserved
Answer: (C)

18. If A and B are acute angles (A > B) such that A + B and A − B satisfy the equation
tan 2θ − 4 tan 2 θ + 1 =0, then

π π π π
=
(A) A = ,B =
(B) A = ,B
3 4 3 6
π π π π
=
(C) A = ,B =
(D) A = ,B
4 6 3 12
Solution: We have

tan 2 (θ ) − 4 tan (θ ) + 1 =0

tan ( A + B) + tan ( A − B) =
4
and tan ( A + B) ⋅ tan ( A − B) =
1

sin ( A + B) ⋅ sin ( A − B)
=1
cos ( A + B) ⋅ cos ( A − B)
cos ( A + B) ⋅ cos ( A − B) − sin ( A + B) ⋅ sin ( A − B) =
0

cos ( A + B + A − B) =
0

cos (2 A) = 0

π
2 A= 90° or
2
π π
=A =,B
4 6

Answer: (C)

19. If tana tan β = a and α + β =


π / 6, then tana and tanβ are roots of the equation

(A) x 2 + 3(1 + a) x − a =0
(B) x 2 − 3(1 − a) x + a =0
(C) 3x 2 − (1 − a) x + a 3 =
0
(D) 3x 2 + (1 + a) x − a 3 =
0
Solution: We have
π  1
tan(a +=
β ) tan =
6 3
tan a + tan β 1
⇒ =
1 − tan a ⋅ tan β 3
(1 − a )
⇒ tan (a ) + tan ( β ) =
3

Solution Manual for Mathematics For JEE (Main & Advanced), Trigonometry Vector
Algebra Probability, Vol 2,
Copyright©2017 Wiley India Pvt. Ltd. All rights reserved
Now, for any quadratic equation, ax 2 + bx + c =0, we know
b 1− a
Sum of roots =− =
a 3

c a 3
and product of roots= =
a 3
So, a = 3, b =−(1 − a ) and c =a 3 , therefore required equation is

3x 2 − (1 − a) x + a 3 =
0

Answer: (C)

=
20. If tan tanβ 1/(2m − 1), then α − β is
a m /(m − 1) and =

π π π 5π
(A) (B) (C) (D)
4 6 3 12
Solution: We know that
tan a − tan β
tan(a − β ) =
1 + tan a ⋅ tan β

m(2m − 1) − (m − 1) 2m 2 − 2m + 1
= = = 1
(m − 1) (2m − 1) + m 2m 2 − 2m + 1
[m / (m − 1)] − [1 / (2m − 1)]
=
1 + [m / (m − 1)] − [1 / (2m − 1)]
m(2m − 1) − (m − 1)
=
(m − 1)(2m − 1) + m
2m 2 − 2m + 1
= = 1
2m 2 − 2m + 1
π
⇒α − β =
4

Answer: (A)

21. If tana and tanβ are roots of the equation x 2 + ax + b= 0 (a ≠ 0), then tan(a + β ) =

a a
(A) (B)
b −1 b +1
b b
(C) (D)
a −1 a +1
Solution: We have
tan a + tan β
tan (a + β ) =
1 − tan a ⋅ tan β
From the given equation,

Solution Manual for Mathematics For JEE (Main & Advanced), Trigonometry Vector
Algebra Probability, Vol 2,
Copyright©2017 Wiley India Pvt. Ltd. All rights reserved
tan (a ) + tan ( β ) =
−a

tan ab
⋅ tan =
b

−a a
tan(ab
+ )= =
1 − b b −1

Answer: (A)

22. If n is a positive integer and

π π n
sin + cos =
2n 2n 2
then
(A) 1 ≤ n ≤ 8 (B) 1 ≤ n ≤ 4 (C) n = 6 (D) n = 8
Solution: We have
1 π 1 n π
⋅ sin + =
cos (1)
2 2n 2 2n 2 2
 π π  n
sin  + = (2)
 2n 4  2 2
Clearly n = 1 does not satisfy Eq. (1). Now, if n ∈ z + − {1} , Then
π π π  π
< + <
4  4 2n  2
Therefore,
1 π π 
< sin  +  ≤ 1
2  4 2n 
from Eq. (2)
1 n
< ≤1
2 2 2
2< n ≤2 2
4<n≤8 (3)
Put n = 5 in Eq. (1), then
π π
=
LHS sin + cos
10 10
5 −1 10 + 2 5
= +
4 4
5
RHS =
2 2
Clearly n = 5 does not satisfy Eq. (1). Put n = 6 in Eq. (1)
π π
=LHS sin + cos
12 12
3 −1 3 +1 3
= + =
2 2 2 2 2
6 3
=
RHS =
2 2
So, n = 6 satisfies Eq. (1) but n = 7, 8 do not satisfy then option (C) is correct.
Solution Manual for Mathematics For JEE (Main & Advanced), Trigonometry Vector
Algebra Probability, Vol 2,
Copyright©2017 Wiley India Pvt. Ltd. All rights reserved
Answer: (C)

23. If cos(θ − a ) =
a and sin(θ − b ) =
b, then the value of cos 2 (abab
− ) + 2ab sin( − ) is

(A) 4a 2b 2 (B) a 2 + b 2
(C) a 2 − b 2 (D) a(a + b)
Solution: We have

cos 2 (abab
− ) + 2ab sin ( − )

cos 2 [(θ − b ) − (θ − a )] + 2ab sin[(θ − b ) − (θ − a )]

⇒ [cos (θ − b ) ⋅ cos (θ − a ) + sin (θ − b ) ⋅ sin(θ − a )]2 + 2ab [sin (θ − b ) ⋅ cos (θ − a ) − sin (θ − a ) ⋅ cos (θ − b )]

2
⇒  (1 − b 2 ) ⋅ a + b ⋅ (1 − a 2 )  + 2ab  ab − (1 − a 2 ) (1 − b 2 ) 
   

=a 2 (1 − b 2 ) + b 2 (1 − a 2 ) + 2ab (1 − a 2 ) (1 − b 2 ) + 2a 2b 2 − 2ab (1 − a 2 ) (1 − b 2 )

= a 2 + b2

Answer: (B)

24. If tana = 3, then

2sin 2α − 3cos 2α
=
4sin 2α + 5cos 2α
9 −9 13 −13
(A) (B) (C) (D)
4 4 4 4
Solution: We know that
2 tan a 3
sin 2a
= =
1 + tan a
2
5

1 − tan 2 a 4
cos 2a = =−
1 + tan a2
5

2(3/ 5) − 3(−4 / 5) 18 −9
= =
4(3/ 5) + 5(−4 / 5) −8 4

Answer: (B)

1 + sin 2aa 1 − tan 2 / 2


25. − =
sinaa + cos 1 + tan 2 a / 2

(A) − cos α (B) cosα (C) − sin α (D) sinα


Solution:

Solution Manual for Mathematics For JEE (Main & Advanced), Trigonometry Vector
Algebra Probability, Vol 2,
Copyright©2017 Wiley India Pvt. Ltd. All rights reserved
[sin(α ) + cos (α )]2
= cos(α )
sin(α ) + cos(α )

sin (α ) + cos (α ) − cos (α ) =


sin (α )

Answer: (D)

26. sin 2 70° ⋅ sin 2 50° ⋅ sin 2 10° =

1 1 1 1
(A) (B) (C) (D)
64 32 128 256
Solution: We have

sin 2 70°⋅ sin 2 50°⋅ sin=


2
10° (sin 70°⋅ sin 50°⋅ sin10°)2
2
1 
=  [sin 70 ⋅ (cos 40° − cos 60°)]
2 
2
1 1 1 
=   (sin (110°) + sin(30°)) − sin 70° 
 
2 2 2 
2
1 1 1 1
=  ⋅ ⋅  =
2 2 2 64

Answer: (A)

27. If 0 < α , β < π / 2, sin(α + β ) = 1 / 2, then tan(a + 2β ) =


1 and sin(α − β ) =

1 −1
(A) 3 (B) − 3 (C) (D)
3 3
Solution: We have
π
sin(α + β ) ==
1 sin
2
π
α +β =
2
1 π 
sin (α − β ) = =sin  
2 6
π
⇒α −β =
6
π
α +β=
2
π π
=α =, β
3 6

Solution Manual for Mathematics For JEE (Main & Advanced), Trigonometry Vector
Algebra Probability, Vol 2,
Copyright©2017 Wiley India Pvt. Ltd. All rights reserved
π  π  2π
α + 2β = + 2   =
3 6 3

 2π 
tan(a + 2β ) =
tan  =− 3
 3 

Answer: (B)
28. If tan a /2 = m, then

1 − 2sin 2 α / 2
=
1 + sin α
1+ m 1− m
(A) (B)
1− m 1+ m
m −m
(C) (D)
1+ m 1+ m
Solution: We know that

1 − 2sin 2 aaaa
/2 cos / 2 − sin / 2 1 − tan / 2 1 − m
= = =
(cos aa/ 2 + sin / 2) 2
cos aaa
/ 2 + sin / 2 1 + tan / 2 1 + m

Answer: (B)

29. If tanaa a, then tan 4aa


+ cot = + cot 4 =

(A) (a 2 − 2)2 − 2 (B) (a 2 + 2)2 − 2


(C) (a 2 − 2)2 + 2 (D) (a 2 − 2)2 + 4
Solution: We have
1
tan a + =
a
tan a

tan 2 aa
+ cot 2 + 2 =a2

tan 2 aa
+ cot 2 ( ) =
a2 − 2

tan 4 (aa
) + cot 4 ( ) = (a 2 − 2) 2 − 2

Answer: (A)

30. If 0 < α < π such that sinα + cos α =


1 / 5, then tan a /2 =

−1 1
(A) (B) (C) 2 (D) 2
3 3
Solution:

Solution Manual for Mathematics For JEE (Main & Advanced), Trigonometry Vector
Algebra Probability, Vol 2,
Copyright©2017 Wiley India Pvt. Ltd. All rights reserved
2
 sec a 
[tan(a ) + 1]2 =
 5 
 

sec2 (a )
tan 2 (aa
) + 1 + 2 tan( ) =
25

25[tan 2 (aaa
) + 2 tan( ) + 1] =1 + tan 2 ( )

24 tan 2 aa
+ 50 tan( ) + 24 =
0

−50 ± (50)2 − 4 ⋅ (24) (24) −50 ± 2500 − 2304 −50 ± 196 −50 ± 14
tan (a )
= = = =
2(24) 48 48 48

4 3
tan(a ) =
− or −
3 4
2 tan a / 2 4
=−
1 − tan (a / 2)
2
3

 aa  
2 tan 2   − 3tan   − 2 =0
 
2 2

a 
tan   = 2
2
a −1
Here, for −3/4, we get tan = and 3
2 3
But ‘3’ is not an option
0, then sin(α + 2β ) =
31. If cos(α + β ) =

(A) sinα (B) cosα (C) sinβ (D) cosβ


Solution: We have
sin (α + β + β=) sin(α + β ) ⋅ cos( β ) + cos(α + β ) ⋅ sin β ⋅1⋅ cos ( β ) + 0 ⋅ sin ( β )

= cos( β )

Answer: (D)

sinβ n
32. If = and |n| < |m|, then
sin(2α + β ) m

1 + (tanβ / tanaa
) 1 − tan tan β
(A) =
m+n m−n
1 − (tanβ / tanaa
) 1 − tan tan β
(B) =
m+n m−n

Solution Manual for Mathematics For JEE (Main & Advanced), Trigonometry Vector
Algebra Probability, Vol 2,
Copyright©2017 Wiley India Pvt. Ltd. All rights reserved
1 + tan a tan β 1 − (tana / tanβ )
(C) =
m+n m−n
1 − tan a tan β 1 + (tanβ / tana )
(D) =
m+n m−n
Solution: We have
sin( β ) n
=
sin(2α + β ) m
By componendo and devidendo
sin(2α + β ) + sin( β ) m + n
=
sin(2α + β ) − sin( β ) m − n
2sin(α + β ) ⋅ cos(α ) m + n
⇒ =
2 cos(α + β ) ⋅ sin(α ) m − n

tan (a + β ) m + n
⇒ =
tan (a ) m−n

1 + [tan ( β ) / tan (aa


)] 1 − tan( ) ⋅ tan( β )
=
m+n m−n

Answer: (A)

33. If α, β, γ are in AP, then

sinα − sin γ
=
cosγ − cos α
(A) cotβ (B) tanβ
(C) tana cot β (D) tanγ cot β
Solution: We have
sin (α ) − sin γ 2 cos (α + γ / 2) ⋅ sin (α − γ / 2) α +γ 
= = cot
cos γ − cos α 2sin (α + γ / 2) ⋅ sin (α − γ / 2)
=
  cot( β )
 2 

Answer: (A)

sin3α ⋅ cos3 α + cos 3α ⋅ sin 3 α


34. =
3

1 1
(A) sin4α (B) sin4α
2 4
1
(C) cos4α (D) 4 cos 4α
4
Solution: Since

Solution Manual for Mathematics For JEE (Main & Advanced), Trigonometry Vector
Algebra Probability, Vol 2,
Copyright©2017 Wiley India Pvt. Ltd. All rights reserved
=
sin 3α 3sin α − 4sin 3α
Therefore,
1
sin 3 α
= (3sin α − sin 3α )
4
Since
cos 3α 4 cos3 α − 3cos α
=
Therefore,
1
cos3 α
= (cos 3α + 3cos α )
4
Now,
1
(sin 3α cos3 α + cos 3α ⋅ sin 3 α )
3
1 1 1  3sin α − sin 3α 
= sin 3α ⋅ (cos 3α + 3cos α ) + cos 3α ⋅  
3 4 3  4 
1 1 1 1
= sin 3α ⋅ cos 3α + sin 3α ⋅ cos α + cos 3α sin α − sin 3α cos 3α
12 4 4 12
1 1 1
= (sin 3α cos α + cos 3α sin=α) sin(3α +=α) sin 4α
4 4 4

Answer: (B)

sin2α − sin 3α + sin 4α


35. =
cos2α − cos 3α + cos 4α

(A) tan2a (B) cot2α


(C) tan3a (D) cot3α
Solution:
2sin (3α ) ⋅ cos (α ) − sin (3α )
2 cos (3α ) ⋅ cos (α ) − cos (3α )
sin (3aa
) (2 cos ( ) −1)
= tan (3a )
cos (3aa
) (2 cos ( ) −1)

Answer: (C)

36. If cot x, cot y, cot z are in AP, then cot( y − x), cot y, cot( y − z ) are in

(A) AP (B) GP (C) HP (D) AGP


Solution: Since cot x, cot y, cot z are in A.P, then
=
2 cot y cot x + cot z
Now,
cot( y − x) + cot( y − z )
Therefore,
sin( A + B)
cot A + cot B =
sin A ⋅ sin B
Therefore,

Solution Manual for Mathematics For JEE (Main & Advanced), Trigonometry Vector
Algebra Probability, Vol 2,
Copyright©2017 Wiley India Pvt. Ltd. All rights reserved
cot( y − x) + cot( y − z )
cot y cot x + 1 cot y cot z + 1
= +
cot x − cot y cot z − cos y
cot y cot x + 1 cot y cot z + 1
= −
cot x − cot y cot x − cot y
cot y cot x + 1 − cot y cot z − 1
=
cot x − cot y
cot y (cot x − cot z )
=
(cot x − cot y )
cot y ⋅ 2(cot x − cot y )
= = 2 cot y
cot x − cot y
⇒ cot (y – x), cot y, cot (y – z) are in A.P.

Answer: (A)

7 / 2 and 0 < θ < π / 6, then tan(θ / 2) =


37. If sinθ + cos θ =

1 1
(A) (3 − 7) (B) ( 7 − 2)
3 3
(C) 7 − 2 (D) 7+2
Solution:
2
 7 
[tan (θ ) + 1] =
2
 sec (θ ) 
 2 
7
⇒ tan 2 (θ ) + 1 + 2 tan (θ =) [1 + tan 2 (θ )]
4

⇒ 3tan 2 (θ ) − 8 tan (θ ) + 3 =0

8 ± 64 − 36 4 ± 7
(θ )
⇒ tan= =
6 3
4± 7
tan (θ ) =
3
π
Now as 0 ≤ θ ≤
6

4− 7
tan (θ ) =
3

2 tan (θ / 2) 4− 7
⇒ =
1 − tan (θ / 2)
2
3

Solution Manual for Mathematics For JEE (Main & Advanced), Trigonometry Vector
Algebra Probability, Vol 2,
Copyright©2017 Wiley India Pvt. Ltd. All rights reserved
θ  θ 
( 7 − 4) tan 2   + 6 tan   − (4 − 7 ) =
0
2 2

 θ  −6 ± 36 + 4(4 + 7 − 8 7 )
tan   =
2 2(4 − 7 )

−6 ± 4 5 − 2 7
=
2(4 − 7 )

2 5− 2 7 −3
=
4− 7

Answer: (B)

If θ π /(2n + 1), the cos θ , cos 2θ , cos 22 θ , , cos 2n θ =


38. =

1 1
(A) 2n (B) 2n+1 (C) n+1
(D)
2 2n
Solution: We have

=M cos (θ ) ⋅ cos (2θ ), , cos 2n (θ )

sin (θ ) ⋅ M
= sin (θ ) ⋅ cos (θ ),, cos 2n (θ )

1
sin (θ )=
⋅M sin 2θ ⋅ cos 2θ ,, cos 2n (θ )
2
1
= sin 22 (θ ),,cos 2n (θ )
22
1
= n
⋅ sin 2n (θ ) ⋅ cos 2n (θ )
2
1
= n +1
sin(2) n +1 (θ )
(2)

1 sin 2n +1 (θ )
=
M n +1

(2) sin (θ )

1
= n+1
2

Answer: (C)
39. The value of tanθ ⋅ tan(θ + 60°) + tan θ tan(θ − 60°) + tan(θ − 60°) tan(θ + 60°) + 3 is

1
(A) 0 (B) 1 (C) 6 (D)
3
Solution: We have
Solution Manual for Mathematics For JEE (Main & Advanced), Trigonometry Vector
Algebra Probability, Vol 2,
Copyright©2017 Wiley India Pvt. Ltd. All rights reserved
tan(θ )[tan(θ + 60°) + tan(θ − 60°)] + tan(θ + 60°) ⋅ tan(θ − 60°) + 3

tan(θ )[sin(2θ )] [sin 2 (θ ) − sin 2 60°]


= 2 + +3
[cos (θ ) − sin 2 60°] [cos 2 (θ ) − sin 2 (60°)]

2sin 2 (θ ) + sin 2 (θ ) − 3 / 4 + 3cos 2 (θ ) − 9 / 4


=
[cos 2 (θ ) − 3 / 4]

3−3 / 4 −9 / 4
= = 0
[cos 2 (θ ) − 3 / 4]

Answer: (A)

40. The value of sin 310° + sin 3 50° − sin 3 70° is

−3 −3 3 2
(A) (B) (C) (D)
8 4 5 5
Solution: Since

=
sin 3θ 3sin θ − 4sin 3 θ
1
= sin
= 3
θ (3sin θ − sin 3θ )
4
1 3 1
= sin=
3
10° (3sin10° − sin= 30°) sin10° −
4 4 8
1
=
sin 3 50° (3sin 50° − sin150°)
4
3 1
= sin 50° −
4 8
1
=
sin 3 70° (3sin 70° − sin 210)
4
3 1
= sin 70° +
4 8
Now,

sin 3 10° + sin 3 50° − sin 3 70°


3 3
= (sin10° + sin 50° − sin 70°) −
4 8
3 3
= (2sin 30° cos 20° − sin 70°) −
4 8
3 1  3
=  2 × sin 70° − sin 70°  −
4 2  8
3 −3
=0 − =
8 8

Answer: (A)
41. If

Solution Manual for Mathematics For JEE (Main & Advanced), Trigonometry Vector
Algebra Probability, Vol 2,
Copyright©2017 Wiley India Pvt. Ltd. All rights reserved
A+ B A− B C+D C−D
sin cos + sin cos =
2
2 2 2 2
then ∑ cos( A / 2)cos(B/2) =
(A) 0 (B) 3 (C) 2 (D) 4
Solution:
1
[sin( A) + sin( B) + sin(C ) + sin( D)] =
2
2
sin( A) + sin( B) + sin(C ) + sin( D) =
4
This is only when
π
A= B= C= D=
2
Now

A B 1   A+ B   A − B  1
∑ cos=
2
⋅ cQ
2
cos 
2  2   + cos =
 2  2


A B 1 C D 1
cos ⋅ cQ = , cos ⋅ cQ =
2 2 2 2 2 2
D A 1
cos ⋅ cos = =
−2
2 2 2
Answer: (C)
42. If 450° < θ < 540° and sinθ = 336 / 625, then sin(θ / 4) =

3 4 7 1
(A) (B) (C) (D)
5 5 8 5 2
Solution: Since
336 −527
sin θ = ⇒ cos θ =
625 625
Now
θ 1152
2sin 2 = 1 − cos θ =
2 625
θ 576
⇒ sin 2 =
2 625
θ 24
⇒ sin = ±
2 25
Since
θ
225° < < 270°
2
Therefore,
θ −24 θ 7
sin = , cos = −
2 25 2 25
Now

Solution Manual for Mathematics For JEE (Main & Advanced), Trigonometry Vector
Algebra Probability, Vol 2,
Copyright©2017 Wiley India Pvt. Ltd. All rights reserved
θ 32 θ
2 sin 2 = 1 − cos =
4 2 25
θ 16
⇒ sin 2 =
4 25
θ 4
⇒ sin = ±
4 5
Since
1° θ
112 < < 135°
2 4
Therefore,
θ 4
sin =
4 5

Answer: (B)
π , then
43. If A + B + C =

sin 2 A + sin 2 B + sin 2 C


=
cos A cos B cos C
(A) 2 (B) 4 (C) 2 (D) 4
Solution: We have
2 (sin 2 A + sin 2 B + sin 2 C )
2 cos ( A) ⋅ cos ( B) ⋅ cos (C )

3 − [cos(2 A) + cos(2 B) + cos(2C )]


=
[cos( A + B) + cos( A − B)] ⋅ cos C

3 − [2 cos ( A + B) − cos( A − B) + cos 2C ]


=
[− cos 2 (C ) − cos( A − B) ⋅ cos( A + B)]

3 − [2(cos 2 A − sin 2 B) + 2 cos 2 (C ) −1]


=
[− cos 2 (C ) − cos 2 ( A) + sin 2 ( B)]

Answer: (A)
π , then
44. If A + B + C =

A B C π − A π − B  π −C 
sin + sin + sin = 1 + k sin   sin   sin  
2 2 2  4   4   4 
where k is equal to
1 1
(A) 2 (B) (C) 4 (D)
2 4

Solution: Let A + B + C = π, then

A B C A+ B A− B π −C 
sin +=
sin + sin 2sin cos + cos  
2 2 2 4 4  2 

Solution Manual for Mathematics For JEE (Main & Advanced), Trigonometry Vector
Algebra Probability, Vol 2,
Copyright©2017 Wiley India Pvt. Ltd. All rights reserved
π −C  A− B π −C 
= 2sin   cos + 1 − 2sin 2  
 4  4  4 
π −C  A− B  π − C 
=1 + 2sin   cos − sin  
 4  4  4 
π −C   A− B   π A + B 
=
1 + 2sin   cos   − cos  − 
 4   4  2 4  
π −C   A− B   2π − A − B  
=1 + 2sin   cos   − cos  
 4   4   4 
 π − A   π − B   π − C 
= 1 + 4sin   sin   sin  
 4   4   4 
So, K = 4
Answer: (C)

45. If A + B + C =
2s, then sin( s − A)sin( s − B) + sin s ⋅ sin( s − C ) =

(A) sin A cos B (B) sin B cos A


(C) cos A cos B (D) sin A sin B
Solution:
1
[2sin ( s − A) ⋅ sin ( s − B) + 2sin ( s ) ⋅ sin ( s − C )]
2
1
= [cos (2 s − A − B) − cos ( A − B) + cos (2 s − C ) − cos (C )]
2
1
= [cos (C ) − cos ( A − B) + cos ( A + B) − cos (C )]
2
1
=[2 ⋅ sin( A) ⋅ sin( B)]
2

= sin( A) ⋅ sin( B)

Answer: (D)
46. If cos 3 A + cos 3B + cos 3C =
1 in a triangle ABC , then one angle of the triangle must be

π 2π 4π
(A) (B) (C) π (D)
3 3 3
Solution: We know that
π
A+ B+C=

3( A + B)  π 3C 
⇒ = π + 2 − 2 
2  

 3( A + B)   π 3C   3C 
⇒ cos  =cos  π + − =− sin  
 2   2 2   2 

Solution Manual for Mathematics For JEE (Main & Advanced), Trigonometry Vector
Algebra Probability, Vol 2,
Copyright©2017 Wiley India Pvt. Ltd. All rights reserved
Now
cos 3 A + cos 3B + cos 3C =
1

3( A + B) 3 3C
2 cos ⋅ cos ( A − B) + 1 − 2sin 2 =1
2 2 2
3C  3 cos 3( A + B) 
−2sin cos ( A − B) − =0
2  2 2 
3C
sin = sin (π )
2

3C 2π
= π=
, c
2 3

Answer: (B)

47. If cos θ= (a cos aa


+ b) /(a + b cos ), then tan 2 (θ / 2) =

a−b a a+b a
(A) tan 2 (B) cos 2
a+b 2 a−b 2
a−b 2 a a−b 2a
(C) sin (D) cot
a+b 2 a+b 2
Solution: Since
a cos a + b
cos θ =
a + b cos a
Using componendo and dividendo
1 − cos θ (a + b cos aa
) − (a cos + b)
=
1 + cos θ (a + b cos aa
) + (a cos + b)
2sin 2 (θ / 2) a(1 − cos aa
) − b(1 − cos )
=
2 cos (θ / 2)
2
a(1 + cos aa
) + b(1 − cos )
θ a − b 1 − cos a
⇒ tan 2 = ⋅
2 a + b 1 + cos a
θ a−b a
⇒ tan 2 = ⋅ tan 2
2 a+b 2

Answer: (A)

48. Minimum value of 4 x 2 − 4 x | sin θ | − cos 2 θ is

−1
(A) –2 (B) –1 (C) 0 (D)
2
Solution:

4(sin θ ) ± 16 (sin 2 θ + cos 2 θ )


x=
8

Solution Manual for Mathematics For JEE (Main & Advanced), Trigonometry Vector
Algebra Probability, Vol 2,
Copyright©2017 Wiley India Pvt. Ltd. All rights reserved
4 (sin θ ) ± 4 1
= = −
8 2

Answer: (D)
49. If 3θ, 9θ, and 27θ are not odd multiples of π / 2, then

 sin θ sin 3θ sin 9θ 


2 + + =
 cos 3θ cos 9θ cos 27θ 
(A) tan 27θ − tan θ (B) tan θ − tan 27θ
(C) cot 27θ − cot θ (D) cot θ − cot 27θ
Solution: Since
2sin θ 2sin θ cos θ sin 2θ
= =
cos 3θ cos 3θ ⋅ cos θ cos 3θ cos θ
sin(3θ − θ)
= = tan 3θ − tan (1)
cos 3θ cos θ
Similarly,
2sin 3θ
= tan 9θ − tan 3θ (2)
cos 9θ
and
2sin 9θ
= tan 27 θ − tan 9θ (3)
cos 27 θ
From Eqs. (1) + (2) + (3), we get
 sin θ sin 3θ sin 9θ 
2 + + = tan 27 θ − tan θ
 cos 3θ cos 9θ cos 27 θ 
Now
bn − bn −1 secn −1 θ ⋅ sin(n − 1)θ sin(−θ )
=
an −1 cos θ ⋅ sin(n − 1)θ secn −1 θ
= − tan θ (4)
and
an sin nθ ⋅ secn θ 1
= = tan (5)
nbn n ⋅ cos nθ ⋅ sec θ n
n

From Eqs. (4) + (5), we get


bn − bn −1 an 1
+ − tan θ + tan n
=
an −1 nbn n
Answer: (A)
50. If

cos x cos( x + θ ) cos( x + 2θ ) cos( x + 3θ )


= = =
a b c d
then (a + c) /(b + d ) is equal to
c b d a
(A) (B) (C) (D)
d c a d
Solution: We have
If

Solution Manual for Mathematics For JEE (Main & Advanced), Trigonometry Vector
Algebra Probability, Vol 2,
Copyright©2017 Wiley India Pvt. Ltd. All rights reserved
cos x cos( x + θ ) cos( x + 2θ ) cos( x + 3θ )
= = = (1)
a b c d
Use addendo
Since
a c a+c
= = 
b d b+d 
Now, Eq. (1) is also equal to
cos x + cos( x + 2θ ) cos( x + θ ) + cos( x + 3θ )
=
a+c b+d
a+b cos( x + 2θ ) + cos x
=
b + d cos( x + 3θ ) + cos( x + 2θ )
cos( x + θ ) ⋅ cos θ b
= =
2 cos( x + 2θ ) cos θ c

Answer: (B)
51. If sin α = −(3 / 5) and π < α < 3π / 2, then cos(α / 2) =

1 −1 −1 1
(A) (B) (C) (D)
5 10 5 10
Solution: We have
3
sin (α ) = −
5
4
cos (α ) = −
5

α  1 + cos(α ) 1
cos   = = −
2 2 10

Answer: (B)

If an sin(nθ ) ⋅ sec
52.= = n
θ and bn cos(nθ )secn θ ≠ 1, then

bn − bn −1 an
+ =
an −1 nbn
(A) 0 (B) tan θ
1 1
(C) − tan θ + tan(nθ ) (D) tan θ + tan(nθ )
n n
Solution: We have
bn − bn −1 an
+
an −1 nbn

sec n −1 (θ )[{cos (nθ ) / cos (θ )} − cos(n −1) (θ )] 1


+ tan (nθ )
sin (n −1) (θ ) ⋅ sec n −1 (θ ) n

Solution Manual for Mathematics For JEE (Main & Advanced), Trigonometry Vector
Algebra Probability, Vol 2,
Copyright©2017 Wiley India Pvt. Ltd. All rights reserved
[cos(nθ ) −1 / 2{(cos (nθ ) + cos (n − 2) θ )}] 1
= + tan (nθ )
cos ⋅[sin (n −1)θ ] n

1
− tan (θ ) +
= tan (nθ )
n

Answer: (C)
53. If sin( y + z − x), sin( z + x − y ), sin( x + y − z ) are in AP, then tan x, tan y, tan z are in

(A) AP (B) GP (C) HP (D) AGP


Solution: Since
sin( y + z − x),sin( z + x − z ),sin( x + y − z )
are in A.P. Therefore,
sin( z + x − y ) − sin( y + z − x)
= sin( x + y − z ) − sin( z + x − y )
⇒ 2 cos z ⋅ sin( x − =
y ) 2 cos x ⋅ sin( y − z )
divide by cos x ⋅ cos y ⋅ cos z
sin( x − y ) sin( y − z )
⇒ =
cos x cos y cos y cos z
⇒ tan x − tan y = tan y − tan z
⇒ tan x, tan y, tan z are in A.P.
Answer: (A)
54. If sec(θ − α ), sec θ , sec(θ + α ) are in AP, then the ratio cos θ : cos(α / 2) can be

(A) 2:1 (B) 1:2 (C) 1: 2 (D) 2 :1


Solution: Since
sec(θ − α ),sec θ ,sec(θ + α )
are in A.P. Therefore,
2sec θ= sec(θ + α ) + sec(θ − α )
2 cos(θ + α ) + cos(θ − α )
⇒ =
cos θ cos(θ + α ) cos(θ − α )
⇒ 2 cos(α + α ) cos(α − α ) =
cos θ ( 2 cos θ cos α )
cos θ − sin α =cos α ⋅ cos 2 θ
2 2

(1 − cos α ) ⋅ cos 2 θ =
sin 2 α
sin 2 α
cos 2 θ =
1 − cos α
sin 2 α
=
2sin 2 (α / 2)
sin α
cos θ = ±
2 sin(α / 2)
2sin(α / 2) cos(α / 2)
= ±
2 sin(α / 2)

Solution Manual for Mathematics For JEE (Main & Advanced), Trigonometry Vector
Algebra Probability, Vol 2,
Copyright©2017 Wiley India Pvt. Ltd. All rights reserved
α
⇒ cos θ : cos =
2 :1
2

Answer: (D)
55. If

2sin A sin C
tan B =
sin( A + C )
then cot A, cot B, cot C are in
(A) AP (B) GP (C) HP (D) AGP
Solution: We have
2sin A ⋅ sin C
tan ( B) =
sin ( A + C )

1  sin A ⋅ cos C + cos A ⋅ sin C 


cot ( B) =  
2 sin A ⋅ sin C 

1
=
cot ( B) (cot C + cot A)
2

Answer: (A)
56. If

(1 + m) tan a (1 − m) tan a
tan x = and tan y =
1 − m tan a
2
1 + m tan 2 a
then
 x − y  x + y
tan   cot  =
 2   2 
(A) 2 m (B) 1 + m 2 (C) m 2 − 1 (D) m
Solution:
 x− y   x+ y 
tan   ⋅ cot  
 2   2 

2sin[( x − y ) / 2}]cos[( x + y ) / 2]
=
2 cos[( x − y ) / 2] ⋅ sin[( x + y ) / 2]

sin ( x) − sin ( y )
= = m
sin ( x) + sin ( y )

Answer: (D)

MULTIPLE CORRECT CHOICE TYPE QUESTIONS


1. Let

n n
 cos A + cos B   sin A + sin B 
=x   + 
 sin A − sin B   cos A − cos B 
Then

Solution Manual for Mathematics For JEE (Main & Advanced), Trigonometry Vector
Algebra Probability, Vol 2,
Copyright©2017 Wiley India Pvt. Ltd. All rights reserved
(A) x = 0 if n is an odd positive integer
=
(B) x tan n ( A − B) / 2 if n is an even positive integer
=
(C) x 2 cot n ( A − B) / 2 if n is an even positive integer
(D) x = 0 if n is an even positive integer

We know that
Solution:

n n
 cos ( A − B / 2)   cos( A − B / 2) 
=x   + 
 sin ( A − B / 2)   sin (( B − A / 2) 

 A−B   A− B 
cot n   + (−1)n cot n  
 2   2 

For n = odd positive


(−1)n = −1
X=0 → (A)
For n = even positive
 A− B 
x = 2 cot n  
 2 
Answer: (A), (C)

2. Which of the following statements are true?


(A) If cos x + sin x =2 cos x, then cos x − sin x =2 sin x
(B) If 3sin θ + 5cos θ = 5, then 5sin θ − 3cos θ = ±3
3
If A sin 2 θ + cos 4 θ , then ≤ A ≤ 1
(C) =
4
(D) If (1 + sin A)(1 + sin B)(1 + sin C ) =
(1 − sin A)(1 − sin B)(1 − sin C ), then each is equal
to ±1

Solution:
(A) cos x + sin x = 2 cos x (1)
Let cos x − sin x =
t (2)
(1)2 + (2)2
=2 2 cos 2 x + t 2
⇒ t =± 2 sin x
(B) 3sin θ + 5cos θ =
5 (1)
Let 5sin θ − 3cos θ =
t (2)
(1)2 + (2)2
t 2 + 25 =3 y ⇒ t =±3
=
(C) A sin 2 θ + cos 4 θ
= cos 4 θ − cos 2 θ + 1
2
 1 3
=  cos 2 θ −  +
 2 4
Since
Solution Manual for Mathematics For JEE (Main & Advanced), Trigonometry Vector
Algebra Probability, Vol 2,
Copyright©2017 Wiley India Pvt. Ltd. All rights reserved
0 ≤ cos 2 θ ≤ 1
−1 1 1
≤ cos 2 θ − ≤
2 2 2
2
 1 1
0 ≤  cos 2 θ −  ≤
 2 4
2
3  2 1
≤  cos θ −  ≤ 1
4  2
3
≤ A ≤1
4
(D) Multiply (1 + sin A)(1 + sin B)(1 + sin C) in both side, then
(1 + sin A)2 (1 + sin B)2 (1 + sin C )2 = cos 2 A ⋅ cos 2 B ⋅ cos 2 C
⇒ (1 + sin A)(1 + sin A)(1 + sin C ) =
± cos A ⋅ cos B ⋅ cos C
Option (D) is incorrect
Answer: (A), (B), (C)

3. If sin θ =
(m 2 − n 2 ) /(m 2 + n 2 ), then

m2 − n2 m2 + n2
(A) tan θ = (B) cosecθ =
2mn m2 − n2
m2 + n2 2mn
(C) sec θ = (D) cot θ =
2mn m2 + n2
Solution: We have

m2 − n2
sin (θ ) =
m2 + n2

m2 − n2 m2 − n2
(A) tan (θ )
= = (True)
(m 2 + n 2 )2 − (m 2 − n 2 )2 2mn

1 m2 + n2
=
(B) cosec (θ ) = (True)
sin (θ ) m 2 − n 2

1 m2 + n2
(C) =
sec (θ ) = (True)
cos (θ ) 2mn
1 2mn
(D) =
cot (θ ) = (False)
tan (θ ) m 2 − n 2

Answer: (A), (B), (C)

4. Which of the following are true?


(A) tan 20° + tan 72° + tan=88° tan 20° ⋅ tan 72° ⋅ tan 88°
(B) tan 51° + tan 62° + tan=67° tan 51° ⋅ tan 62° ⋅ tan 67°
(C) cot 27° + cot 32° + cot=
31° cot 27° ⋅ cos 32° ⋅ cot 31°
(D) 3 + tan 40° +=
tan 80° 3 tan 40° tan 80°

Solution Manual for Mathematics For JEE (Main & Advanced), Trigonometry Vector
Algebra Probability, Vol 2,
Copyright©2017 Wiley India Pvt. Ltd. All rights reserved
Solution: We know that, when A + B + C = 180°
tan ( A) + tan ( B) + tan (C ) = tan ( A) ⋅ tan ( B) ⋅ tan (C )

(A) Here 20° + 72° + 88° = 180°

tan (20°) + tan (72°) + tan (88


= °) tan (20°) ⋅ tan (72°) ⋅ tan (88°)

(B) tan (51°) + tan (62°) + tan (67


= °) tan (51°) ⋅ tan (62°) ⋅ tan (67°)
(C) Again, we know that, if A + B + C = 90°,

cot ( A) + cot ( B) + cot (C ) = cot ( A) ⋅ cot ( B) ⋅ cot (C )


cot (27°) + cot (32°) + cot (31
= °) cot (27°) ⋅ cot (32°) ⋅ cot (31°)
(D) tan (60°) + tan (40°) + tan (80
= °) tan (60°) ⋅ tan (40°) ⋅ tan (80°)

Answer: (A), (B), (C), (D)

5. Which of the following are true?


3
(A) cos10° ⋅ cos 30° ⋅ cos 50° ⋅ cos 70° =
16
1
(B) cos 20° ⋅ cos 40° ⋅ cos 60° ⋅ cos80° =
16
(C) tan 20° ⋅ tan 40° ⋅ tan 80° = 3
(D) cos 55° + cos 65° + cos175° = 0

Solution:
3 1
(A) ⋅ (2 ⋅ cos10°⋅ cos 50°⋅ cos 70°)
2 2
3
= (cos 60° + cos 40°) cos 70°
4
3  cos 70° 1 
= + (cos110° + cos 30°) 
4  2 2 
3 3 3
= ⋅ =
4 4 16
1 1
(B) ⋅ (2 ⋅ cos 20°⋅ cos 40°⋅ cos80°)
2 2

1
=(cos 60° + cos 20°) cos80°
4
11 1 1 
=  cos80° + cos100° + cos 60° 
42 2 2 
1
=
16
(C) tan 20°⋅ tan (60 − 20°) tan (60 + 20)°

Solution Manual for Mathematics For JEE (Main & Advanced), Trigonometry Vector
Algebra Probability, Vol 2,
Copyright©2017 Wiley India Pvt. Ltd. All rights reserved
 tan (60° − 20°) + tan (60° + 20)° 
tan 20°  −1
 tan (120°) 
 sin (120°) 
tan 20°  −1 =3
 tan (120°) 
(D) cos 55° + cos 65° cos(175°)

2 cos (60)° ⋅ cos (115°) + cos 65°


cos (180 − 65)° + cos (65°) − cos 65° + cos (65°) = 0
Answer: (A), (B), (C), (D)

6. Which of the following are true?


(A) If tan(a +=θ ) n tan(a − θ ), then sin 2θ / sin 2α =
(n − 1) /(n + 1)
sin θ n sin(θ + 2α ), then (1 − n) tan(θ + aa
(B) If= ) =(1 + n) tan
π and sin[ A + (C / 2)] =
(C) If A + B + C = n sin(C / 2), then
tan( A / 2) ⋅ tan( B / 2) =
(n − 1) /(n + 1)
π , then tan( A / 2) + tan( B / 2) + tan(C / 2) =
(D) If A + B + C = 1

Solution:
tan (a + θ )
(A) =n
tan (a − θ )
tan (a + θ ) − tan (a − θ ) n −1
=
tan (a + θ ) + tan (a − θ ) n + 1

sin (2θ ) n −1
=
sin (2α ) n + 1

sin ( θ )
(B) =n
sin (θ + 2α )
sin ( θ + 2α ) − sin (θ ) 1 − n
=
sin (θ + 2α ) + sin (θ ) 1 + n
2 ⋅ cos ( θ + α ) ⋅ sin (α ) 1 − n
=
2 ⋅ sin (θ + α ) ⋅ cos (α ) 1 + n
(1 + n) tan ( a ) =
(1 − n) tan (θ + a )
sin ( A + C / 2)
(C) =n
sin (C / 2)
sin ( A + C / 2) − sin (C / 2) n −1
=
sin ( A + C / 2) + sin (C / 2) n + 1
2 ⋅ cos ( A + C / 2) ⋅ sin ( A / 2) n −1
=
2 ⋅ sin ( A + C / 2) ⋅ cos (C / 2) n + 1
 A  A + C  n −1
tan   ⋅ cot  =
 2  2  n+1

Solution Manual for Mathematics For JEE (Main & Advanced), Trigonometry Vector
Algebra Probability, Vol 2,
Copyright©2017 Wiley India Pvt. Ltd. All rights reserved
 A  B  n −1
tan   ⋅ tan   =
2  2  n +1
(D) Wrong:
Answer: (A), (B), (C)

7. Which of the following are (is) irrational?

(A) sin15° (B) cos15°


(C) sin15° ⋅ cos15° (D) sin15° ⋅ cos 75°
Solution:
1 − cos 30° 2− 3
(A) =
sin15° =
2 2
(B) cos15=° 1 − sin 2 115°
2+ 3
=
2
4− 3 1
(C) sin15°⋅ cos15
= ° = False
4 4
2− 3
(D) sin15° ⋅ cos 75° =
4
Answer: (A), (B), (D)

8. If p = sin(α − β )sin(γ − δ ), q = sin(β − γ )sin(α − δ ) and r =


sin(γ − α )sin(β − δ ), then

(A) p + q + r =0 (B) p + q − r =0
(C) q + r − p =0 (D) p3 + q3 + r 3 − 3 pqr =
0
Solution:
p = (sin α ⋅ cos β − cos α ⋅ sin β )(sin γ ⋅ cos δ − cos α ⋅ sin δ )
q = (sin β ⋅ cos γ − sin γ ⋅ cos β )(sin α ⋅ cos δ − cos α ⋅ sin δ )
r = (sin γ ⋅ cos α − sin α ⋅ cos γ )(sin β ⋅ cos δ − cos β ⋅ sin δ )
p+q+r =0
(D) Now, as p + q + r =0
so, p 3 + q 3 + r 3 =
0
Answer: (A), (D)

9. If 0 ≤ θ ≤ π / 2,=x α cos θ + β sin θ=


, y α sin θ − β cos θ and x 2 + 4 xy + y 2 = aab
2
+ b 2,
then

π
(A) a =
−1, b =
3 (B) θ =
4
π
(C) a = 3, b = −1 (D) θ =
3

Solution Manual for Mathematics For JEE (Main & Advanced), Trigonometry Vector
Algebra Probability, Vol 2,
Copyright©2017 Wiley India Pvt. Ltd. All rights reserved
Solution:
( x 2 + 2 xy + y 2 ) + 2 xy
( x + y ) 2 + 2 xy
[α (cos θ + sin θ ) + β {sin θ − cos(θ )}] 2 + 2(α cos θ + β sin θ )(α sin θ − β cos θ )
= [α 2 + α 2 sin 2θ + β 2 − β 2 sin 2θ + 2αβ (sin 2 θ − cos 2 θ )
+ 2αβ (sin 2 θ − cos 2 θ ) + α 2 sin 2θ − β 2 sin 2θ ]
=α 2 + β 2 + 2α 2 sin 2θ − 2β 2 sin 2θ + 4αβ cos 2θ
= aab 2
+b 2
π
Now, for θ = , we get
4
a = 3, b = −1

Answer: (B), (C)

n
∑ am sin(mx) is an identity in x, then
10. If cos3 x ⋅ sin 2 x =
m =1

=
(A) a3 3=
/ 8, a2 0 =
(B) n 6,=
a1 1 / 2
n
=
(C) n 5,=
a1 1 / 4 (D) ∑ am = 3/ 4
m =1
Solution: Since
=
cos 3x 4 cos3 x − 3cos x
Therefore,
1
=
cos3 x (cos 3x + 3cos x)
4
Now
cos3 x sin 2 x
1
= − sin 2 x(cos 3x + 3cos x)
4
1 3
= (2 cos 3x sin 2 x) + (2sin 2 x ⋅ cos x)
8 8
1 3
= (sin 5 x − sin x) + (sin 3x + sin x)
8 8
1 3 1
= sin x + sin 3x + sin 5 x
4 8 8
n
= ∑ am sin mx
m =1
Therefore,
1 3 1
=
n 5, a=
1 , a=
3 , a=
5 and a=
2 a=
4 0
4 8 8
Answer: (A), (C), (D)
11. If 7 cos x − 24sin x = λ cos( x + θ ) and 0 < θ < π / 2, then

Solution Manual for Mathematics For JEE (Main & Advanced), Trigonometry Vector
Algebra Probability, Vol 2,
Copyright©2017 Wiley India Pvt. Ltd. All rights reserved
(A) λ = 25 (B) cos θ = 7 / 25
(C) sin θ = 24 / 25 (D) λ = 15
Solution:
(7)2 + (24)2 =
25
 7 24 
25  cos ( x) − sin ( x) 
 25 25 
Let
7
= cos(θ )
25
then
24
= sin(θ )
25
25(cos ( x + θ=
) ) λ cos ( x + θ )
(A) Here, λ = 25
7
(B) cos (θ ) =
25
24
(C) sin (θ ) =
25
Answer: (A), (B), (C)

12. If 1 + cos( x − y ) =
0, then

(A) cos x − cos y =0 (B) cos x + cos y =


0
(C) sin x + sin y =
0 (D) cos x + sin y =
1
Solution: We have
1 + cos ( x − y ) = 0
cos ( x − y ) = −1
cos ( x − y ) = cos (π )
x− y= π
x (π + y )
=
sin ( x) = sin (π + y ) = − sin y
sin x + sin y = 0
cos ( x) = cos (π + y ) = − cos y
=
cos ( x) cos= ( y) 0

Answer: (B), (C)

13. If=y sin x ⋅ sin(60° − x)sin(60° + x), then

(A) Minimum value of y is −1 / 4


(B) Maximum value of y is 1
(C) Minimum value of y is −1
(D) y < 1
Solution: We have
y =sin ( x) ⋅ sin (60 − x) ⋅ sin (60 + x)
=y sin ( x)[sin 2 (60) − sin 2 ( x)]

Solution Manual for Mathematics For JEE (Main & Advanced), Trigonometry Vector
Algebra Probability, Vol 2,
Copyright©2017 Wiley India Pvt. Ltd. All rights reserved
3 
= sin ( x)  − sin 2 ( x) 
4 
1
= [3sin ( x) − 4sin 3 ( x)]
4
1
y = [sin (3x)]
4
We know that −1 ≤ sin (3x) ≤1 , so,
1 1
− ≤ y≤
4 4
Answer: (A), (D)

14. If (cos 2 x + sec2 x)(1 + tan 2 2 y )(3 + sin 3z ) =


4, then
(A) x can be a multiple of π
(B) x ≠ 2nπ , n ∈ 
π
(C) z = (4n − 1) , n ∈ z
6

(D) y =
2

Solution Since
cos 2 x + sec 2 x ≥ 2
1 + tan 2 2 y ≥ 1
3 + sin 3z ≥ 2
Therefore,
(cos 2 x + sec2 x)(1 + tan 2 2 y )(3 + sin 3z ) ≥ 4
here equality is hold when
cos 2 x =1 ⇒ x =nπ , n ∈ z

tan 2 2 y = 0 ⇒ y = ,n∈ z
2
π
sin 3z =−1 ⇒ z =(4n − 1) ,n∈ z
6
Answer: (A), (C), (D)

π , which of the following are true?


15. If A + B + C =
(A) sin 2 A + sin 2 B + sin 2C = 4sin A sin B sin C
(B) sin 2 A + sin 2 B − sin 2C = 4 cos A cos B sin C
(C) cos A + cos B + cos C = 1 + 4sin( A / 2)sin( B / 2)sin(C / 2)
(D) cot B cot C + cot C cot A + cot A cot B =
1

Solution:
(A) sin 2 A + sin 2 B + sin 2C
2sin ( A + B) ⋅ cos ( A − B) + 2sin (C ) cos(C )

Solution Manual for Mathematics For JEE (Main & Advanced), Trigonometry Vector
Algebra Probability, Vol 2,
Copyright©2017 Wiley India Pvt. Ltd. All rights reserved
2sin C[cos ( A − B) − cos( A + B)]
= 4sin A ⋅ sin B ⋅ sin C

(B)
sin (2A) +sin (2B) − sin (2C )
=2sin (A + B) ⋅ cos(A − B) − 2sin (C ) ⋅ cos (C )
=2sin (C )(cos (A − B) + cos (A + B))
= 4sin (C ) cos A cos B
(C)
cos( A) + cos( B) + cos(C ) =
 A+ B   A− B  2C
2 cos   ⋅ cos  2  + 1 − 2sin  2 
 2     
 C   A − B   A + B 
2sin   ⋅   − cos   +1
 2   2   2 
 A B C
1 + 4sin   ⋅ sin   ⋅ sin  
 
2  
2 2
(D)
cot B ⋅ cot C + cot C ⋅ cot A + cot A ⋅ cot B = 1
(cot A + cot B)
cot C (cot B + cot A) = 1 − cot A ⋅ cot B ⋅ cot (C ) =
−1
1 − cot A.cot B
tan (C ) = − tan ( A + B) = − tan(π − C ) = tan C
Answer: (A), (B), (C), (D)
16. In a triangle ABC, which of the following are true?
(A) tan A + tan B + tan C = tan A tan B tan C
A B C
(B) tan 2 + tan 2 + tan 2 ≥ 1
2 2 2
sin 2 A + sin 2 B + sin 2C A B C
(C) = 8sin sin sin
sin A + sin B + sin C 2 2 2
A B C A B C
(D) cot + cot + cot = cot cot cot
2 2 2 2 2 2

Solution:
(A)
tan A + tan B + tan
= C tan A ⋅ tan B ⋅ tan C
(tan A + tan
= B) tan C (tan A ⋅ tan B − 1)
tan A + tan B
tan (C ) = − =
− tan ( A + B)
1 − tan A ⋅ tan B
= tan (C )
(B)

Solution Manual for Mathematics For JEE (Main & Advanced), Trigonometry Vector
Algebra Probability, Vol 2,
Copyright©2017 Wiley India Pvt. Ltd. All rights reserved
2
 A B C
 tan 2 − tan 2 tan 2 
 
A B C  A B
= tan 2 + tan 2 + tan 2 − 2  tan ⋅ tan  ,
2 2 2  2 2
A B C  A B C
tan 2 + tan 2 + tan 2 =  tan − tan − tan  + 2(.1)
2 2 2  2 2 2
A B C
tan 2 + tan 2 + tan 2 ≥1
2 2 2
(C)
sin 2 A + sin 2 B + sin 2C
sin A + sin B + sin C
4sin A ⋅ sin B ⋅ sin C A B C
= = 8sin ⋅ sin ⋅ sin
sin A + sin B + sin C 2 2 2
(D)
A B C
cot + cot + cot
2 2 2
A B C
= cot ⋅ cot ⋅ cot
2 2 2

Answer: (A), (B), (C), (D)

MATRIX-MATCH TYPE QUESTIONS


In each of the following questions, statements are given in two columns, which have to be
matched. The statements in column I are labeled as (A), (B), (C) and (D), while those in column II
are labeled as (p), (q), (r), (s) and (t). Any given statement in column I can have correct matching
with one or more statements in column II. The appropriate bubbles corresponding to the answers
to these questions have to be darkened as illustrated in the following example.
Example: If the correct matches are (A) → (p), (s), (B) → (q), (s), (t), (C) → (r), (D) → (r), (t) ,
that is if the matches are (A) → (p) and (s); (B) → (q), (s) and (t); (C) → (r); and (D) → (r), (t),
then the correct darkening of bubbles will look as follows:

1. Match the items in Column I with those in Column II.

Column I Column II
 A (p) sec A + tan A
(A) tan  45° + 
 2
 π A  π A 1
(B) sin 2  +  − sin 2  −  (q) sin A
 8 2  8 2 2

(C) (cos A + cos B) 2 + (sin A + sin B) 2  A − B


(r) 4 cos 2 
 2 

Solution Manual for Mathematics For JEE (Main & Advanced), Trigonometry Vector
Algebra Probability, Vol 2,
Copyright©2017 Wiley India Pvt. Ltd. All rights reserved
(D) (cos A − cos B) 2 + (sin A − sin B) 2 1 + sin A
(s)
1 − sin A
 A − B
(t) 4sin 2 
 2 
Solution:
(A)
 A  1 + tan A / 2
tan  45° +  =
 2  1 − tan A / 2
cos A / 2 + sin A / 2 (cos A / 2 + sin A / 2)
×
cos A / 2 − sin A / 2 (cos A / 2 + sin A / 2)
1 + sin A
=
cos A
Answer: (p), (s)
(B)
π A π A π A π A
sin  + + −  sin  + − + 
8 2 8 2 8 2 8 2
π  1
sin   ⋅ sin( A) = sin A
4 2

Answer: (q)
(C)
cos 2 A + cos 2 B + sin 2 A + sin 2 B +
2(cos ( A − B))
A−B
=2[1 + cos ( A − B)] = 4cos 2
2

Answer: (r)

(D)
 A− B 
(cos A − cos B) 2 + (sin A − sin B) 2 =
4sin 2  
 2 

Answer: (t)

2. Match the items in Column I with those in Column II.

Column I Column II
(A) cos 2 θ + cos 2 (θ + 120°) + cos 2 (θ − 120°) (p)
3
2

Solution Manual for Mathematics For JEE (Main & Advanced), Trigonometry Vector
Algebra Probability, Vol 2,
Copyright©2017 Wiley India Pvt. Ltd. All rights reserved
π 3π 5π 7π (q) 2
(B) cos 4 + cos 4 + cos 4 + cos 4
8 8 8 8

π 3π 5π 7π 1
(C) sin 4 + sin 4 + sin 4 + sin 4 (r)
8 8 8 8 16

π 13π −1
(D) sin ⋅ sin (s)
10 10 4
1
(t)
4
Solution:
(A)
Since
1
θ
cos 2= (1 + cos 2θ )
2
Therefore,
1
cos 2 (θ + 120°=
) [1 + cos(2θ + 240°)]
2
1
and cos 2 (θ − 120°=
) [1 + cos(2θ − 240°)]
2
So,
cos 2 θ + cos 2 (θ + 120) + cos 2 (θ − 120)
3 1
=+ [cos 2θ + cos(2θ + 120°) + cos(2θ − 120°)]
2 2
3 1 1 
=+  cos 2θ + cos 2θ ⋅ cos120° 
2 2 2 
3 1 3
=+ (cos 2θ − cos 2θ ) =
2 2 2
Answer: (p)
(B)
3π π 5π 7π
cos 4 + cos 4
+ cos 4 + cos 4
8 8 8 8
π 3π 3π π
=cos 4 + cos 4 + cos 4 + cos 4
8 8 8 8
 π 3π 
= 2  cos 4 + cos 4 
 8 8 
 π π
= 2  cos 4 + sin 4 
 8 8
 π π
= 2 1 − 2sin 2 cos 2 
 8 8
π π
= 2 − 4sin 2 cos 2
8 8
 π
2
3
=
2 −  sin  =
 4  2
Answer: (p)
(C)

Solution Manual for Mathematics For JEE (Main & Advanced), Trigonometry Vector
Algebra Probability, Vol 2,
Copyright©2017 Wiley India Pvt. Ltd. All rights reserved
3π π 5π 7π
sin 4 + sin 4
+ sin 4 + sin 4
8 8 8 8
π 3π 3π π
=sin 4 + sin 4 + sin 4 + sin 4
8 8 8 8
 π 3π 
= 2  sin 4 + sin 4 
 8 8 
 π π
= 2  sin 4 + cos 4 
 8 8
 π π 3
=2 1 − 2sin 2 cos 2  =
 8 8 2
Answer: (p)
(D)
π 13π
sin ⋅ sin
10 10
π  3π 
= sin ⋅ sin  π + 
10  10 
π 3π
= − sin ⋅ sin
10 10
= − sin18° ⋅ sin 54°
= − sin18° ⋅ cos 36°
5 − 1 5 + 1 −1
=
− ⋅ =
4 4 4
Answer: (s)

π , match the items in Column I with those in Column II.


3. Assuming that A + B + C =

Column I Column II
(A) cos 2 A + cos 2 B + cos 2 C A B
(p) 4 cos cos cos
C
2 2 2
(B) cos A + cos B + cos C − 1 A B C
(q) 4sin cos sin
2 2 2

(C) sin A − sin B + sin C (r) 1 − 2 cos A cos B cos C


(D) sin A + sin B + sin C A B C
(s) 4sin sin sin
2 2 2
Solution:
(A)
1
(1 + cos 2 A + 1 + cos 2 B + 1 + cos 3C )
2
= + [1 + cos 2 A + 2 cos( B + C ) ⋅ cos( B − C ) ]
1
2
= 1 + [2 cos 2 A − 2 cos ( A) ⋅ cos ( B − C )]
=
1 − 2 cos A ⋅ cos B ⋅ cos C

Solution Manual for Mathematics For JEE (Main & Advanced), Trigonometry Vector
Algebra Probability, Vol 2,
Copyright©2017 Wiley India Pvt. Ltd. All rights reserved
Answer: (r)

(B)
cos A + cos B + cos C −1
 A+ B   A− B  2 C
2 cos   ⋅ cos  2  − 2sin 2
 2   
C   A− B   A + B 
2sin cos   − cos  
2  2   2 
 A B C
4sin   ⋅ sin   ⋅ sin  
2 2 2

Answer: (s)

(C)
sin A − sin B + sin C
 A+ B   A− B  C C
2 cos   ⋅ sin   + 2sin ⋅ cos ⋅
 2   2  2 2
C   A− B   A + B 
2sin sin   + sin  
2  2   2 
A B C
= 4sin ⋅ cos ⋅ sin
2 2 2

Answer: (q)

(D)
sin A + sin B + sin C
A+ B A− B C C
= 2sin
⋅ cos + 2sin ⋅ cos
2 2 2 2
C A− B A+ B 
= 2 cos cos + cos
2 2 2 
A B C
= 4 cos ⋅ cos ⋅ cos
2 2 2

Solution Manual for Mathematics For JEE (Main & Advanced), Trigonometry Vector
Algebra Probability, Vol 2,
Copyright©2017 Wiley India Pvt. Ltd. All rights reserved
Answer: (p)

Let θ 22(1/ 2)°. Match the items in Column I with those in Column II.
4. =

Column I Column II
(A) sin θ (p) 2 + 1

(B) cos θ (q) 2 −1

(C) tan θ 1
(r) 2+ 2
2

(D) cot θ 1
(s) 2− 2
2
1
(t) ( 2 + 1)
2

Solution:
1 − cos 2θ 2− 2
(A)sin θ = =
2 2
1 + cos (2θ ) 2+ 2
(B) cos (θ )
= =
2 2
sin (θ )
(C) tan (θ=) = 2 −1
cos (θ )
cos (θ )
(D) cot (θ=) = 2 −1
sin (θ )
Answer: (A) → s, (B) → r, (C) → q, (D) → p
5. Match the items in Column I with those in Column II.

Column I Column II
(A) If tan( x − y ) / 2, tan z , tan( x + y ) / 2 are in GP, then (p) cos 2θ
cos x(dy / dx) is equal to
(B) If sin x, sin θ , cos x are in GP, then 2 cos 2 [(π / 4) + x] is 3 sin θ
(q)
equal to 2 + cos θ
(C) If sec(θ − x), sec θ , sec(θ + x) are in AP, then cos θ is x
(r) 2 cos
equal to 2
(D) If cos θ= (2 cos x − 1) /(2 − cos x), then sin x is equal to 2
(s) sin x
2

Solution Manual for Mathematics For JEE (Main & Advanced), Trigonometry Vector
Algebra Probability, Vol 2,
Copyright©2017 Wiley India Pvt. Ltd. All rights reserved
(t) cos y cos 2 z
Solution:
(A)
( x − y)  x+ y 
=
tan 2 ( z ) tan ⋅ tan  
2  2 
sin 2 ( z ) sin( x − y / 2) ⋅ sin( x + y / 2)
=
cos ( z ) cos( x − y / 2) ⋅ cos( x + y / 2)
2

1 cos[ x − y / 2 − ( x + y / 2)]
=
cos ( z ) − sin ( z ) cos( x − y / 2 + x + y / 2)
2 2

1 cos( y )
=
cos(2 z ) cos( x)
=
cos( x) cos y ⋅ cos 2 z

Answer: (t)

(B)
sin=
2
(θ ) sin ( x) ⋅ cos ( x)
π 
2sin=
2
(θ ) sin=
(2 x) cos  + 2 x 
z 
π
2 cos 2 + x =1 − 2sin 2 (θ ) =cos (2θ )
4

Answer: (p)

(C)
2sec (θ=) sec (θ − x) + sec (θ + x)
2 cos(θ + x) + cos(θ − x)
=
cos (θ ) cos(θ − x) ⋅ cos(θ + x)
2[(cos 2 (θ ) − sin 2 ( x)]
2 cos θ ⋅ cos x
=⋅
cos(θ )
sin 2 ( x) x
cos 2 (θ )
= = 2 cos 2
1 − cos( x) 2
cos x
cos (θ ) = 2
2

Answer: (r)

(D)

Solution Manual for Mathematics For JEE (Main & Advanced), Trigonometry Vector
Algebra Probability, Vol 2,
Copyright©2017 Wiley India Pvt. Ltd. All rights reserved
cos (θ ) 2 cos ( x) −1 2 cos( x) −1
= =
2 2[(2 − cos( x)] 4 − 2 cos ( x)
cos(θ ) 2 cos ( x) −1
=
2 + cos(θ ) 3
1  3cos (θ ) 
=cos ( x)   +1
2  2 + cos (θ ) 

sin ( x=) 1 − cos 2 ( x)


3 sin (θ )
=
2 + cos (θ )

Answer: (q)

6. Match the items in Column I with those in Column II.

Column I Column II
tan( A − B) + tan B (p) tan A
(A)
1 − tan( A − B) tan B
tan 4 A − tan 3 A (q) cot A
(B)
1 − tan 3 A ⋅ tan 4 A

(C) sin2 A(1 + cot 2 A ⋅ cot A) (r) tan 2A


(D) cos 2 A(tan 2 A + cos A ⋅ cosec A) (s) cot 2A

Solution:
(A) tan ( A − B + B) =
tan A
Answer: (p)
(B) tan(4 A − 3 A) =
tan A
Answer: (p)

(C)
sin 2 A(cot 2 A − cot A)
cot A
2sin A ( sin A )
= = cot A
sin A .sin 2 A
Answer: (q)

(D)
Solution Manual for Mathematics For JEE (Main & Advanced), Trigonometry Vector
Algebra Probability, Vol 2,
Copyright©2017 Wiley India Pvt. Ltd. All rights reserved
cot 2 A (tan 2 A + cos A cosec A)
= cot A
Answer: (q)

COMPREHENSION TYPE QUESTIONS


c can be solved when | c |≤ a + b . Based on this
1. Passage: The equation a cos θ + b sin θ = 2 2

answer the following questions (i), (ii) and (iii).


(i) If 0 ≤ x ≤ π and sin x + cos x = 1, then the number of values of x is

(A) 1 (B) 2 (C) 4 (D) 0


(ii) The maximum and minimum values of 3cos θ + 4sin θ − 5 are, respectively (θ is real),

(A) 5, −5 (B) 4, 3 (C) 0, −10 (D) 4, −3


(iii) The maximum and minimum values of 5sin θ + 12 cos θ + 13 are, respectively (θ is real),

(A) 13, 0 (B) 1, −1 (C) 26, 0 (D) 26, 13


Solution:
(i)
sin( x) + cos( x) =
1
 1 1 
2 sin x + cos x  =
1
 2 2 
 π
2 sin  x +  = 1
 4
 π 1 π 3π
sin  x +  = = sin or sin
 4  2 4 4
π π 3π
x + = or
4 4 4
π
x = 0 or .
2

Answer: (B)

(ii)
3 4 
y= ± 32 + 42  cos θ + sin θ (r )  − 5
5 5 
=±5[sin(θ + α )] − 5
Now
−1 ≤ sin(θ + α ) ≤ 1
so,
−5−5< y ≤5−5
−10 ≤ y ≤ 0

Answer: (C)

(iii)

Solution Manual for Mathematics For JEE (Main & Advanced), Trigonometry Vector
Algebra Probability, Vol 2,
Copyright©2017 Wiley India Pvt. Ltd. All rights reserved
 5 12 
y= ±13  sin θ + cos θ  + 13
 13 13 
=±13(sin (θ + α )) + 13
Now
−1 ≤ sin(θ + α ) ≤ 1
−13 + 13 ≤ y ≤ 13 + 13
0 ≤ y ≤ 26

Answer: (C)

2. Passage: If α, β and γ are roots of the equation ax3 + bx 2 + cx + d =0 and a ≠ 0, then


ab+ + γ =−b / a, abbγ + + γa =c / a and aβγ = − d / a. It is given that
cos π / 7, cos 3π / 7 and cos 5π / 7 are roots of 8 x3 − 4 x 2 − 4 x + 1 =0.
Answer questions (i), (ii) and (iii).
(i) The value of sec(π / 7) + sec(3π / 7) + sec(5π / 7) is

(A) 4 (B) 2 (C) 8 (D) 6


(ii) The value of sin(π /14)sin(3π /14)sin(5π /14) is

7 1 7 1
(A) (B) (C) (D)
8 8 4 4

(iii) The value of cos (π /14) cos (3π /14) cos (5π /14) is

1 1 7 7
(A) (B) (C) (D)
8 4 4 8
Solution:

(i)
π   3π   5π 
sec   + sec   + sec  
7  7   7 
cos(3π / 7) cos(5π / 7) + cos(5π / 7) cos(5π / 7) + cos(π / 7) cos(3π / 7)
=
cos(π / 7) ⋅ cos(3π / 7) ⋅ cos(5π / 7)
−4 / 8
= = 4
−1/ 8

Answer: (A)

(ii)

Solution Manual for Mathematics For JEE (Main & Advanced), Trigonometry Vector
Algebra Probability, Vol 2,
Copyright©2017 Wiley India Pvt. Ltd. All rights reserved
2
 π   3π   5π  
y = sin   ⋅ sin 
2
 ⋅ sin  
  14   14   14  
π   3π   5π 
8 y 2 =2sin 2   ⋅ 2sin 2   ⋅ 2sin 2  
 14   14   14 
  π    3π    5π 
=
1 − cos    1 − cos    1 − cos  7 
   
14  7    
 π 3π 5π π 3π π 3π 5π 
= 1 − cos − cos − cos + Σ cos ⋅ cos − cos ⋅ cos ⋅ cos
 7 7 7 7 7 7 7 7 
 1 4 1 1
=1 +  −  − + =
 2 8 8 8
1 1
=y =
8 ∗8 8

Answer: (B)

(iii)
  π    3π     5π  
8 y2 = 1 + cos    1 + cos    1 + cos  
  7    7    7 
π π 3π π 3π 5π
= 1 + Σ cos + Σ cos ⋅ cos + cos ⋅ cos ⋅ cos
7 7 7 7 7 7
1  1 1 7
=1 +   +  −  − =
2  2 8 8
7
y=
8

Answer: (D)

3. Passage: To eliminate a parameter, we need two equations involving the parameter. For
example, if x = r cos θ and y = r sin θ , then by squaring and adding, we have x 2 + y 2 =
r2.
This shows that θ is eliminated from the given equations.

Answer the following questions (i), (ii) and (iii).


a and tan θ − sin θ =
(i) If tan θ + sin θ = b, then after eliminating θ , (a 2 − b 2 )2 is equal to

(A) 4 ab (B) 4ab (C) 16 ab (D) 16ab


(ii) Eliminating θ from the equations
= =
x cot θ + tan θ and y sec θ − cos θ , we have

(A) xy ( x + y ) =
1 (B) ( x 2 y ) 2/3 − ( xy 2 )2/3 =
1
(C) x 2/3 − y 2/3 =
1 (D) xy = 1
(iii) Eliminating θ from the equations x= / b sin θ + sin 2θ , which
/ a cos θ + cos 2θ and y=
of the following relations between x and y can be obtained?

Solution Manual for Mathematics For JEE (Main & Advanced), Trigonometry Vector
Algebra Probability, Vol 2,
Copyright©2017 Wiley India Pvt. Ltd. All rights reserved
2 x  x2 y 2   x2 y 2 
(A) =  2 + 2   2 + 2 − 3
a a b a b 
x2 y2 2x
(B) + = +3
a2 b2 a
x2 y2
x
(C) 2
+ = +3
2
a b a
x 2
y   x2 y 2
2   x y
(D)  2 + 2   2 + 2 − 3= 2  + 
a b a b   a b
Solution:

(i)
(a 2 − b 2 )2
= [{tan θ + sin(θ )}2 − {tan θ − sin(θ )2 }]2 (4sin θ ⋅ tanθ )2
2
 sin 2 θ  sin 4 (θ )
= 16=  16
 cos θ  cos 2 (θ )
  a + b 2   a − b 2
=16 1 +    ⋅ 
  2    2 
= 16ab

Answer: (D)

(ii)
sec4 (θ ) tan 2 θ
x2 y = ⋅ = sec3 (θ )
tan 4 θ sec(θ )
tan 4 (θ ) sec2 (θ )
xy 2 = ⋅ = tan 3 (θ )
sec2 (θ ) tan(θ )
Now
= [sec3 (θ )]2/3 − [tan 3 (θ )]2/3
( x 2 y ) 2/3 − ( xy 2 ) 2/3
=1

Answer: (B)

(iii)
x2 y2
+ =(2 + 2 cos 3θ )
a2 b2
 x2 y 2 
and  2 + 2 =− 3  (2 cos 3θ −1)

a b 
Now
2x
2(1 + cos 3θ ) (2 cos 3θ −1) =
a

Answer: (A)

Solution Manual for Mathematics For JEE (Main & Advanced), Trigonometry Vector
Algebra Probability, Vol 2,
Copyright©2017 Wiley India Pvt. Ltd. All rights reserved
ASSERTION–REASONING TYPE QUESTIONS
In the following set of questions, a Statement I is given and a corresponding Statement II is given
just below it. Mark the correct answer as:
(A) Both Statements I and II are true and Statement II is a correct explanation Statement I.
(B) Both Statements I and II are true but Statement II is not a correct explanation for Statement I.
(C) Statement I is true and Statement II is false.
(D) Statement I is false and Statement II is true.
1. Statement I: If x cos aa
+ y sin = x cos β + y sin β = 2a and 2sin α / 2sin β / 2 = 1, then
=
y 2 4a ( a − x )

Statement II: If x1 and x2 are roots of the quadratic equation ax 2 + bx + c =0, then
x1 + x2 = −b / a and x1 x2 = c / a
Solution:
(4a)2 = ( x cos aa
+ y sin )( x cos β + y sin β )
= x 2 ⋅ cos a ⋅ cos β + y 2 ⋅ sin a ⋅ sin β + ( xy )(sin(a + β ))
Now
=4ax x( x cos aa
+ y sin )
Now
(4a 2 − 4ax) =
x 2 (cos α .cos β ) + y 2 (sin α .sin β ) + xy sin (α + β )
−( x 2 cos + xy sin α )
= y2

Answer: (A)

=
2. Statement I: If and y cosecθ − sin θ , then ( x 2 y ) 2/3 − ( xy 2 )2/3 =
x cot θ + tan θ = 1

Statement II: The conclusion of statement I is that cosec2θ − cot 2 θ =


1
Solution:
(sec4 θ ) cos 2 θ
x2 y = ⋅ = sec3 (θ )
tan θ sin θ
2

2
sec2 (θ )  cos 2 θ 
xy 2 = ⋅ tan 3 (θ )
 =
tan θ  sin θ 
2 2

( x y)
2 3 −(y 2
x) 3 = sec2 (θ ) − tan 2 (θ ) = 1

Answer: (A)
3. Statement I: For all values of θ , 2(sin θ + cos θ ) − 3(sin θ + cos θ ) =
6 6
−1 4 4

Solution Manual for Mathematics For JEE (Main & Advanced), Trigonometry Vector
Algebra Probability, Vol 2,
Copyright©2017 Wiley India Pvt. Ltd. All rights reserved
Statement II: x3 + y 3 = ( x + y )( x 2 − xy + y 2 )
Solution:
2(sin 6 θ + cos6 θ ) − 3(sin 4 θ + cos 4 θ ) (sin 2 θ + cos 2 θ )
2(sin 6 θ + cos6 θ ) − 3(sin 6 θ + cos6 θ + sin 4 θ ⋅ cos 2 θ + sin 2 θ ⋅ cos 4 θ )

− [sin 2 θ + cos 2 θ ]3 =
−1

Answer: (A)
4. Statement I: If tan(θ / 2) = m, then

1 − 2sin 2 (θ / 2) 1 + m
=
1 + sin θ 1− m
2 tan θ 1 − tan 2 θ
Statement II: sin 2θ = and cos 2θ =
1 + tan 2 θ 1 + tan 2 θ
Solution: Statement II is direct formula.
2sin θ ⋅ cos θ ⋅
=
sin 2θ = sin 2θ ⋅
sin 2 θ + cos 2 θ
cos 2 θ − sin 2 θ
=
cos 2θ = cos 2θ ⋅
cos 2 θ + sin 2 θ
Statement I
θ 
tan   = m
2
1 + tan θ / 2 1 + m
= False
1 − tan θ / 2 1 − m
1 + sin θ 1 + m
= False
cos θ 1− m

Answer: (D)
5. Statement I: If

sin 3α 11
=
sin α 25
then tan(a / 2) has four values.
2 tan(θ / 2)
Statement II: sin θ =
1 + tan 2 (θ / 2)
Solution:
3sin α − 4sin 3 α 11
=3 − 4sin 2 α =
sin α 25
11
3− = 2
4sin (2)
25
64 4
=sin α =
25 ⋅ 4 5
2 tan(a / 2) 4
= sin a =
1 + tan (a / 2)
2
5

Solution Manual for Mathematics For JEE (Main & Advanced), Trigonometry Vector
Algebra Probability, Vol 2,
Copyright©2017 Wiley India Pvt. Ltd. All rights reserved
 aa

=
10 tan   4 tan
2
+4
2 2
aa
4 tan 2 −10 tan + 4 = 0
2 2
a 10 ± (10)2 − 4 ⋅ 4 ⋅ 4
tan =
2 2(4)
100 − 64 10 ± 6
=
10 ± =
8 8
a 1
tan = 2 or
2 4

Answer: (A)

6. Statement I: If 0 < α < π / 2 and cot α = 3, then the value of tan (a / 2) is 1/(3 + 10) .

aa
sin
Statement II: tan =
2 1 + cos a
Solution:
1
tan (a ) =
3
2 tan aa sin 1/ 10
= =
1 − tan a 1 + cos a 1 + 3/ 10
2

1
=
10 + 3

Answer: (A)

7. Statement I: If

3 3
tan A = and tan B =
4− 3 4+ 3
Then
tan( A − B) =
0.355 .
tan A − tan B
Statement II: tan( A − B) =
1 + tan A tan B

Solution:
Statement I
tan A − tan B 6
tan( A − B=) = = 0.6 False
1 + tan A ⋅ tan B 13 − 3
Statement II Direct formula

Answer: (D)

Solution Manual for Mathematics For JEE (Main & Advanced), Trigonometry Vector
Algebra Probability, Vol 2,
Copyright©2017 Wiley India Pvt. Ltd. All rights reserved
8. Statement I: If α + β + γ =
π / 2, then ∑ tan β tan γ =
1.

Statement II: If A + B + C = π , then tan A + tan B + tan C = tan A tan B tan C .


Solution:
π
Statement I α + β +γ =
2
π
α + β = −γ
2
⇒ tan(a + β ) = cot γ
tan a + tan β 1
⇒ =
1 − tan a tan β tan γ
⇒ tan a tan β + tan β tan γ + tan γ tan a = 1
Statement II A+ B+C = π
A + B =π − C
tan( A + B) = − tan C
tan A + tan B
= − tan C
1 − tan A tan B
⇒ tan A + tan B + tan C =
tan A tan B tan C

Answer: (B)

9. Statement I: If α + β + γ =
π / 2, then

sin 2α + sin 2β + sin 2γ


= cot α cot β
sin 2α + sin 2β − sin 2γ
Statement II: If A + B + C = π , then cos 2 A + cos 2 B + cos 2C =−1 − 4 cos A cos B cos C
Solution
Statement I
sin 2α + sin 2β + sin 2α
sin 2α + sin 2β − sin 2α
2sin (α + β ) ⋅ cos(α − β ) + sin 2α
=
2sin (α + β ) ⋅ cos(α − β ) − sin 2α
2cos(α )[cos(α − β ) + cos(α + β )]
= cot α ⋅ cot β
2cos(α )[cos(α − β ) − cos(α + β )]

Statement II ⋅
cos 2 A + cos 2 B + cos 2C =
2 cos( A + B) ⋅ cos( A − B) + 2 cos 2 c −1
− 2 cos (c)[cos( A − B) + cos ( A + B)] −1
=− 1 − 4 cos A ⋅ cos B ⋅ cos C ⋅ True

Answer: (B)

10. Statement I:

Solution Manual for Mathematics For JEE (Main & Advanced), Trigonometry Vector
Algebra Probability, Vol 2,
Copyright©2017 Wiley India Pvt. Ltd. All rights reserved
π , then
If A + B + C =
A B C  π − A  π − B π − C
cos + cos + cos =4 cos   cos   cos  
2 2 2  4 4 4 
Statement II: cos(α + β ) cos(α − β
= ) cos 2 α − cos 2 β for all α and β.
Solution
Statement I
A B C
cos + cos + cos
2 2 2
( A + B) A− B C
= 2 cos ⋅ cos − 2sin 2 + 1
4 4 4
C   A− B   A + B 
= 2sin cos   − cos   +1
2  2   2 
A B C
=
1 + 4sin ⋅ sin ⋅ sin
4 4 2
 π − A   π − B   π − C
= 4 cos   ⋅ cos   ⋅ cos   True
 4   4   4 
Statement II Direct formula and wrong one

Answer: (C)

INTEGER ANSWER TYPE QUESTIONS


The answer to each of the questions in this section is a non-negative integer. The appropriate
bubbles below the respective question numbers have to be darkened. For example, as shown in the
figure, if the correct answer to the question number Y is 246, then the bubbles under Y labeled as
2, 4, 6 are to be darkened.
X Y Z W
ⓞ ⓞ ⓞ ⓞ
① ① ① ①
② ② ②
③ ③ ③ ③
④ ④ ④
⑤ ⑤ ⑤ ⑤
⑥ ⑥ ⑥
⑦ ⑦ ⑦ ⑦
⑧ ⑧ ⑧ ⑧
⑨ ⑨ ⑨ ⑨
<COMP: In the empty squares under Y write 2, 4, 6 in a circle and shade them>
1. tan12° tan 24° tan 48° tan 84° is equal to ____.

Solution: We have
sin12°⋅ sin 24°⋅ sin 48°⋅ sin 84°
cos12°⋅ cos 24°⋅ cos 48°⋅ cos84°
(cos 60° − cos 36°) (cos108° − cos 60°)
=
(cos 60° + cos 36°) (cos108° + cos 60°)

Solution Manual for Mathematics For JEE (Main & Advanced), Trigonometry Vector
Algebra Probability, Vol 2,
Copyright©2017 Wiley India Pvt. Ltd. All rights reserved
+ [2 − ( 5) + 1]( 5 −1 + 2)
= = 1
(3 + 5) ( 5 −1 + 2)

Answer: (1)

2. If sin18° and cos 36° are roots of the equation ax 2 − 2 5 x + c =0, then the value of a + c is
____.

Solution:
C
sin18° ⋅ cos 36° =
a
( 5 − 1) ( 5 + 1) C
=
16 a
C 1
=
a 4
C+a 5
=
a 4
Now
2 5 5 −1 5 +1 2 5
= + =
a 4 4 4
a=4
5a 5 ⋅ 4
Hence =
C = = 5
4 4

Answer: (5)

3. tan 6° tan 42° tan 66° tan 78° is equal to ____.

Solution: We have
sin 6°⋅ sin 42°⋅ sin 66°⋅ sin 78°
cos 6°⋅ cos 42°⋅ cos 66°⋅ cos 78°
(cos 72° − cos 60°) (cos120° − cos36°)
=
(cos 72° + cos 60°) (cos120° + cos36°)
( 5 + 1 − 2) (− 2 − 5 −1)
= = 1
( 5 −1 − 2) (− 2 − 5 + 1)

Answer: (1)

sin 3α cos 3α
4. + k cos 2α , where k is equal to ____.
=
sin α cos α

Solution: We have
sin 3α ⋅ cos α + cos 3α ⋅ sin α
sin α ⋅ cos α
2sin 4α
= = 4sin 2α ⋅ cos 2α
sin 2α

Solution Manual for Mathematics For JEE (Main & Advanced), Trigonometry Vector
Algebra Probability, Vol 2,
Copyright©2017 Wiley India Pvt. Ltd. All rights reserved
K =4

Answer: (4)

5. (1 − 2sin θ )(1 + 2sin θ )(1 + 2 cos 2θ ) =


1 + k cos 4θ , where k is equal to ____.

Solution:
[1 − 4sin 2 (θ )](1 + 2 cos 2θ )
(cos 2θ + cos 2θ −1) (1 + 2 cos 2θ )
= 4 cos 2 2θ −1
1 + 2cos 4θ K =
= 2

Answer: (2)

6. If 2 tan( A + B) =
3tan A, then sin(2 A + B) =
k sin B, where k is equal to ____.

Solution: We have
tan ( A + B) 3
=
tan ( A) 2
sin ( A + B) ⋅ cos A 3
=
cos ( A + B) ⋅ sin A 2
By componendo and dividendo
sin ( A + B) ⋅ cos A + cos ( A + B) ⋅ sin A 3 + 2
= = 5
sin ( A + B) ⋅ cos A − cos ( A + B) ⋅ sin A 3 − 2
sin (2 A + B)
=5
sin ( B)
sin (2 A + B) =
5sin B

Answer: (5)

=
7. If 3cos x 2 cos( x − 2 y ), then cot( x − y ) cot y is equal to ____.

Solution: We have
cos ( x − 2 y ) 3
=
cos( x) 2
By componendo and dividendo
cos ( x − 2 y ) + cos ( x) 3 + 2
=
cos ( x − 2 y ) − cos ( x) 3 − 2
cos ( x − y ) ⋅ cos ( y )
= = 5
sin ( x − y ) ⋅ sin ( y )
cot ( x − y ) ⋅ cot y =
5
Answer: (5)

8. If sin 2° + sin 4° + sin 6° +  + sin178


= ° n cot1°, then n is equal to ____.

Solution:

Solution Manual for Mathematics For JEE (Main & Advanced), Trigonometry Vector
Algebra Probability, Vol 2,
Copyright©2017 Wiley India Pvt. Ltd. All rights reserved
2sin1°(5) = 2 ⋅ sin(1°)(sin 2° + sin 4° +  + sin178°)
= cos3° − cos1° + cos5° − cos3° + cos179° − cos177° 2sin(1°)( S ) = − 2cos(1)°
(S ) =
−1cot(1)°
n = −1
Answer: (−1)

9. Let 0 < α j < π / 2 for j = 1, 2, 3,,8 and cot α1 ⋅ cot α 2  cot α8 =


1. If M is the maximum
value of cot α1 cot α 2  cot α8 , then the value of (32) M is ____.

Solution:
π
Since 0 < α j < for j = 1, 2, 3,… and cot α1 ⋅ cot α 2  cot α8 =
1
z
π
⇒ Let α3 − α1
2
π
α 7= − α2
2
π
α 6= − α3
2
π
α 5= − α4
2
and = z cos α1 ⋅ cos α 2  cos α 3
= cos aaaaaaaa
1 cos 2 ⋅ cos 3 ⋅ cos 4 ⋅ sin 4 ⋅ sin 3 ⋅ sin 2 ⋅ sin 1

1
z= sin 2aaa
, ⋅ sin 2 2 ⋅ sin 2 3 sin
16
1 π
= M
zmax = ataaaa
= 1 =2 =
3 =4
16 4
⇒ 32 M = 2
Answer: (2)
10. The value of (1 + cot A − cosec A)(1 + tan A + sec A) is ____.

Solution:
(−1 + sin A + cos A) (sin A + cos A + 1)

sin A cos A
(sin A + cos A) 2 −1
=
sin A ⋅ cos A
2sin A ⋅ cos A
= = 2
sin A ⋅ cos A
Answer: (2)

3, then | cos 2 x + cos 2 y + cos 2 z | is equal to ____.


11. If sin x + sin y + sin z =

Solution
sin x + sin y + sin z =
3
That mean’s
=
sin =
x sin =
y sin z 1

Solution Manual for Mathematics For JEE (Main & Advanced), Trigonometry Vector
Algebra Probability, Vol 2,
Copyright©2017 Wiley India Pvt. Ltd. All rights reserved
π
x= y= z=
2
π  π  π 
cos 2   + cos 2   + cos 2   =3
 
2  
2 2
Answer: (3)

12. (sin aaaaaa


+ cosec ) 2 + (cos + sec ) 2 − (tan 2 + cot 2 ) is equal to ____.

Solution:

sin 2α + cos ec2α + 2 + cos 2 α + sec2 α + 2


− tan 2 aa
− cot 2
= 2 + 2 +1+1+1= 7
Answer: (7)

13. If tan 2 θ = 1 − e2 , then sec θ + tan 3 θ cosecθ =


(k − e2 )3/2 , where k is ____.

Solution: We have
tan 2 (θ ) = 1 − e2
1
sec (θ ) = 2 − e2 , cos (θ ) =
2 − e2
Now
sec (θ ) ⋅ cos3 θ + sin 3 (θ ) ⋅ cosec(θ )
cos3 (θ )
1 1/1
= = (2 − e2 )3/2
cos (θ ) (2 − e2 )3/2
3

K =2

Answer: (2)

14. If θ j ( j = 1, 2, 3, 4) satisfy the equation 3tan


= 3θ tan(45° + θ ), then the value of
tan θ1 + tan θ 2 + tan θ3 + tan θ 4 is ____.

Solution
1 + tan θ
3tan (3θ ) =
1 − tan θ
3(3tan θ − tan 3 θ ) 1 + tan θ
=
1 − 3tan 2 θ 1 − tan θ
(− 3tan 3 θ + 9 tan θ ) (1 − tan θ ) =
(1 + tan θ ) (1 − 3tan 2 θ )
3tan 4 θ − 6 tan 2 θ + 8 tan θ −1 = 0
−0
tan θ1 + tan θ 2 + tan θ3 + tan θ 4 = = 0
3
Answer: (0)

Solution Manual for Mathematics For JEE (Main & Advanced), Trigonometry Vector
Algebra Probability, Vol 2,
Copyright©2017 Wiley India Pvt. Ltd. All rights reserved
15. For n > 2, the value of

π 3π 5π π
sin + sin + sin +  + sin(2n − 1)
n n n n
is ____.
Solution:

π 3π π
S= sin + sin
+  + sin (2n −1)
n n n
π
  π π 3π π π
2sin  =  ⋅ 5 2sin
2
+ 2sin ⋅ sin +  + 2sin ⋅ sin(2n −1)
n n n n n n
cos 2π cos 4π cos 2π cos 2nπ cos 2(n −1)
= 1− + −  −
n n n n n
cos 2π cos 2π
=1 − −1 + =0
n n
S =0
Answer: (0)

<H1>ANSWERS
<H2>Single Correct Choice Type Questions
1. (D) 28. (B)
2. (C) 29. (A)
3. (A) 30. (C)
4. (B) 31. (A)
5. (D) 32. (A)
6. (B) 33. (A)
7. (A) 34. (B)
8. (C) 35. (C)
9. (D) 36. (A)
10. (C) 37. (B)
11. (A) 38. (D)
12. (C) 39. (A)
13. (B) 40. (A)
14. (A) 41. (B)
15. (D) 42. (B)
16. (A) 43. (A)
17. (C) 44. (C)
18. (C) 45. (D)
19. (C) 46. (B)
20. (A) 47. (A)
21. (A) 48. (B)
22. (A) 49. (A)
23. (B) 50. (B)
24. (B) 51. (B)
25. (D) 52. (C)
26. (A) 53. (A)
27. (B) 54. (D)

Solution Manual for Mathematics For JEE (Main & Advanced), Trigonometry Vector
Algebra Probability, Vol 2,
Copyright©2017 Wiley India Pvt. Ltd. All rights reserved
55. (A) 56. (D)
<H2> Multiple Correct Choice Type Questions
1. (A), (C) 9. (B), (C)
2. (A), (B), (C) 10. (A), (C), (D)
3. (A), (B), (C) 11. (A), (B), (C)
4. (A), (B), (C), (D) 12. (B), (C)
5. (A), (B), (C), (D) 13. (A), (D)
6. (A), (B), (C) 14. (A), (C), (D)
7. (A), (B), (D) 15. (A), (B), (C), (D)
8. (A), (D) 16. (A), (B), (C), (D)

<H2>Matrix-Match Type Questions


1. (A) → p, s, (B) → q, (C) → r, (D) → t 4. (A) → s, (B) → r, (C) → q, (D) → p,
2. (A) → p, (B) → p, (C) → p, (D) → s 5. (A) → t, (B) → p, (C) → r, (D) → q
3. (A) → r, (B) → s, (C) → q, (D) → p 6. (A) → p, (B) → p, (C) → q, (D) → q

<H2> Comprehension-Type Questions

1. (i) (B); (ii) (C); (iii) (C) 3. (i) (D); (ii) (B);(iii) (A)
2. (i) (A); (ii) (B); (iii) (D)

<H2>Assertion–Reasoning Type Questions


1. (A) 6. (A)
2. (A) 7. (D)
3. (A) 8. (B)
4. (D) 9. (B)
5. (A) 10. (C)
<H2> Integer answer Type Questions

1. 1
2. 5
3. 1
4. 4
5. 2
6. 5
7. 5
8. 1
9. 2
10. 2
11. 3
12. 7
13. 2
14. 0
15. 0

Solution Manual for Mathematics For JEE (Main & Advanced), Trigonometry Vector
Algebra Probability, Vol 2,
Copyright©2017 Wiley India Pvt. Ltd. All rights reserved
Solutions to Exercises
Chapter 2

SINGLE CORRECT CHOICE TYPE QUESTIONS

1. sin −1[sin(33π / 7)] is equal to

33π π −π 2π
(A) (B) (C) (D)
7 7 7 7

Solution: We have

  2π  
sin −1 sin  5π − 
  7  
  2π   2π
sin −1 sin   =
  7  7

Answer: (D)

2. tan −1[tan(2π / 3)] is equal to

2π π −π −2π
(A) (B) (C) (D)
3 3 3 3

Solution: We have

  π 
tan −1  tan  π −  
  3 

  π  π
tan −1  − tan    =−
  3  3
Answer: (C)

3. If p ≤ tan −1[(1 − x) / (1 + x)] ≤ q for all 0 ≤ x ≤ 1, then

p −p
(A)=p 0,=q =
(B) p = ,q 0
4 4

pp −p
=
(C) p = ,q =
(D) p = ,q 0
4 2 2

Solution: We have

 1− x 
p ≤ tan −1  ≤q
 1+ x 
p 
p ≤ tan −1  − x  ≤ q
4 

Solution Manual for Mathematics For JEE (Main & Advanced), Trigonometry Vector
Algebra Probability, Vol 2,
Copyright©2017 Wiley India Pvt. Ltd. All rights reserved
p 
tan p ≤  − x  ≤ tan q
4 
 p
− tan q ≤  x −  ≤ − tan p
 4
pp
− tan q ≤ x ≤ − tan p
4 4
as
0 ≤ x ≤1
so,
tan=p 0,= p 0
π
tan q =1, q =
4
Answer: (A)

4. The value of cos −1 (cos10) is

(A) 4π − 10 (B) 2π − 10 (C) 2π + 10 (D) 10

Solution: We have

cos −1[cos (10)]


cos −1[cos (4π −10)] = 4π −10
Answer: (A)

5. tan −1 (1/ 2) + tan −1 (1/ 3) is equal to

π π π π
(A) (B) (C) (D)
2 4 3 6

Solution: We have

1 1
tan −1   + tan −1  
2  3
 1/ 2 + 1/ 3  −1  5  π
= tan −1  =  tan=   tan=−1
(1)
 1 −1/ 2 ⋅ 1/ 3  5 4
Answer: (B)

6. 2 tan −1 (1/ 5) + sec −1 (5 2 / 7) + 2 tan −1 (1/ 8) =

π π −π −π
(A) (B) (C) (D)
2 4 2 4

Solution: We have

Solution Manual for Mathematics For JEE (Main & Advanced), Trigonometry Vector
Algebra Probability, Vol 2,
Copyright©2017 Wiley India Pvt. Ltd. All rights reserved
1  2 ⋅1 / 5  −1  10  −1  5 
tan −1   tan −1  =
2= 2
=
tan   tan  
 
5 1 − (1 / 5)   
24  12 
5 2  −1  7 
sec−1   = cos  
 7  5 2 
1
= tan −1  
7
1 −1  2/8   16 
2 tan − 1   tan
= =  2
tan −1  
8 1 − (1 / 8)   63 
Now,
5 1
tan − 1   + tan − 1  
 12  7
 5 /12 + 1/ 7  −1  47 
tan − 1   = tan  
 1 − 5.1/12.7   79 
Now,
 16   47 
tan − 1   + tan − 1  
 63   79 
 16 / 63 + 47 / 79  −1  4225  π
= tan − 1  =  tan  = =
 tan
−1
(1)
 1 −16.47 / 63.79   4225  4
Answer: (B)

7. tan −1 (1/ 4) + tan −1 (2 / 9) =

π π π 1  3
(A) (B) (C) (D) cos −1  
3 4 6 2 5

Solution: We have

 1/ 4 + 2 / 9  − 1  17  −1  1 
tan − 1  = =
 tan   tan  
 1 −1/ 9 ⋅ 2 / 9   34  2
Now, let
1
tan − 1   = a
2
1
tan a =
2
1  2 ⋅1 / 2 
2 tan −1   = tan −1  2
2 1 − (1 / 2) 
4
= tan −1  
3
Now,
4  3
tan −1   = cos −1  
 
3 5
Hence,
1  3
2 tan −1   = cos −1  
2 5
Solution Manual for Mathematics For JEE (Main & Advanced), Trigonometry Vector
Algebra Probability, Vol 2,
Copyright©2017 Wiley India Pvt. Ltd. All rights reserved
1 1  3
tan −1   = cos −1  
 
2 2 5
Answer: (D)

8. If x =
( 2 − 3 ) / 2, y =
12 / 4 and z = 2, then the value of
sin −1 (cot[sin −1 x + cos −1 y + sec −1 z ]) is

π π π
(A) 0 (B) (C) (D)
4 3 2

Solution: Let

2− 3
sin −1 ( x) αα
= =
, x = sin( )
2
12
sin −1 ( y ) β=
= , y = sinβ
4
−1
sec ( z ) γ =
= , z = sec(γ ) 2
1
cos(γ ) =
2
2+ 3
cot(α )= = 2+ 3
2− 3
2 3 1
cot( β=) = = = 60°
12 3 3
cot(γ =) 1, γ= 45°
Now,
cot(α + β= + γ ) cot(105° + α )
cot[90° + (a + 15°)] = − tan (a + 15°)
[tan (a ) + tan (15°)]
= −
1 − tan a ⋅ tan15°
2− 3 +2− 3 
= − 2 
 1 − (2 − 3) 
 2− 3 + 3 −2 
=
− 2   =
0
 −6+2 3 
sin −1 (0) = 0

Answer: (A)

9. tan −1 (1/ 21) + tan −1 (1/13) + tan −1 (−1/ 8) =


π π π
(A) (B) (C) (D) 0
3 4 2
Solution: We have

Solution Manual for Mathematics For JEE (Main & Advanced), Trigonometry Vector
Algebra Probability, Vol 2,
Copyright©2017 Wiley India Pvt. Ltd. All rights reserved
 1/ 21 + 1/13 
tan −1  
 1 −1/ 21⋅1/13 
−1  34  −1  1 
= tan
=   tan  
 272  8
1  1
= tan −1   + tan −1  − 
8  8
−1  1 / 8 −1 / 8 
= tan
=   0
 1 + 1 / 64 
Answer: (D)

n
 1 
10. ∑ tan −1  1 + k + k 2  is equal to
k =1

π π
(A) tan −1 (n + 1) + (B) tan −1 (n + 1) −
4 4

π  1  π
(C) tan −1 (n + 2) + (D) tan −1  +
4  n +1 4

Solution: We have

n  1 
∑ tan −1  1+ k + k 2 
k =1  
 k +1 − k  1
tan −1  = tan −1 (k + 1) − tan −1 (k )
 1+ k (k +1)  2
tan −1 (2) − tan −1 (1) + tan −1 (3) − tan −1 (2) tan −1 (n + 1) − tan −1
π
= tan −1 (n + 1) − tan −=
1
(1) tan −1 (n + 1) −
4
Answer: (B)

11. tan −1 (1/ 7) + tan −1 (1/ 3) is equal to

π 1 π
(A) + tan −1   (B)
4  3 4

π 1 2
(C) − tan −1   (D) tan −1  
4  3 3

Solution: We have

1 1
tan −1   + tan −1  
 
7  3
 1/ 7 + 1/ 3 
= tan −1  
 1 −1/ 7 ⋅ 1/ 3 

Solution Manual for Mathematics For JEE (Main & Advanced), Trigonometry Vector
Algebra Probability, Vol 2,
Copyright©2017 Wiley India Pvt. Ltd. All rights reserved
−1  10  −1  1 
= tan
=   tan  
 20  2
Now,
π
= tan −1 (1)
4
1  1 −1 / 3 
=tan −1 (1) − tan −1   =tan −1  
 3  1+1 / 3 
−1  2  −1  1 
= tan
=   tan  
4 2
Answer: (C)

12. If tan −1 x = 2 tan −1 (2 / 3), then x is equal to

7 9 11 12
(A) (B) (C) (D)
5 5 5 5

Solution: We have

−1  2 ⋅ 2 / 3 
tan −1 ( x) tan
= =  2
tan −1 (12 / 5)
1 − (2 / 3) 
12
X=
5
Answer: (D)

13. If sin −1 x + sin −1 y + sin −1 z =


3π / 2, then the value of
sin[(π / 2)( x + y + z )] is

1
(A) 0 (B) –1 (C) 1 (D)
2

Solution: We have

−1 −1 −1 π
=
sin =
( x) sin =
( y ) sin ( z)
2
π 
= =
x sin   1
2
x = y = z =1
π 
sin  (3)  = −1
2 
Answer: (B)

14. Suppose that cos −1 x + cos −1 y + cos −1 z =


3π and
α sin[π ( x + y + z )] + i cos[π ( x + y + z )], then Argα is
=

Solution Manual for Mathematics For JEE (Main & Advanced), Trigonometry Vector
Algebra Probability, Vol 2,
Copyright©2017 Wiley India Pvt. Ltd. All rights reserved
π 3π 3π −π
(A) (B) (C) (D)
2 4 2 2

Solution: We have

cos −1 ( x) = cos −1 y = cos −1 z = π


x = y = z = −1
α =sin π (−3) + i cos π (−3) =0 − i
−1 π
Arg(a ) = = tan −1 (−∞) = −
0 2
Answer: (D)

15. sin −1 (sin 5) is equal to

(A) 5 (B) 5 − 2π (C) 2π − 5 (D) π − 5

Solution: We have

sin −1 sin (5)


sin −1 [sin(5 − 2π )] =−
5 2π
Answer: (B)

If uk cosec−1 (k 2 + 1)(k 2 + 2k + 2), then ∑ k =1 uk =


n
=
16.

π π
(A) tan −1 (n + 1) − (B) tan −1 (n + 1) +
4 4

π π
(C) cot −1 (n + 1) − (D) cot −1 (n + 1) +
4 4

Solution: We have
=uk cosec−1 (k 2 + 1)(k 2 + 2k + 2)
1
= tan −1
1+ k + k2
(k + 1) − k
= tan −1
1 + k (k + 1)
= tan −1 (k + 1) − tan −1 k
Now,
n
∑ uk
k =1

= (tan 2 − tan 1) + (tan 3 − tan 2) + (tan −1 4 − tan −1 3) + (tan −1 5 − tan −1 4) +  + (tan −1 (n + 1) − tan −1 n
−1 −1 −1 −1

π
= tan −1 (n + 1) −
4

Answer: (A)

Solution Manual for Mathematics For JEE (Main & Advanced), Trigonometry Vector
Algebra Probability, Vol 2,
Copyright©2017 Wiley India Pvt. Ltd. All rights reserved
17. If

(sin −1 x)3 + (cos −1 x)3


=7
(tan −1 x + cot −1 x)3

then the value(s) of x is(are)

1 1
(A) –1 (B) 1 (C) (D) −
2 2

Solution: We have
(sin −1 x)3 + (cos −1 x)3
=7
(tan −1 x + cot −1 x)3

π 
3
⇒ [(sin −1 x + cos −1 x)3 − 3sin −1 x cos −1 x ⋅ (sin −1 x + cos −1 x)] =
7⋅ 
2
π  π π 
3 3
⇒   − 3sin −1 x cos −1 x ⋅ =7 ⋅  
2 2 2
3π π π 
3

⇒− sin −1 x  − sin −1 x  = 6 ⋅  
2  2  2
3π 2 3π 6π 3
⇒− sin −1 x + (sin −1 x)2 =
4 2 8
3π 3 6 3
⇒ (sin −1 x)2 − π 2 sin −1 x = π
2 4 8
π π2
⇒ (sin −1 x)2 − sin −1 x =
2 2
−π
⇒ sin −1 x =
2
⇒ x =−1
Answer: (A)

18. The value of x satisfying the equation

 x +1 −1  x − 1 
tan −1   + tan  =π + tan −1 (−7)
 x − 1   x 

is

−1 1
(A) −2 (B) (C) 2 (D)
2 2

Solution: We have
 x + 1 / x −1 + x −1 / x 
tan −1  
1 − ( x + 1)( x −1) / x( x −1) 

Solution Manual for Mathematics For JEE (Main & Advanced), Trigonometry Vector
Algebra Probability, Vol 2,
Copyright©2017 Wiley India Pvt. Ltd. All rights reserved
 x 2 + x + x 2 − 2 x +1 
= tan −1  
 x − x − x +1 
2 2

 2x 2 − x +1 
tan −1 
=  =π + tan −1 (−7)
 1 − x 
2x 2 − x +1
= −7
1− x
2x 2 − x +1 =7x − 7
2x 2 − 8 x +8 =
0
x2 − 4 x + 4 =
0
( x − 2)2 =
0, x = 2
Answer: (C)

19. Domain of the function

π
=f ( x) sin −1 (2 x) +
6

is

 −1 1   −1 1 
(A)  ,  (B)  , 
 4 2  2 4

 −1 1   −1 1 
(C)  ,  (D)  , 
 2 2  4 4

Solution:
π 1
sin −1 (2 x=
)+ sin −1 (2 x) + sin −1  
6 2
 3 1
sin −1  2 x ⋅ + 1− 4 x2 ⋅ 
 2 2
 1− 4 x2 
sin −1  3x + 
 2 
 
Domain:
(1 − 4 x 2 ) ≥ 0
1 1
− ≤ x≤+
2 2
1
But at x = −
2
 1  π
=
f ( x) sin −1  − ⋅ 2  +
 2  6
π π π
= − + = −
2 6 3
This is not a valid. So,

Solution Manual for Mathematics For JEE (Main & Advanced), Trigonometry Vector
Algebra Probability, Vol 2,
Copyright©2017 Wiley India Pvt. Ltd. All rights reserved
 1 1
x=− , 
 4 2
Answer: (A)

20. If

 x 2 x3 x 4   x 4 x 6 x8  π
sin −1  x − + − +  ∞  + cos −1  x 2 − + − +  ∞  =
 2 4 8   2 4 8  2

for 0 <| x |< 2 , then x is equal to

1 −1
(A) (B) 1 (C) (D) –1
2 2

Solution: We have
 x 2 x3   x 4 x6  π
sin −1  x − +  ∞  + cos −1  x 2 − +  ∞  =
 2 4   2 4  2
 x  −1  x2  π
sin −1   + cos  1 − x 2 / 2  =

 1− x / 2    2
−1  2x
2 
 2x  π
sin −1   + cos  2 
 =
 2− x   2− x  2
Where
2x 2x 2
=
2 − x 2 − x2
2 x(2 − x 2 =) 2 x 2 (2 − x)
2 x (2 − x 2 − 2 x + x 2 ) =
0
As
x≠0
So,
2 − 2 x = 0, x =1
Answer: (B)

=
21. The equation 2tan −1 x sin −1 (4 x /[1 + 4 x 2 ]) is valid if x belongs to the
interval

1   −1 
(A)  , ∞  (B)  −∞, 
2   2 

 −1 1 
(C)  ,  (D) (−1, 1)
 2 2

Solution:
We have
 2x   4x 
tan −1  2 
 = sin −1  2 

 1− x   1+ 4 x 

Solution Manual for Mathematics For JEE (Main & Advanced), Trigonometry Vector
Algebra Probability, Vol 2,
Copyright©2017 Wiley India Pvt. Ltd. All rights reserved
 2x  −1  4 x 
sin −1  
2 
= sin  2 

 1+ x   1+ 4 x 
2x 4x
=
1+ x 1+ 4 x2
2

2 x[(1 + 4 x 2 ) − 2(1 + x 2 )] =
0
either
x=0
or
2 x 2 − 2 + 1 =0
1 1 1
=x2 , x=− ,
2 2 2
Answer: (C)

22. If 0 < x < 1, then

 1 − x2  −1  1 − x 
2
tan −1   + cos  =
2 
 2x  1+ x 

−π π
(A) 0 (B) (C) (D) π
2 2

Solution
 2x  −1  1 − x 
2
cos −1  
2 
+ cos 
 2 

 1+ x   1+ x 
 1− x2  −1  1 − x  π
2
sin −1  
2 
+ cos  1 + x 2  =

 1+ x    2
Answer: (C)

23. If

1+ x  −1  1 − x 
2
= tan −1 
a 2=  and β sin  2 

1− x  1+ x 

where 0 < x < 1, then α + β is equal to

π
(A) 2π (B) π (C) 0 (D)
2

Solution: We have
 1+ x  −1  2(1 + x )(1 − x ) 
= tan −1 
a 2=  tan  2
 1− x   (1 − x) − (1 + x) 
2

 1− x2 
= π − tan −1  
 2x 

Solution Manual for Mathematics For JEE (Main & Advanced), Trigonometry Vector
Algebra Probability, Vol 2,
Copyright©2017 Wiley India Pvt. Ltd. All rights reserved
 1− x2 
= β= sin −1  
 1+ x
2

 1− x2   1− x2 
−1 
= tan
=  tan −1  
 2 2
 (1 + x ) − (1 − x )   2x 
2 2

Now,
α + β =π + 0
Answer: (B)

n  k − k −1
24. ∑ sin −1  =
k =1  k (k + 1) 

π π
(A) tan −1 n + (B) tan −1 n + 1 +
4 4

π
(C) tan −1 n − (D) tan −1 n
4

Solution: Since
k − k −1 k − k +1
sin −1 = tan −1
k (k + 1) 1 + k (k − 1)
= tan −1 k − tan −1 k − 1
Now,
n
k − k −1
∑ sin −1 k (k + 1)
k =1
n
= ∑ (tan −1 k − tan −1 k − 1)
k =1

= (tan −1 1 − tan −1 0) + (tan −1 2 − tan −1 1) + (tan −1 3 − tan −1 2) +  + (tan −1 n − tan −1 n − 1)


= tan −1 n
Answer: (D)

MULTIPLE CORRECT CHOICE TYPE QUESTIONS

1. Which of the following are integer pair solutions of the equation


tan −1 x + tan −1 (1/ y ) =
tan −1 (3) ?

(A) (1, 2) (B) (4, −13) (C) (5, −8) (D) (2, 7)

Solution

1
(A) tan −1 (1) + tan −1  
2
 1 + 1/ 2 
tan −1  −1
 = tan (3) True
 1 − 1 ⋅ 1/ 2 

Solution Manual for Mathematics For JEE (Main & Advanced), Trigonometry Vector
Algebra Probability, Vol 2,
Copyright©2017 Wiley India Pvt. Ltd. All rights reserved
 1 
(B) tan −1 (4) + tan −1  
 −13 
 4 −1 / 3 
= tan −1  
 1+ 4 / 3 
−1  51  −1
= tan
=   tan (3) True
 17 
 1 
(C) tan −1 (5) + tan −1  
 −8 
 5 −1 / 8 
= tan −1  
 1+ 5 / 8 
 39 
tan −1   = tan −1 (3) True
 13 
1
(D) tan −1 (2) + tan −1  
7
 2 +1 / 7  −1  15 
= tan −1  = =
 tan
−1
  tan (3) True
1 − 2(1 / 7)  5
Answer: (A), (B), (C), (D)

=
2. Let y (π / 2) + cos −1 (cos x). Then the values of y such that
=y | tan x | (0 ≤ x ≤ 2π ) are

π π 5π 3π
(A) x + (B) x + (C) − x + (D) − x +
4 2 2 2

Solution: We have

π
y= + cos −1[cos( x)]
2
π
= +x
2
and
π
y− = cos −1[cos( x)]
2
 π 
= cos  y − = cos ( x)
 2 
π
=y− =π − x
2
π 3π
y=π + −x= −x
2 2
Answer: (B), (D)

3. If x and y are real numbers such that tan −1 y = 4 tan −1 x, then

Solution Manual for Mathematics For JEE (Main & Advanced), Trigonometry Vector
Algebra Probability, Vol 2,
Copyright©2017 Wiley India Pvt. Ltd. All rights reserved
4 x − 4 x3 4 x + x3
(A) y = (B) y =
1 − 6 x2 + x4 1 − 6 x2 + x4

4 x − x3
(C) x ≠ tan22 12 ° (D) y =
1 + 6 x2 − x4

Solution:

 2x 
tan −1 y = 2 tan −1  
2 
 1− x 
 2(2x)(1 − x 2 ) 
= tan −1  2 

 (1 − x ) − (2 x) 
2 2

 4 x − 4x 3 
= tan −1  4 

 1− 6 x + x 
2

Also, when,
1a
x = tan 22
2
4 x= 90°
and = =
y tan (90 °) undefind
Hence
1a
x ≠ tan 22
2
Answer: (A), (C)

4. If sin −1 x + sin −1 y + sin −1 z =


3π / 2, then

(A) x + y + z =xyz

(B) x + y + z =3xyz

(C) 1 + w x + y + z =
2, where w ≠ 1 is a cube root of unity

0, where w is a non-real root of x3 − 1 =0


(D) w x + w2 y + w3 z =

Solution: Here,
π
sin − 1 ( x=) = sin − 1 y= sin − 1 ( z )
2
x= y= z= 1
(B) So, x + y + z = 3xyz
(C) As, w is cube root of unity,
So, we know that
w3 =1
Now,
w x + y +=
z
w1 + 1 +=
1
w=
3
1
1 + w x + y + z =1 + 1 = 2

Solution Manual for Mathematics For JEE (Main & Advanced), Trigonometry Vector
Algebra Probability, Vol 2,
Copyright©2017 Wiley India Pvt. Ltd. All rights reserved
(D) w1 + w2 + w3 =
0
As, sum of cube root of unity = 0
Answer: (B), (C), (D)

5. If f ( x) = sin −1 x and g ( x) = cos −1 x, then

1
(A) f ( x) > g ( x) if < x ≤1
2

1
(B) f ( x) ≤ g ( x) if −1 ≤ x ≤
2

1
(C) g ( x) ≥ f ( x) if ≤ x ≤1
2

1
(D) g ( x) ≤ f ( x) if −1 ≤ x ≤
2

Solution:
x = sin[ f ( x)] and y = cos[ g ( x)]
From graphical view, we know that sine is above Cosine function for
π π
≤θ ≤
4 2
1
or, ≤ x≤1
2
Hence, f ( x) > g ( x)
(B) and for
π π
− ≤θ ≤
2 4
That is,
1 1
− ≤ x≤
1 2
Cosine function is above sine function.
Hence g ( x) ≥ f ( x).
Answer: (A), (B)

6. Which of the following statements are true?


(A) x = 1 is a solution of the equation cos −1 (1 − x) + sin −1 x =
π
(B) If cos −1 x + cos −1 y =
π / 3, then x 2 − xy + y 2 =
3/ 4
−1
=
(C) tan (4 / 3) tan −1 (1/ 7) + (π / 4)
x a−x
(D) For 0 < x < a, 2 tan −1 =cos −1  
a a+ x

Solution:
(A) For x =1

Solution Manual for Mathematics For JEE (Main & Advanced), Trigonometry Vector
Algebra Probability, Vol 2,
Copyright©2017 Wiley India Pvt. Ltd. All rights reserved
cos −1 (1 −1) + sin −1 (1)
cos −1 (0) + sin −1 (1)
π π
+ π
=
2 2
(B) (
cos −1 xy − 1 − x 2 1 − y 2 =
3
) π

1
xy − 1 − x 2 1 − y 2 =
2
2
 1  2 
2
 xy −  =  (1 − x )(1 − y ) 
2
 2
1
x 2 y 2 − xy + =(1 − x 2 )(1 − y 2 ) =1 − x 2 − y 2 + x 2 y 2
4
1 3
x 2 + y 2 − xy =1 − =
4 4
1
(C) tan −1   + tan −1 (1)
7
 1/ 7 + 1  −1  4 
tan −1   = tan  
 1 −1/ 7  3
 2 xa 
(D) tan −1  
 a−x 
a−x
cos −1  
a+ x
Answer: (A), (B), (C), (D)

MATRIX-MATCH TYPE QUESTIONS


In each of the following questions, statements are given in two columns, which have to be
matched. The statements in column I are labeled as (A), (B), (C) and (D), while those in column II
are labeled as (p), (q), (r), (s) and (t). Any given statement in column I can have correct matching
with one or more statements in column II. The appropriate bubbles corresponding to the answers
to these questions have to be darkened as illustrated in the following example.

Example: If the correct matches are (A) → (p), (s), (B) → (q), (s), (t), (C) → (r), (D) → (r), (t) ,
that is if the matches are (A) → (p) and (s); (B) → (q), (s) and (t); (C) → (r); and (D) → (r), (t),
then the correct darkening of bubbles will look as follows:

1. Match the items in Column I with those in Column II.

Column I Column II

Solution Manual for Mathematics For JEE (Main & Advanced), Trigonometry Vector
Algebra Probability, Vol 2,
Copyright©2017 Wiley India Pvt. Ltd. All rights reserved
(A) sin −1 (1/ 2) + cos −1 (−1/ 2) is equal to π
(p)
6

(B) tan −1 3 + cot −1 (− 3) is equal to 5π


(q)
6

(C) sin −1 (−1/ 2) + cos −1 (1/ 2) is equal to π


(r)
3

(D) 7π
(s)
−1 −1
cos [cos(7π / 6)] + sin [sin(2π / 3)] = 6


(t)
3

Solution:
1 π  1
(A) sin −1   + − sin −1  − 
2 2  2
π 1 2 2 
 1  1 1
+ sin −1  ⋅ 1 −  −  −  −  ⋅ 1−   
2 2  2  2 2 
 
π  3  3 
+ sin −1  −  −  
2  4  4  

π  3  π π 5π
+ sin −1   = + =
2  2  2 3 6
Answer: (q)

(B) tan −1 ( 3) + cot −1 (− 3)


π  π  7π
+ π −  =
3  6 6
Answer: (s)

 1 1
(C) sin −1  −  + cos −1  
 2  2
π 1  1
− sin −1   + sin −1  − 
2 2  2
π  1   3  3  1  
+ sin −1  −    − ⋅  
2  2   2  2  2  
π π π
− =
2 3 6

Solution Manual for Mathematics For JEE (Main & Advanced), Trigonometry Vector
Algebra Probability, Vol 2,
Copyright©2017 Wiley India Pvt. Ltd. All rights reserved
Answer: (p)

  π   2π 
(D) cos −1 cos  π +   + sin −1  sin 
  6   3 
π 2π 7π
= + =
2 3 6
Answer: (s)

2. Match the items in Column I with those in Column II.

Column I Column II

π (p) 6
(A) If sin −1 x − cos −1 x = , then x =
6

(B) If tan −1 x + tan −1 3 =


tan −1 8, then x = 5
(q)
3

(C) If tan(sec−1 x) = sin(tan −1 2) and x > 0, then x is equal to (r)


1
5

(D) If tan −1 (4 / x) + tan −1 (9 / x) =


π / 2, then x = 3
(s)
2

1
(t)
3

Solution:
π  π
(A) sin −1 ( x) −  − sin −1 ( x)  =
 2  6
π π 7π
2sin −1 ( x) = + =
6 2 6

(
−1
sin 2 x ⋅ 1 − x 2 = ) 7π
6
1
2x ⋅ 1 − x = 2

2
1
x 2 (1 − x 2 ) =
16
1
− x4 + x2 =
16
Let y = x 2

Solution Manual for Mathematics For JEE (Main & Advanced), Trigonometry Vector
Algebra Probability, Vol 2,
Copyright©2017 Wiley India Pvt. Ltd. All rights reserved
1
y2 − y + = 0
16
3
y =+ 1 ±
2
3
x=
2
Answer: (s)

 8−3 
(B) tan −1 ( x) = tan −1  
 1 − 24 
x+3
or, =8
1 − 3x
= x + 3 = 8 − 24 x
1
x=
5
Answer: (r)

  1 
(C) tan (sec−1 x) = tan cos −1   
  x 
  1   −1  2  
= tan
=  tan −1    sin sin  
  5 
  x − 1   
2

1 2
=
x −1
2 5
 4  5 
or, x 2 −=
1  =  
 5  4
5
x=
3
Answer: (q)

 4+9/ x  π
(D) tan −1  2  =
 x − 36  2
x 2 − 36 =
0
x=6
Answer: (p)

3. Match the items in Column I with those in Column II.

Column I Column II

π  1  3− 5
sin  − sin −1    = (p)
(A) 
3  2  2

Solution Manual for Mathematics For JEE (Main & Advanced), Trigonometry Vector
Algebra Probability, Vol 2,
Copyright©2017 Wiley India Pvt. Ltd. All rights reserved
  3 π 24
(B) cos cos −1   +  =
(q)
7
  2  6 

1 5 1
(C) tan  cos −1 = (r)
2 2  2

 4 24
(D) − tan  2sin −1  = (s)
 5  7

1
(t) −
2

Solution:
1
(A) sin −1   = α
2
1 3
= sin αα =, cos
2 2
π  π  π 
ααα
sin  −= sin   ⋅ cos − cos   ⋅ sin .
3  3 3
3 3 1 1 1
= ⋅ − ⋅ =
2 2 2 2 2
Answer: (r)

π π  π  1
(B) cos  = +  cos −1=
 
 6 6  3 2
Answer: (r)

−1  5  5
(C) Let cos=   αα
=
, cos
 2  2
 a  3− 5
tan   =
2 2
Answer: (p)

  4  24
(D) − tan  2sin −1    = −
  5  7
 2 tan a  2 * 4 * 3 24
− tan (2a ) = −   == −
 1 − tan a 
2
9 − 16 7
Answer: (q)

4. Match the items in Column I with those in Column II.

Solution Manual for Mathematics For JEE (Main & Advanced), Trigonometry Vector
Algebra Probability, Vol 2,
Copyright©2017 Wiley India Pvt. Ltd. All rights reserved
Column I Column II

(A) If cot −1 x + cot −1 2 x =


3π / 4, then x = −3 ± 17
(p)
4

(B) If tan −1 x = cot −1 x, then x = (q) 0

(C) If cos −1 x − sin −1 x =


cos −1 ( 3x), then x = (r) 1

1
(D) If tan −1 x + tan −1 (1
= − x) 2 tan −1 x − x 2 , then x (s)
2
=

1
(t) −
2

Solution:
 2 x 2 −1  3π
(A) cot −1   =
 3x  4
2 x 2 −1 =− 3x
2 x 2 + 3x −1 =0
−3 ± 17
x=
4
Answer: (p)

(B) tan −1 ( x) cot


= = −1
( x), x 1
Answer: (r)

π
(C) 2 cos −1 x − cos −1 ( 3x)
=
2
π
= cos ( 3x) + cos −1 ( 1 − x 2 )
−1
2
π
cos −1  3x ⋅ 1 − x 2 − (1 − 3x 2 ) x 2  =
  2
(3x)(1 − x 2 ) = x 2 (1 − 3x 2 )
3 − 3x 2 =−
x 3 x3
3 x3 − 3 x 2 − x + 3 =0
1 1
= x 0, , −
2 2
Answer: (q), (s), (t)

 2 
 x +1− x  −1 2 x − x  −b ± b − 4ac
2
(D) tan −1   = tan 
1 − [( x)(1 − x)]  1 − ( x − x 2 ) 
 
2a

Solution Manual for Mathematics For JEE (Main & Advanced), Trigonometry Vector
Algebra Probability, Vol 2,
Copyright©2017 Wiley India Pvt. Ltd. All rights reserved
1
= x − x2
2
1 1
x −=
x2 , =x
4 2
Answer: (s)

5. Match the items in Column I with those in Column II.

Column I Column II

(A) If f1 ( x) = cos −1 x + sin −1 x + cot −1 x, then  3π 5π 


(p)  , 
f1 ( x) belongs to  4 4 

(B) If f 2 ( x) = tan −1 x + cot −1 x + cos ec−1 x, then (q)


(0, π ) − (π / 2)
f 2 ( x) belongs to

(C) Range of the function  π 3π 


−1 −1 −1 (r)  , 
f3 ( x) = cot x + tan x + cos x lies in the 2 2 
interval

(D) The range of  π


−1 −1 −1
(s)  0, 
f 4 ( x) = cot x + tan x + sin x lies in  2

(t) (0, π)

Solution:
(A) f1 ( x) =sin −1 ( x) + cos −1 ( x) + cot −1 ( x)
π
+ cot −1 ( x)
2
Now,
π 3π
≤ cot −1 ( x) ≤
4 4
π π π 3π
+ ≤ f1 ( x) ≤ +
2 4 2 4
3π 5π
≤ f1 ( x) ≤
4 4
Answer: (p)

(B) f 2 ( x) = tan −1 ( x) + cot −1 ( x) + cos ec −1 ( x)

Solution Manual for Mathematics For JEE (Main & Advanced), Trigonometry Vector
Algebra Probability, Vol 2,
Copyright©2017 Wiley India Pvt. Ltd. All rights reserved
π
+ cos ec−1 ( x)
2
Now,
 π   π
cos ec−1 ( x) =
 − 2 , 0  ∪  0, 2 
   
So,
π 
2 ( x ) (0, π ) −  
f=
2
Answer: (q)
π −1
(C) f3 ( x=) + cos ( x)
2
Now,
cos −1 ( x) = (0, π )
 π 3π 
f3 ( x) =  , 
2 2 
Answer: (r)
π
(D) f 4 ( x=) + sin −1 ( x)
2
Now,
 π π
sin −1 ( x) =  − , 
 2 2
f 4 ( x) = (0, π )

Answer: (t)

COMPREHENSION TYPE QUESTIONS

1. Passage: If x > 0 and y > 0.


 x− y 
tan −1 x − tan −1 y =
tan −1  
 1 + xy 
Answer the following questions.
(i) tan −1 2 − tan −1 x =
π / 4 implies that the value of x is

1 1
(A) 2 (B) (C) 2 (D)
2 3

(ii) If x and y are positive, then

x  y−x
tan −1   + tan −1  
 y  y+ x

is equal to

π −π π −π
(A) (B) (C) (D)
4 4 3 3

(iii) If

Solution Manual for Mathematics For JEE (Main & Advanced), Trigonometry Vector
Algebra Probability, Vol 2,
Copyright©2017 Wiley India Pvt. Ltd. All rights reserved
 1  −1  1  1
3tan −1   − tan   = tan −1  
2+ 3  
x  3

then x is equal to

1
(A) 2 (B) (C) 1 (D) −1
2

Solution:
(i)
 2− x  π
tan −1  =
 1+ 2 x  4
2− x
=1
1+ 2 x
1 + 2 x =2 − x
1
3x =
2 − 1, x=
3
Answer: (D)

(ii)
 x / y+ y−x / x+ y 
tan −1  
1 − x ( y − x ) / y ( x + y ) 
 x( x + y ) + y ( y − x) 
tan −1  
 y ( x + y ) − x( y − x) 
 x2 + y 2  π
tan −1  2 =
2 
x +y  4
Answer: (A)

(iii) For x = 2,
1 1
tan −1   + tan −1  
 3 2
5
tan −1   = tan −1 (1)
5
Answer: (A)
Now,
 1 
3 tan −1  = −1
 3tan (2 − 3)
 2+ 3 

 (2 − 3)[3 − (2 − 3)] 
2

= tan −1  

 1 − 3(2 − 3) 2


 (2 − 3) + ( 3 −1) 
= tan −1  
 4(3 3 − 5) 

−1 3 3 − 5  −1
= tan
=   tan (1)
 3 3 −5 

2. Passage: If x > 0 and y > 0.

Solution Manual for Mathematics For JEE (Main & Advanced), Trigonometry Vector
Algebra Probability, Vol 2,
Copyright©2017 Wiley India Pvt. Ltd. All rights reserved
 x− y 
tan −1 x − tan −1 y =
tan −1  
 1 + xy 

Answer the following questions.

(i) Sum to n terms of the series

x x x
tan −1 + tan −1 + tan −1 +
1 + 2x 2
1 + 6x 2
1 + 12 x 2

is

(A) tan −1 (n + 1) − tan −1 x


(B) tan −1 nx − tan −1 x
 1 
(C) tan −1  −1
 − tan x
 n +1
(D) tan −1[n(n + 1) x] − tan −1 x
(ii) Sum of the first 2010 terms of the series

x x x
tan −1 x + tan −1 + tan −1 + tan −1 +
1 + 1⋅ 2x 2
1 + 2 ⋅ 3x 2
1 + 3 ⋅ 4 x2

is

(A) tan −1 (2011x) − tan −1 x


(B) tan −1 (2010 x) − tan −1 x
(C) tan −1 (2010 x)
(D) tan −1 (2009 x)
(iii) If the sum of the first n terms of the series

4 6 8
tan −1 + tan −1 + tan −1 +
13 73 241

is tan −1[(n + 1)(n + 2)] − tan −1 x then x is equal to

(A) 1 (B) −2 (C) −1 (D) 2

Solution:
(i)
 x   2x − x 
tan −1  2 
 = tan −1  
 1+ 2 x  1 + (2 x)( x) 
= tan −1 (2 x) − tan −1 ( x)
For nth term
 x 
tan −1  = −1 −1
 tan [(n + 1) x] − tan (nx)
2 
 1 + ( n th) x 

Solution Manual for Mathematics For JEE (Main & Advanced), Trigonometry Vector
Algebra Probability, Vol 2,
Copyright©2017 Wiley India Pvt. Ltd. All rights reserved
tan −1 (n + 1) x − tan −1 ( x)
Answer: (A)

(ii)
tan −1 ( x) + tan −1 (2 x) − tan −1 ( x) +  + tan −1 (2010) x − tan −1
tan −1 (2010)x

Answer: (C)
(iii)
4 6 8
tan −1 + tan −1 + tan −1 
13 73 241
= (tan −1 6 − tan −1 2) + (tan −1 12 − tan −1 6) + (tan −1 20 − tan −1 12) +  + to n terms
= tan −1 (n + 2)(n + 1) − tan −1 2
⇒x=2
Answer: (D)

ASSERTION–REASONING TYPE QUESTIONS


In the following set of questions, a Statement I is given and a corresponding Statement II is given
just below it. Mark the correct answer as:

(A) Both Statements I and II are true and Statement II is a correct explanation Statement I.
(B) Both Statements I and II are true but Statement II is not a correct explanation for Statement I.
(C) Statement I is true and Statement II is false.
(D) Statement I is false and Statement II is true.
1. Statement I: If a = tan −1 (1 / 7) and β = tan −1 (1 / 3), then
cos 2α = sin 4β .

1 − tan 2θ 2 tanθ
Statement II: cos 2θ = and sin 2θ =
1 + tan θ2
1 + tan 2θ

Solution:
1 1
=tan a =, tan ( β )
7 3
1 − tan a 1 − (1/ 7) 2
2
cos 2a =
=
1 + tan 2 a 1 + (1/ 7) 2
48 24
==
50 25
sin 4β 2sin 2β ⋅ cos 2β
=
 2sin β   1 − tan 2 β 
= 2   ⋅  
 1 + tan β   1 + tan β 
2 2

(1 / 3) 1 − (1 / 3) 2 
= 4⋅  
1 + (1 / 3) 2 1 + (1 / 3) 2 
12 6 8 4 24
= * =
10 5 10 5 25

Solution Manual for Mathematics For JEE (Main & Advanced), Trigonometry Vector
Algebra Probability, Vol 2,
Copyright©2017 Wiley India Pvt. Ltd. All rights reserved
Answer: (A)

=
2. Statement I: If β cot −1 cos a − tan −1 cos a , then
sin β = tan 2 (a / 2)

Statement II: cot −1 x = tan −1 (1/ x) for any x > 0

Solution:
 1 
= β tan −1  −1
 − tan ( cos a )
 cos a 
 1 − cos a 
β = tan −1  
 (1 + 1) cos a 
 2sin 2 a / 2   sin 2 a / 2 
= tan −1  =  ⇒ tan β  
 2 ⋅ cos a   cos a 
 sin 2 α / 2 
sin( β ) =  
 cos αα + sin / 2 
4

 sin 2 a / 2  2a
=  =  tan
 cos a / 2 
2
2
Answer: (A)

3. Statement I: If tan −1 x + tan −1 y + tan −1 z =


π / 2 and x + y + z + = 3,
then x = y = z

 x+ y 
Statement II: tan −1 x + tan −1 y =
tan −1   , if xy < 1
 1 − xy 

Solution:
 x + y + z − xyz  π
tan −1  =
1 − ( xy + yz + zx)  2
xy + yz + zx =
1
and
x + y + z =3
( x + y + z= =
) 2 ( 3) 2
3
x 2 + y 2 + z 2 + 2(1) =
3
x 2 + y 2 + z 2 =1, x = y =z
Answer: (A)

 2x 
4. Statement I: tan −1 x + sin −1  =π for x > 1
 1 + x2 

Statement II: For −1 ≤ x ≤ 1, − π / 2 ≤ sin −1 x ≤ π / 2

Solution Manual for Mathematics For JEE (Main & Advanced), Trigonometry Vector
Algebra Probability, Vol 2,
Copyright©2017 Wiley India Pvt. Ltd. All rights reserved
Solution:
 2 tan a 
tan −1 ( x) + sin −1  
 1 + tan a 
2

tan −1 (tan a ) + sin −1 (sin 2a )


ααα
+2 =
3
3sin −1 ( x) = π
π π 
sin −1 ( x)
= =, sin   x
3 3
3
x=
2
Answer: (A)

 3 8  77 
5. Statement I: sin −1   + sin −1   = sin −1  
5  17   85 

−1
Statement II: For 0 ≤ x ≤ 1, sin= x cos −1 1 − x 2 and

−1
=
cos x sin −1 1 − x 2

Solution:
 3 8
sin −1   + sin −1  
5  17 
3 8
tan −1   + tan −1  
4  15 
 3 4 + 8 15 
tan −1  
 1 − 3 / 4 * 8 2 / 15 5 
 45 + 32 
tan −1  
12(5 − 2) 
 77  −1  77 
tan −1   = tan  
 12 * 3   36 
 
77 −1  77  −1  77 
tan −1  =  sin=   sin  
 (77) + (36) 
2 2   
85  85 

Answer: (A)

INTEGER ANSWER TYPE QUESTIONS


The answer to each of the questions in this section is a non-negative integer. The appropriate
bubbles below the respective question numbers have to be darkened. For example, as shown in the
figure, if the correct answer to the question number Y is 246, then the bubbles under Y labeled as
2, 4, 6 are to be darkened.

Solution Manual for Mathematics For JEE (Main & Advanced), Trigonometry Vector
Algebra Probability, Vol 2,
Copyright©2017 Wiley India Pvt. Ltd. All rights reserved
X Y Z W
ⓞ ⓞ ⓞ ⓞ
① ① ① ①
② ② ② ②
③ ③ ③ ③
④ ④ ④ ④
⑤ ⑤ ⑤ ⑤
⑥ ⑥ ⑥ ⑥
⑦ ⑦ ⑦ ⑦
⑧ ⑧ ⑧ ⑧
⑨ ⑨ ⑨ ⑨

1. 4tan −1 (1/ 5) − tan −1 (1/ 239) =


π / k , where k is equal to ____.

Solution:
  2 *1 5   −1  2 * 5 *12  −b ± b − 4ac
2
2  tan −1    = tan  
1 − (1 / 5)    144 − 25 
2
 2a
 120 
= tan −1  
 119 
 120  −1  1 
tan −1   − tan  
 119   239 
−1 π
= tan = (1)
4
k =4
Answer: (4)

2. sin −1 (1/ 2) + cos −1 (1/ 2) =


kπ /12, where k is equal to ____.

Solution:
1  1 
sin −1   = tan −1  
 
2  3
π π (2 + 3)π 5π
=+ =
6 4 12 12
Answer: (5)

3. If 2 tan −1 (1/ 2) + cos −1 (4 / 5) =


π / k , then k is equal to ____.

Solution:
 2⋅2  −1  4 
tan −1   + cos  
 4 −1  5
4 3
tan −1   + tan −1  
3 4
 4 / 3 + 3/ 4  −1  25  π
tan −1  = =
 tan  
 1 − 4 / 3 ⋅ 3/ 4   0  2

Solution Manual for Mathematics For JEE (Main & Advanced), Trigonometry Vector
Algebra Probability, Vol 2,
Copyright©2017 Wiley India Pvt. Ltd. All rights reserved
k =2
Answer: (2)

4. If 2 tan −1 (1/ 5) + tan −1 (1/ 4) =


tan −1 k , then the integral part of k is
____.

Solution:

 2.5  −1  5 
tan −1   = tan  
 25 −1   12 
5 1
tan −1   + tan −1  
 12  4
 5 +3  −1  8 
tan −1   = tan  
 1 − 5 /12 ⋅ 1/ 4   43 
k =0
Here integral part of k = 0
Answer: (0)

5. cot −1 (cot 4)= k − π , where k is equal to ____.

Solution:
cot −1[cot (4)]= 4 − π
k =4
Answer: (4)

6. If sin −1 (4 / 5) + sin −1 (3 / 5) =
2 tan −1 k , then k is equal to ____.

Solution:
4  3
sin −1   + sin −1  
5 5
4 3
tan −1   + tan −1  
3 4
 4 / 3 + 3/ 4  π
tan −1  =  tan =
−1
(∞ )
 1 −1  2
 2k 
= tan −1  
2 
 1− k 
1− k2 =
0, k=
±1
Answer: ( ±1 )

15   −1  1   −1 
7. sin  2 tan    + cos[tan (2 2)] =
____.
2   3  

Solution:

Solution Manual for Mathematics For JEE (Main & Advanced), Trigonometry Vector
Algebra Probability, Vol 2,
Copyright©2017 Wiley India Pvt. Ltd. All rights reserved
15   1 
 sin  2 + tan −1   + cos(tan −1 2 2)
2  3 
15   −1 (2 / 3)   1  
sin  tan  + cos  cos −1  
2   1 − (1 / 9)   3  
15  −1 3 1
 sin tan + cos cos −1 
2 4 3
15  3 1 
 + = 7
2  5 3
Answer: (7)

1  4  2 
8. + tan cos −1   + tan −1    =____.
6   
5  3 

Solution:
1  3  2 
+ tan  tan −1   + tan −1   
6   
4  3 
1   3 / 4 + 2 / 3 
+ tan  tan −1  
6   1− 3 / 4 * 2 / 3 
1   17  
+ tan  tan −1   
6   6 
1 17
+ = 3
6 6
Answer: (3)

9. If x satisfies the equation tan −1 ( x + 3) − tan −1 ( x − 3) =


sin −1 (3 / 5), then
|x| is ____.

Solution:
 x + 3− x + 3 
tan −1  
1 + ( x + 3)( x − 3) 
 6  −1  3 
tan −1  2  = tan  
 x −8  4
6 3
=
x −8 4
2

=
8 x2 − 8
x= ± 4
x =4
Answer: (4)

10. If sin −1 x + sin −1 y + sin −1 z =


π , then
x 4 + y 4 + z 4 + 4 x 2 y 2=
z 2 k ( x 2 y 2 + y 2 z 2 + z 2 x 2 ), where k is equal to
____.
Solution Manual for Mathematics For JEE (Main & Advanced), Trigonometry Vector
Algebra Probability, Vol 2,
Copyright©2017 Wiley India Pvt. Ltd. All rights reserved
Solution:
sin −1 x + sin −1 y + sin −1 z =
π
Let
sin −1 ( x) αα
= =
, sin x
sin −1 ( y ) β=
= , sin( β ) y
sin −1 ( z ) γ=
= , sin(γ ) z
α + β +γ = π
[sin 4 α + sin 4 ( β ) + sin 4 (γ )] + 4sin 2 (α ) ⋅ sin 2 ( β ) ⋅ sin 2 (γ )
[(sin 2 α ) 2 + (sin 2 β ) 2 ]2 − 2sin 2 α ⋅ sin 2 β − sin 4 (αα
) + 4[sin 2 ( ) ⋅ sin 2 ( β ) ⋅ sin 2 (γ )]
k =2

Answer: (2)

<H1>ANSWERS

<H2>Single Correct Choice Type Questions

1. (D) 13. (B)


2. (C) 14. (D)
3. (A) 15. (B)
4. (A) 16. (A)
5. (B) 17. (A)
6. (B) 18. (C)
7. (D) 19. (A)
8. (A) 20. (B)
9. (D) 21. (D)
10. (B) 22. (C)
11. (C) 23. (B)
12. (D) 24. (D)

<H2>Multiple Correct Choice Type Questions

1. (A), (B), (C), (D) 4. (B), (C), (D)


2. (B), (C) 5. (A), (B)
3. (A), (C) 6. (A), (B), (C), (D)

<H2>Matrix-Match Type Questions

1. (A) → (q), (B) → (s), 4. (A) → (p), (B) → (r),


(C) → (p), (D) → (s) (C) → (q), (s), (t), (D)
2. (A) → (s), (B) → (r), → (s)
(C) → (q), (D) → (p) 5. (A) → (p), (B) → (q),
3. (A) → (r), (B) → (r), (t), (C) → (r), (D) →
(C) → (p), (D) → (q) (t)

Solution Manual for Mathematics For JEE (Main & Advanced), Trigonometry Vector
Algebra Probability, Vol 2,
Copyright©2017 Wiley India Pvt. Ltd. All rights reserved
<H2> Comprehension Type Questions

1. (i) (D); (ii) (A); (iii) 2. (i) (A); (ii) (C); (iii)
(A) (D)

<H2>Assertion–Reasoning Type Questions

1. (A) 4. (A)
2. (A) 5. (A)
3. (A)

<H2>Integer Answer Type Questions

1. 4 6. 1
2. 5 7. 7
3. 2 8. 3
4. 0 9. 4
5. 4 10. 2

Solution Manual for Mathematics For JEE (Main & Advanced), Trigonometry Vector
Algebra Probability, Vol 2,
Copyright©2017 Wiley India Pvt. Ltd. All rights reserved
Solutions to Exercises
Chapter 3

SINGLE CORRECT CHOICE TYPE QUESTIONS

1. The number of roots of the equation

cot 2 x cot x
+ +2=0
cot x cot 2 x

in the interval (0, 2π) is

(A) 1 (B) 2 (C) 3 (D) 4

Solution:
cos (2 x) sin ( x) cos ( x) sin (2 x)
⋅ + ⋅ +2=0
sin (2 x) cos ( x) sin ( x) cos (2 x)
cos (2 x) 2 cos 2 ( x)
+ +2=0
2 cos 2 ( x) cos(2 x)
cos 2 (2 x) + 4 cos 4 ( x) + 4 cos 2 ( x) ⋅ cos(2 x) =
0
[2 cos 2 ( x) −1]2 + 4 cos 4 ( x) + 4 cos 2 ( x)[2 cos 2 ( x) −1] =
0
4 cos 4 ( x) − 4 cos 2 ( x) + 1 + 4 cos 4 ( x) + 8cos 4 ( x) − 4 cos 2 ( x)
16 cos 4 ( x) − 8cos 2 ( x) + 1 =0
[4 cos 2 ( x) −1]2 =
0
1
4 cos 2 ( x) = 1, cos( x) = ±
2
π 2π 4π 5π
x= , , ,
3 3 3 3
Answer: (D)
2. General solution of the equation sin 6 x + 2 =2cos 4 x is

nπ π
(A) , n ∈ (B) nπ + , n ∈
6 3

nπ π
(C) , n ∈ (D) 2nπ + , n ∈
2 3

Solution:
2 + 3sin(2 x) − 4sin 3 (2 x) =
2[1 − 2sin 2 (2 x)]
4sin 3 (2 x) − 4sin 2 (2 x) − 3sin(2 x) =
0
sin(2 x)[4sin 2 (2 x) − 4sin(2 x) − 3] =
0
Either,
sin=
(2 x) 0,=2 x nπ
= , x nπ / 2
or,
4sin 2 (2 x) − 4sin(2 x) − 3 =0
[2sin(2 x) − 3][2sin(2 x) + 1] = 0

Solution Manual for Mathematics For JEE (Main & Advanced), Trigonometry Vector
Algebra Probability, Vol 2,
Copyright©2017 Wiley India Pvt. Ltd. All rights reserved
1  7π   11π 
sin(2 x) = − = sin   or sin  
2  6   6 

2 x= nπ ± (−1) n or
6
so,

=x , n∈ z
2
Answer: (C)
3. The number of solutions of the equation tan x ⋅ tan 3x ⋅ tan 4 x = tan x − tan 2 3x + tan 4 x
2 2 2

in the interval (0, π) is

(A) 0 (B) 2 (C) 3 (D) 4


Solution:
tan(4 x) (1 − tan 2 x ⋅ tan 2 3x) = tan 2 3x − tan 2 x
tan 2 3x − tan 2 x
tan(4 x) =
(1 − tan 2 x ⋅ tan 2 3x)
=
tan(4 x) tan(4 x) ⋅ tan(2 x)
tan(4 x)[tan(2 x) −1] = 0
Either
tan 4 x = 0 ,

= π, x
4 x n=
4
or,
tan 2 x =1
π
2=x nπ +
4
nπ π
= x +
2 8
π π 3π
x = 0, , , , π
4 2 4
Answer: (D)

4. General solution of the equation

π 
(sin19°)cot x + cot
= 3x
sin 2 (2π − x) − cos(π − x)sin  + x 
2 

is


=
(A) x , n ∈
4


=
(B) x , n ∈ and n ≠ 4k , k ∈ 
4


=
(C) x , n ∈
2

Solution Manual for Mathematics For JEE (Main & Advanced), Trigonometry Vector
Algebra Probability, Vol 2,
Copyright©2017 Wiley India Pvt. Ltd. All rights reserved

=
(D) x , n ∈ and n ≠ 2k , k ∈ 
2

Solution:
sin(19°)cot x +=
cot 3 x
sin 2 ( x) − cos( x) ⋅[− cos( x)]= sin 2 ( x) + cos 2 ( x)= (1)°= 1
Hence,
cot x + cot 3x = 0

=
sin(4 x) 0,= 4 x nπ x= , n ∈ , n ≠ 4 k , k ∈ 
4
Answer: (B)

5. General solution of the equation

sin 3 ( x / 2) − cos3 ( x / 2) 1
= cos x
2 + sin x 3

is

π π
(A) x =
(4n + 1) , n ∈ (B) x =
(2n + 1) , n ∈
2 2


(C) x =(2n + 1)π , n ∈ =
(D) x , n ∈
4

Solution:
[sin( x / 2) − cos( x / 2)](1 + sin x / 2 ⋅ cos x / 2) cos x
=
2 + sin x 3
 x x  2cos x
 sin 2 − cos 2  =
  3
 x x   2   x   x   
 sin 2 − cos 2  1 − 3 sin  2  + cos  2    =0
         
Either,
x x
sin − cos = 0
2 2
x π 
tan = 1= tan  
2 4
π
x =2nπ + , n∈ z
2
or,
2 x x
1 −  sin + cos  =
0
3 2 2
x x 3
sin + cos =
2 2 2
This is impossible. Hence,
π
x =+
(4n 1) , n∈ z
2
Answer: (A)

Solution Manual for Mathematics For JEE (Main & Advanced), Trigonometry Vector
Algebra Probability, Vol 2,
Copyright©2017 Wiley India Pvt. Ltd. All rights reserved
6. General solution of the equation tan x sin x − cos x =
(1 / 2)sec x is

π π
(A) (2n ± 1) , n ∈ (B) (3n ± 1) , n ∈
3 3

π π
(C) (4n + 1) , n ∈ (D) (2n ± 1) , n ∈
6 6

Solution:
sin 2 x 1
− cos ( x) = sec ( x)
cos( x) 2
1
sin 2 ( x) − cos 2 ( x) =
2
1  2π 
cos(2 x) = − = cos  
2  3 
2π π
2x = 2nπ ± , x= nπ ±
3 3
Answer: (B)

7. If cos 2 3x + sin 2 [(π / 2) +=


5 x] cos 2 (7 x) + cos 2 (9 x), then 12x is equal to

(A) 4nπ , n ∈ (B) 3nπ , n ∈

(C) 2nπ , n ∈ (D) nπ , n ∈

Solution:
cos 2 3x + cos 2 5 x = cos 2 7 x + cos 2 9 x
⇒ sin 2 7 x + sin 2 9 x = sin 2 5 x + sin 2 3x
⇒ (sin 2 9 x − sin 2 5 x) + (sin 2 7 x − sin 2 3x) =
0
⇒ sin14 x ⋅ sin 4 x + sin10 x ⋅ sin 4 x = 0
⇒ sin 4 x(sin14 x + sin10 x) = 0
⇒ sin 4 x(2sin12 x ⋅ cos 2 x) = 0
⇒ sin12 x ⋅ sin 4 x ⋅ cos 2 x = 0
sin12 x =⇒ 0 12 x = nπ
Answer: (D)

8. The number of solutions of the equation 1 − 2sin x − 2 cos x + cot x= 0 (0 < x < 2π ) is

(A) 4 (B) 6 (C) 2 (D) 1

Solution:
1 + cot( x) − 2[sin x + cos( x) ] =0
(sin x + cos x)
− 2 (sin x + cos x) = 0
sin( x)
 1 
(sin x + cos x)  − 2=0
 sin x 

Solution Manual for Mathematics For JEE (Main & Advanced), Trigonometry Vector
Algebra Probability, Vol 2,
Copyright©2017 Wiley India Pvt. Ltd. All rights reserved
Either,
1 π 
sin( x)= = sin  
2 6
π
x= nπ + (−1) n
6
or,
 3π 
tan( x) =− 1 =tan  
 4 

=x nπ +
4
π 5π 3π 7π
x= , , ,
6 6 4 4

Answer: (A)
9. If cos 3x + sin[2 x − (7π / 6)] =
2, then x is of the form

π π
(A) (4m + 1) , m ∈ (B) (6m + 1) , m ∈
3 3

π π
(C) (3m + 1) , m ∈ (D) (2m + 1) , m ∈
3 3

Solution:
cos(3x) =− 1 =cos(π )
3x= 2nπ ± π = (2n ± 1)π
π
x =± (2n 1) , n∈ z
3
 7π   3π 
sin  2 x −  =−1 =sin  2 
 6   
7π 3π
2 x − = nπ + (−1)n
6 2
π 3π
2 x − = nπ + (−1)n
6 2
π π
− 2 x = (4n + 1)
6 2
π π
2 x = − (4n + 1)
6 2
On combining the two values
π
= x (6m + 1)
3
Answer: (B)

10. General solution of 1 + sin 3 x + cos3 x =


(3 / 2)sin 2 x is

 π 
(A) [(2n + 1)π , n ∈ ] ∪ (4k − 1) , k ∈  
 2 

Solution Manual for Mathematics For JEE (Main & Advanced), Trigonometry Vector
Algebra Probability, Vol 2,
Copyright©2017 Wiley India Pvt. Ltd. All rights reserved
 π 
(B) (nπ , n ∈ ) ∪ (2k − 1) , k ∈  
 2 

(C) [(4n + 1)π , n ∈ ] ∪ (2kπ , k ∈ )

(D) [(3n + 1)π , n ∈ ] ∪ (kπ , k ∈ )

Solution:
3
1 + (sin x + cos x) (1 − sin x ⋅ cos x) = sin 2 x
2
2 + (sin x + cos x) (2 − sin 2 x) = 3sin 2 x.
2 + (1 + sin x + cos x=) sin 2 x (3 + sin x + cos x)
π
sin x =
− 1, x =−
(4k 1)
2
cos( x) = (2n + 1)π
− 1, x =
Answer: (A)
=
11. If ( 3 / 2)sin x cos x + cos 2 x, then x is equal to

π
(A) nπ or (2n ± 1) , n ∈
3

π
(B) (2n ± 1)π or (6n ± 1) , n ∈
3

nπ π
(C) or (2n − 1) , n ∈
2 3

π
(D) 2nπ or nπ + , n ∈
3

Solution:
2
 3 
 =
sin( x)  [cos( x) + cos 2 x]2
 2 
3 2
sin ( x) = cos 2 ( x) + cos 4 ( x) + 2 cos3 ( x)
4
3
[1 − cos 2 ( x)] = cos 2 ( x) + cos 4 ( x) + 2 cos3 ( x)
4
4 cos 4 ( x) + 8cos3 ( x) + 7 cos 2 ( x) − 3 =0
4 cos3 ( x)[cos( x) + 1] + 4 cos 2 ( x)[cos( x) + 1] + 3cos( x)[cos( x) + 1] − 3[cos( x) + 1] =
0
[cos( x) + 1][4 cos3 ( x) + 4 cos 2 ( x) + 3cos( x) − 3] =
0
Either,
cos( x) + 1 =0 , cos( x) =−1 =cos(π )
=x (2n ± 1)π
or,
4 cos3 ( x) + 4 cos 2 ( x) + 3cos( x) − 3 =0

Solution Manual for Mathematics For JEE (Main & Advanced), Trigonometry Vector
Algebra Probability, Vol 2,
Copyright©2017 Wiley India Pvt. Ltd. All rights reserved
π 
cos( x) = cos  
3
=x (2n ± 1)π
or
π
(6n ± 1)
3
Answer: (B)

12. The general value of x satisfying the equation sin 2 x + 5sin x + 5cos x + 1 =0 is

π π
(A) (3n + 1) , n ∈ (B) (2n + 1) , n ∈
4 4

π π
(C) (4n − 1) , n ∈ (D) (6n + 1) , n ∈
4 4

Solution:
sin 2 x + 5sin x + 5cos x + 1 =
0 ⇒ 2sin x cos x + 5sin x + 5cos x + sin 2 x + cos 2 x =
0
⇒ (sin x + cos x)(sin x + cos x + 5) =
0
Now,
(sin x + cos x) = 0
or (sin x + cos x + 5) = 0, not possible
So, x lies in II or IV quadrant, that is,
3π 7π 11π
x= , ,
4 4 4

Answer: (C)

13. The number of solutions of the equation 2sin 3 x = cos x in the interval (−π, 5π) is

(A) 4 (B) 6 (C) 5 (D) 2

Solution:
2sin 3 ( x) = cos( x)
2
= cot( x)
cos ec2 ( x)
2
= cot( x)
1 + cot 2 ( x)
=2 cot( x) + cot 3 ( x)
cot 3 ( x) + cot ( x) − 2 =0
cot 2 ( x)[cot( x) −1] + cot( x)[cot( x) −1] + 2[cot( x) −1]
[cot( x) −1][cot 2 ( x) + cot( x) + 2] =
0
Either,
cot( x) =1 ,
π
=
x (4n + 1)
4

Solution Manual for Mathematics For JEE (Main & Advanced), Trigonometry Vector
Algebra Probability, Vol 2,
Copyright©2017 Wiley India Pvt. Ltd. All rights reserved
Answer: (B)

14. General solution of the equation sin x + cos=


x 2(sin 3 x + cos3 x) belongs to
 π   π 
(A) (2n − 1) , n ∈   ∪  nπ + (−1)n , n ∈  
 2   4 
 nπ  1    π 
(B)  + (−1)n sin −1   , n ∈   ∪ (2n + 1) , n ∈  
 2 2 2   2 
 1   π 
(C)  2nπ ± cos −1   , n ∈   ∪ (2n + 1) , n ∈  
  3   2 
 −π   nπ (−1) (2 − 2) 
n
(D)  + nπ , n ∈   ∪  + sin −1 , n ∈ 
 4   2 2 2 

Solution:
  1 
[(sin ( x) + cos ( x=
)] 2 [sin ( x) + cos ( x)] 1 + sin 2 x  
  2 
 1 
[sin ( x) + cos ( x)]  2 + sin 2 x −1 =
0
 2 
Either,
−π
0, x = + nπ
sin( x) + cos( x) =
4
or,
sin(2 x) + 2 − 2 =0
nπ (−1)n  2 −2 
2= + sin −1 
 2 
x
2 2  
Answer: (D)

15. If a > 0, the number of solutions of the equation=


sin x (| x | /2) + a in the interval (−2, 0)
is

(A) Infinite (B) 4 (C) 2 (D) 0

Solution: Here, there are no values of x, which satisfy the given equation.
Answer: (D)

16. The number of solutions of the equation

 2 +1   2 −1 
cos  x ⋅ cos  x =
1
 2   2 

in the interval (−π, π) is

(A) 0 (B) 1 (C) 2 (D) 4

Solution:

Solution Manual for Mathematics For JEE (Main & Advanced), Trigonometry Vector
Algebra Probability, Vol 2,
Copyright©2017 Wiley India Pvt. Ltd. All rights reserved
 2 +1   2 −1 
2 ⋅ cos  x  ⋅ cos  x=2
 2   2 
   
cos ( 2 x) + cos( x) =
2
then,
cos ( 2 x) =1
2 x = 2nπ
x = 2 nπ
and
cos ( x) =1
= nπ , x 0
x 2=
Answer: (B)
17. The number of solutions of the equation

π  π 
tan  + x=
 tan  − x 
4  4 

in the interval (−∞, ∞) is

(A) 0 (B) 2 (C) finite (D) infinite

Solution:
1 + tan x 1 − tan x
=
1 − tan x 1 + tan x
(1 + tan x) = ± (1 − tan x)
=
tan x 0= , x nπ , n∈ z
Answer: (D)

18. If the pair (x, y) satisfies both the equations sin 2 x + sin 2 y =
1 / 2 and x − y =4π / 3, then

7π −π
nπ +
(A) x = , y = + nπ , n ∈
6 6

5π π
(B) x =nπ + , y=nπ − , n ∈
6 3

7π π
(C) x =nπ − , y=nπ + , n ∈
6 6

5π π
(D) x =nπ − , y=nπ + , n ∈
6 3

Solution:
2sin 2 x + 2sin 2 y = 1
1 − cos 2 x + 1 − cos 2 y = 1
2 cos ( x + y ) ⋅ cos ( x − y ) =1

Solution Manual for Mathematics For JEE (Main & Advanced), Trigonometry Vector
Algebra Probability, Vol 2,
Copyright©2017 Wiley India Pvt. Ltd. All rights reserved
 4π 
2cos ( x + y ) ⋅ cos  =1
 3 
  1 
cos ( x + y )  2 ⋅  −   =1
  2 
cos ( x + y ) = −1
( x + y ) = 2nπ ± (π ) = (2n ± 1)π

and x − y=
3
7π −π
x= nπ + , y = + nπ
6 6
Answer: (A)

19. If (x, y) satisfies the system of equations tan x + tan y =


4 and cot x + cot y =
5, then
nπ + t an −1[(10 ± 4 5) / 5], y =
(A) x = mπ + t an −1[(10  4 5) / 5], n and m ∈ (either
both have upper signs or both have lower signs)
(B) x = (nπ / 2) + t an −1[2 ± (0.8) 5], y = (mπ / 2) + tan −1[2  (0.8) 5], n, m ∈ 
(C) x =2nπ + t an −1 (±4 / 5), y =2mπ + t an −1 (4 / 5), n, m ∈ 
[(2n + 1)π / 2] + tan −1 (2 ± 5), y =
(D) x = [(2m + 1)π / 2] + tan −1 (2  5), n, m ∈ 

Solution:
tan x + tan y =4 (1)
cot x + cot y =5
tan x + tan y
⇒ = 5
tan x tan y
4
⇒ tan x tan y = (2)
5
Solve Eqs. (1) and (2)
4
tan x + =4
5 tan x
5 tan 2 x + 4 =20 tan x
5 tan 2 x − 20 tan x + 4 =0
10 ± 4 5
tan x =
5
 10 ± 4 5 
⇒ x = n π + tan −1  
 5 
From Eq. (2)
4 5
tan y= ⋅
5 10 ± 4 5
10  4 5
tan y =
5
10  4 5
⇒ y= mπ +
5
Answer: (A)

Solution Manual for Mathematics For JEE (Main & Advanced), Trigonometry Vector
Algebra Probability, Vol 2,
Copyright©2017 Wiley India Pvt. Ltd. All rights reserved
20. The number of solutions in the interval (−π, π) of the equation
4sin 3x + sin 5 x − 2sin x cos 2 x =
0 is

(A) 8 (B) 7 (C) 6 (D) 4

Solution:
4sin 3x + sin 5 x − (sin 3x − sin x) =
0
3sin 3x + sin 5 x + sin x = 0
3sin 3x + 2sin 3x ⋅ cos 2 x = 0
sin(3x)[3 + 2 cos(2 x)] = 0
Either
sin (3x)= 0= sin (nπ )
3x = − π , 0, π , − 2π , 2π ,3π , − 3π
−π π −2π 2π
= x , 0, , , ,π , − π
3 3 3 3
or,
−3
cos(2 x) =
2
This is not possible.
Answer: (B)
21. The number of values of x such that tan[(2π x) / (1 + x + x 2 )] =− 3 is

(A) 8 (B) 7 (C) 5 (D) 6

Solution:
 2π x   2π 
tan  2 
 =− 3= tan  
 1+ x + x   3 
2π x 2π
=
1+ x + x 2
3
−π 5π
or, or
3 3
x 2 + x + 1 =+ 3x
x 2 − 2 x=
+ 1 0, =x 1
2π x −π
or, =
1+ x + x 2
3
x 2 + x + 1 =− 6 x
x 2 + 7 x + 1 =0
−7 ± 3 5
x=
2
Hence x = 5 value or,
2π x 5π
=
1+ x + x 2
3
5x2 − x + 5 =0

Solution Manual for Mathematics For JEE (Main & Advanced), Trigonometry Vector
Algebra Probability, Vol 2,
Copyright©2017 Wiley India Pvt. Ltd. All rights reserved
1 ± 26
x=
10
Answer: (C)

22. The general solution of the equation cos 2 x + sin 2 x =cos x + sin x belongs to

 π 
(A) (2nπ , n ∈ ) ∪ (4m + 1) , m ∈  
 6 

 π 
(B) (nπ , n ∈ ) ∪ (2m + 1) , m ∈  
 6 

 π 
(C) (nπ , n ∈ ) ∪ (4m + 1) , m ∈  
 3 

 nπ   π 
(D)  , n ∈   ∪ (2m + 1) , m ∈  
 2   3 

Solution:

cos 2 x + sin 2 x =cos x + sin x


(sin 2 x − sin x) + (cos 2 x − cos x) =
0
x 3x 3x  −x 
2sin cos + 2sin sin   = 0
2 2 2  2 
x 3x 3x 
2sin  cos − sin  = 0
2 2 2 
x
⇒ sin = 0 ⇒ x = 2nπ , n ∈ z
2
3x 3x
cos − sin = 0
2 2
3x
⇒ tan = 1
2
3x π
⇒ = nπ +
2 4
2nπ π
= x +
3 6
π
= (4n + 1) ⋅n∈ z
6
Answer: (A)
23. The number of solutions of the equation 2 sin x + cot x =
0 in the interval (−π, 2π) is

(A) 6 (B) 4 (C) 3 (D) 2

Solution:
2 sin x + cot x =
0
2 sin 2 ( x) + cos( x) =
0

Solution Manual for Mathematics For JEE (Main & Advanced), Trigonometry Vector
Algebra Probability, Vol 2,
Copyright©2017 Wiley India Pvt. Ltd. All rights reserved
2 (1 − cos 2 x) + cos( x) =
0
2 cos 2 x − cos( x) − 2 =
0
1± 1+ 8 1± 3
=
cos( x) =
2 2 2 2
2 1
cos(=x) ,−
2 2
 3π 
cos( x) = cos  
 4 

= x 2nπ ±
4
−3π 3π 5π
x= , ,
4 4 4
Answer: (C)

24. The number of solutions of the equation 2(sin 3 x + cos3 x) − 3(sin x + cos x) =
8 in the
interval (−π, π) is

(A) infinite (B) 4 (C) 2 (D) 0

Solution:
2 (sin x + cos x) (1 − sin x ⋅ cos x) − 3(sin x + cos x) =
8
(sin x + cos x) (sin 2 x + sin x + cos x) = −8
Now,
(sin x + cos x) ≥ − 2
and (sin 2 x + sin x + cos x) ≥ − (1 + 2)
Hence
Left side ≠ Right side
so, number of solution x = 0
Answer: (D)

25. The equation sin 6 x + cos6 x =


a has a solution if and only if

1 1
(A) 1 ≤ a ≤ 2 (B) ≤a≤
6 3

1 1 1
(C) ≤ a ≤1 (D) ≤a≤
4 8 4

Solution:
(sin 2 x + cos 2 x) (sin 4 x + cos 4 x − sin 2 x ⋅ cos 2 x) =
a
(1 − 3sin 2 x ⋅ cos 2 x) =
a
 3 
1 − 4 ⋅ sin 2 x  =
2
a
 
3 2
sin 2 x = 1 − a
4

Solution Manual for Mathematics For JEE (Main & Advanced), Trigonometry Vector
Algebra Probability, Vol 2,
Copyright©2017 Wiley India Pvt. Ltd. All rights reserved
3
(1 − cos 4 x) = 1− a
4
3cos 4 x = 8a − 8 + 3 = 89 − 5
As,
−1≤ cos 4 x ≤1
− 3 ≤ 89 − 5 ≤ 3
2 8 1
≤ 9 ≤ , ≤ 9 ≤1
8 8 4
Answer: (C)

26. The number of solutions of the equation cos7 x + sin 4 x =


1 in the interval (−π, π) is

(A) 2 (B) 3 (C) 4 (D) 5

Solution:
cos7 ( x) + sin 4 ( x) =
1
cos7 ( x) − cos 2 x (1 + sin 2 x) =
0
cos 2 ( x)[cos5 ( x) −1 − sin 2 x] =0
Either
cos( x)= 0 ⇒ 2
π
x=
(2n ± 1) , n = z
2
or,
cos 2 ( x) + cos5 ( x) =
2
cos(= x) 1= , x 2nπ
Answer: (B)

27. The smallest positive value of x (in degrees) for which


tan( x + 100=
°) tan( x + 50°) tan x tan( x − 50°) is

(A) 30° (B) 45° (C) 55° (D) 60°

Solution:
tan ( x + 100) = tan ( x + 50a ) tan( x) ⋅ tan ( x − 50a )
tan ( x + 100)
=tan ( x + 50) tan ( x − 50)
tan x
sin (2 x + 100)  sin 2 x − sin 2 50 + cos 2 x − sin 2 
= −  
sin (100 )  cos x − sin 50 − sin x + sin
2 2 2 2

sin (2 x + 100) 2sin 2 50 −1
=
sin (100) cos 2 x
sin (2 x + 100) ⋅ cos (2 x) = − sin100 ⋅ cos100
sin (4 x + 150
) + sin (100 ) =
− sin 200
sin (2 x + 150
)=
− [sin (200 ) + sin (100)]
− sin150
sin (2 x + 100) = ⋅ cos 50
sin 40
sin (2 x + 100) =

Solution Manual for Mathematics For JEE (Main & Advanced), Trigonometry Vector
Algebra Probability, Vol 2,
Copyright©2017 Wiley India Pvt. Ltd. All rights reserved
40
2 x + 100 =
2 x = 60
x = 30
Answer: (A)

28. The smallest positive value of p for which the equation cos( p sin x) = sin( p cos x) has a
solution is

π π π π
(A) (B) (C) (D)
2 3 3 2 2 2

Solution:
p
p sin x= − p cos x
2
p
p (sin x + cos x) =
2
Now,
sin x + cos x ≤ 2
p
p ( 2) ≥
2
p
p=
2 2
Answer: (D)

29. The number of integral values of k for which the equation 7 cos x + 5sin x =
2k + 1 has a
solution is

(A) 4 (B) 8 (C) 10 (D) 12

Solution:
7 cos x + 5sin x =+
2k 1
 7 5 
± 74  cos x + sin x  =
2k + 1
 74 74 
− 74 ≤ 2k + 1≤ 74
This is satisfy for
k =−4, −3, −2, −1, 0, 1, 2, 3
Answer: (B)

sin 3x in the interval (0, π) is


30. The number of solutions of the equation sin x + sin 5 x =

(A) 6 (B) 5 (C) 4 (D) 3

Solution:
sin x + 2sin x ⋅ cos 4 x = 0
sin x (1 + 2 cos 4 x) =0
=
sin x 0,= x nπ =, x 0, π

Solution Manual for Mathematics For JEE (Main & Advanced), Trigonometry Vector
Algebra Probability, Vol 2,
Copyright©2017 Wiley India Pvt. Ltd. All rights reserved
1
cos 4 x = −
2
2π π π 2π 4π
4x =2nπ ± ,x= , , ,
3 6 3 3 3
Answer: (A)

=
31. The number of solutions of the equation tan 2
x cos 2 x − 1 in the interval (−π, π) is

(A) 6 (B) 5 (C) 4 (D) 3

Solution:
tan 2 x = − 2sin 2 x
 1 + 2 cos 2 x 
sin 2 x  =0
 cos 2 x 
 
sin ( x) = nπ , x = − π , 0, π
2cos 2 x = − 1
Not possible
Answer: (D)

MULTIPLE CORRECT CHOICE TYPE QUESTIONS

1. Solution of the equation 1 + sin 4 2 x =


cos 2 6 x is

(A) nπ / 3, n ∈ (B) nπ / 2, n ∈

(C) nπ , n ∈ (D) (2n + 1)π / 4, n ∈

Solution:
1 + sin 4 2 x =
cos 2 6 x
sin 2 6 x + sin 4 2 x =
0
(3sin 2 x − 4sin 3 2 x)2 + sin 4 2 x =
0
16sin 6 2 x − 23sin 4 (2 x) + 9sin 2 (2 x) =
0
sin 2 (2 x)[16sin 4 (2 x) − 23sin 2 (2 x) + 9] =
0
Either,

=
sin 2x 0 =
, x , n∈ z
2
or,
16sin 4 (2 x) − 23sin 2 (2 x) + 9 =0
[4sin 2 (2 x) − 3]2 + sin 2 2 x =
0
that is
4sin 2 2 x − 3 =0
3 π 
sin(2 x) = = ± sin  
2 3

Solution Manual for Mathematics For JEE (Main & Advanced), Trigonometry Vector
Algebra Probability, Vol 2,
Copyright©2017 Wiley India Pvt. Ltd. All rights reserved
nπ ± π
=x (−1) n
2 6
Answer: (B), (C)

2. If 7 cos 2 x + cos x sin x − 3 =0, then x is

(A) nπ + tan −1 (4 / 3), n ∈ 

(B) nπ + (3π / 4), n ∈ 

(C) (nπ / 2) + tan −1 (4 / 3), n ∈ 

(D) (nπ / 2) + (3π / 4), n ∈ 

Solution:
7 + tan x + (−3)sec2 ( x) =0
7 + tan x =
3 + 3tan 2 x
3tan 2 x − tan x − 4 = 0
(3tan x − 4) (tan x + 1) = 0
Either,

tan x = nπ +
−1, x = , n∈ z
4
or,
4
3tan x = nπ + tan −1   , n∈ z
4, x =
3
Answer: (A), (B)
− sin 2 x
3. If 2 cos 2 x
+1 = 3⋅ 2 , then x maybe equal to

(A) nπ (B) 2nπ

(C) nπ / 2 (D) (2n ± 1)π / 2(n ∈ )

Solution:
Let
2− sin
2
x
=z
then,
21− 2 sin + 1 = 3 ⋅ 2− sin
2 2
x x

2 ⋅ ( z )2 + 1 =3( z )
2 z 2 − 3z + 1 =0
2 z 2 − 3z + 1 =0
(2 z − 1) ( z − 1) =0
z =1 and 1/2 or 2−1
so,
2− sin = 1= 2
2
x

Solution Manual for Mathematics For JEE (Main & Advanced), Trigonometry Vector
Algebra Probability, Vol 2,
Copyright©2017 Wiley India Pvt. Ltd. All rights reserved
Either,
=
sin x 0,=x nπ , n ∈ z
or,
2nπ π
sin 2 x = 1, x = ± , n∈ z
2 2
Answer: (A), (B), (D)

(3 + 2i sin θ ) / (1 − 2i sin θ ), then


4. If z =

if θ nπ , n ∈
(A) z is pure real,=

=
(B) z is pure real, if θ 2nπ , n ∈

(C) z is pure imaginary, if θ =


nπ + (π / 3), n ∈ 

(D) z is pure imaginary, if θ =


nπ − (π / 3), n ∈ 

Solution:
(3 + i 2sin θ ) (1 + i 2sin θ )
=Z ⋅
(1 − i 2sin θ ) (1 + i 2sin θ )
3 − 4sin 2 θ + 7 i sin θ
Z=
1 + 4sin 2 θ
3 − 4sin 2 θ 7 sin θ
=Z +i
1 + 4sin θ 2
1 + 4sin 2 θ
(A) For, θ = nπ ,
Imaginary terms
7(0)
i =0
1 + 4(0)
Hence Z = pure seal, True
(B) For θ = 2nπ
Imaginary terms = 0. True
(C) For θ nπ + π / 3
=
Real part = 0, Z = Imaginary
π
(D) For θ nπ − ,
=
3
Real part = 0, Z = Imaginary
Answer: (A), (B), (C), (D)

5. If α and β are smallest positive values of x and y satisfying the equation


2(sin x + sin y ) − 2 cos( x − y ) =
3, then

(A) sin[(α + β ) / 2] =
1

(B) cos[(α − β ) / 2] =
1/ 2

(C) the number of pairs (α, β) is 2

(D) the number of pairs (α, β) is 4

Solution Manual for Mathematics For JEE (Main & Advanced), Trigonometry Vector
Algebra Probability, Vol 2,
Copyright©2017 Wiley India Pvt. Ltd. All rights reserved
Solution:
 x+ y x− y   x− y 
2  2sin ⋅ cos  − 2  2cos 2 −1 = 3
 2 2   2 
 x− y   x+ y   x − y 
4 cos   sin   − cos   = 1
 2   2   2 
As α, β are solution, so,
α −β  α +β α −β 
− cos =
2  2 
4cos sin 1
2
Now, for,
α +β α −β 1
sin =1 and cos =
2 2 2
1 1
4 ⋅ 1 −  = 1
2 2
Hence, options (A) and (B) are true
α +β
sin = 1, α + = β π
2
α −β π π 5π π
and = or, − , α = ,
2 3 3 6 6
2π −2π π 5π
= α −β ,= β ,
3 3 6 6
Answer: (A), (B), (C)

6. For the equation sin 2 x + (1 / 4)sin 2 3x =


sin x sin 2 3x

(A) x = 13π / 6 is a solution

(B) x = 0, π are solutions

(C) the number of solutions in the interval (−π, π) is 5

(D) x = π / 2 is not a solution

Solution:
1
sin 2 ( x) + (9sin 2 x + 16sin 6 x − 24sin 4=
x) sin x(9sin 2 x + 16sin 6 x − 24sin 4 x)
4
⇒ 64sin 7 x −16sin 6 x − 96sin 5 x + 24sin 4 x + 36sin 3 x −13sin 2 x =
0
sin 2 x (64sin 5 x −16sin 4 x − 96sin 3 x + 24sin 2 x + 36sin x −13) =
0
Either,
sin=
( x) 0,=x nπ =
, x 0, π
or,
64sin 5 x −16sin 4 x − 96sin 3 x + 24sin 2 x + 36sin x −13 =
0
13π
x=
6
Also, for,
π
x=
2
64 −16 − 96 + 24 + 36 −13 ≠ 0

Solution Manual for Mathematics For JEE (Main & Advanced), Trigonometry Vector
Algebra Probability, Vol 2,
Copyright©2017 Wiley India Pvt. Ltd. All rights reserved
Answer: (A), (B), (C), (D)

7. For the equation tan x ⋅ tan 4 x =


1

(A) The number of solutions in (0, π) is 5

(B) x = 3π / 4 is a solution

(C) x = 11π / 4 is not a solution

x (2n + 1)π / 10 is the general solution


(D)=

Solution:

tan x ⋅ tan 4 x =
1
sin x ⋅ sin 4 x =cos x ⋅ cos 4 x
π 
cos5 x= 0= cos  
2
Here
π 3π 5π 7π 9π
x= , , , ,
10 10 10 10 10
π
=
5 x (2n + 1)
2
π
=
x (2n + 1)
10
Answer: (A), (C), (D)

8. For the equation cos x cos[(π / 2) − x]cos[(π / 3) + x] =


1/ 4 ,

(A) The number of solutions in (−π, π) is 3

(B) The number of solutions in (0, 6π) is 10

(C) The sum of all the solutions in (0, 6π) is 30π

=
(D) The general solution is x 2nπ / 3, n ∈

Solution:
π  π 
4cos( x) ⋅ cos  − x  ⋅ cos  + x  =1
3  3 
 π 
2cos( x) cos + cos( x)  = 1
 3 
cos( x) + 2 cos 2 x = 1
cos( x) + cos 2 x = 0
3x x
cos ⋅ cos = 0
2 2
Either,

Solution Manual for Mathematics For JEE (Main & Advanced), Trigonometry Vector
Algebra Probability, Vol 2,
Copyright©2017 Wiley India Pvt. Ltd. All rights reserved
x π 
cos = 0,= cos  
2 2
x π
= (2n ± 1)
2 2
=x (2n ± 1)π
or,
3x π
cos = 0= cos
2 2
π
=
x (2n ± 1)
3
−π π π
(A) x= , ,
2 3 2
π 3π 5π 7π 9π 11π π 5π 7π 13π
(B) (0, 6π ), x =
, , , , , , , , ,
2 2 2 2 2 2 3 3 3 3
(C) Sum of all value of x
180π
= 30π
6
(D) General solution
2nπ
x=
3
Answer: (A), (B), (C), (D)

9. Let f (=
x) sin 4 x + 4(1 − sin 2 x) − cos 4 x + 4(1 − cos 2 x). Then

(A) General solution of f (x) = 0 is nπ , n ∈

(B) The number of values of x belonging to (−π, π) satisfying f (x) = 0 is 4

(C) General solution of f (x) = 0 is 2nπ , n ∈

(D) The sum of all the solutions of f (x) = 0 belonging to (−2π, 2π) is 0

Solution:

=
f ( x) [sin 2 ( x) − 2]2 − (cos 2 x − 2) 2
= sin 2 ( x) − 2 − cos 2 x + 2
f ( x) = − cos (2 x)
Now, when
f ( x) = 0
cos (2 x) = 0
π
2=x (2n ± 1)
2
π
= x (2n ± 1)
4
−3π −π π 3π
(B) x= , , ,
2 4 4 4
(D) Sum of all x’s = 0
Answer: (B), (D)
Solution Manual for Mathematics For JEE (Main & Advanced), Trigonometry Vector
Algebra Probability, Vol 2,
Copyright©2017 Wiley India Pvt. Ltd. All rights reserved
10. The region S consists of all the points (x, y) such that x 2 + y 2 ≤ 100 and sin( x + y ) ≥ 0.
Then

(A) S is cut by the lines of the form x + y =kπ for some k ∈

(B) If (x, y) belongs to the region S and lies between two lines of the form x + y =kπ ,
then either sin( x + y ) > 0 or sin( x + y ) < 0

(C) The area of the region S is 50π square units

(D) The area of the region S is 100π square units

Solution:
x 2 + y 2 ≤ 100 and sin( x + y ) ≥ 0
2nπ ≤ x + y ≤ (2n + 1)π

(A) Clearly line x + y = kπ cuts region S, for some k ∈ z.


(B) Option (B) is true.
(C) Clearly region S is the half of area of circle.
1
= S = π (10)2 50π
2

Answer: (A), (B), (C)


Solution Manual for Mathematics For JEE (Main & Advanced), Trigonometry Vector
Algebra Probability, Vol 2,
Copyright©2017 Wiley India Pvt. Ltd. All rights reserved
11. Consider the equation

3 −1 3 +1
+ =
4 2
sin x cos x

For this equation

(A) x π / 12, n ∈ , is a solution


=

(B) x 11π / 36, n ∈ , is a solution


=

(C) the number of solutions in (0, π / 2) is 2

(D) 2 x= nπ + ( −1)n ( x + π / 12)

Solution:
3 −1 3 +1
+ =
4 2
sin x cos x
3 +1 3 −1
sin x ⋅ + cos x ⋅ =
sin 2 x
2 2 2 2
π π
sin x ⋅ cos + cos x ⋅ sin
= sin 2 x
12 12
 π 
sin  π +  = sin 2 x
 12 
 π 
⇒ 2 x= nπ + (−1)n  x + 
 12 
Let n = 2m (even)
π
x 2mπ + x +
2=
12
π
=x 2mπ +
12
π
If =
m 0,=
x lim
12 x →∞
let n = 2m + 1 (odd)
π
2 x= (2m + 1)π − x −
12
11π
3x 2mπ +
=
12
2mπ 11π
=x +
3 36
11π
If =
m 0,=
x
36

Answer: (A), (B), (C), (D)

12. In triangle ABC, the angles A and B satisfy the relations 3sin A + 4cos B =
6 and
3cos A + 4sin B = 1. Then

Solution Manual for Mathematics For JEE (Main & Advanced), Trigonometry Vector
Algebra Probability, Vol 2,
Copyright©2017 Wiley India Pvt. Ltd. All rights reserved
(A) A + B = 30° (B) C = 150°

(C) A + B = 150°(D) C = 30°

Solution:
3sin A + 4 cos B =6
and 3cos A + 4sin B = 1
Squaring and adding the above two equations
(3sin A + 4 cos B) 2 =
(6) 2
(3cos A + 4sin B) 2 =(1) 2
9 + 16 + 24sin ( A + B) =
37
24sin ( A + B) =12
1
sin ( A + B) = =sin (150
) or sin(30 )
2
150
A+ B=
30 , we get
For A + B =
sin A< 0.5
so,
3sin A + 4 cos B < 6

Hence A + B =
30 is not correct
Answer: (C), (D)

13. Consider the following two statements:


I. If 0 < x < π / 2 and logsin x cos x
2, then x = π / 4.
cos x + log sin x =
II. If α and β are any two angles such that sin α = sin β and cos α = cos β , then
α −=
β 2nπ , n ∈ .

Which of the following are correct?

(A) I is true (B) II is false

(C) I is false (D) II is true

Solution:
(I) log cos x sin x + logsin x cos x = 2
log sin x log cos x
+ =
2
log cos x log sin x
(log sin x) 2 + (log cos x) 2 − 2(log sin x)(log cos x) =
0
⇒ (log sin x − log cos x)2 =
0
⇒ log sin x = log cos x
π
⇒x=
4
(II) sin a sin
= = β and cos a cos β
α +β α −β
⇒ 2 cos sin =
0
2 2

Solution Manual for Mathematics For JEE (Main & Advanced), Trigonometry Vector
Algebra Probability, Vol 2,
Copyright©2017 Wiley India Pvt. Ltd. All rights reserved
α +β α − β 
and −2sin ⋅ sin  =0
2  2 
α − β 
⇒ sin  = 0
 2 
satisfy both equations
⇒ α −=
β 2nπ , n ∈ z
Answer: (A), (D)

14. Consider the following statements


I. The equation cos x + cos 3x = cos 2 x has two solutions in the interval (0, π / 2) .
II. If 1 + sin x ⋅ sin 2 x = , then x 2nπ / 3, n ∈ .
cos x cos 2 x=

Which of the following are correct?

(A) I is true

(B) II is true

(C) I is true, but II is false

(D) I is false, but II is true

Solution:
P → 2 cos 2 x ⋅ cos x =
cos 2 x
cos 2 x (2cos x − 1) =0
Either,
π
=
cos 2 x 0,=x
4
or,
2 cos ( x) − 1 =0
1 π
=
cos ( x) = , x
2 3
Q=
→1 cos (2 x= + x) cos 3x
2nπ
x=
3
Answer: (A), (B)

15. Consider the equation 2 2 | cos x |= cosec x.


(A) This has four solutions in (0, 2π).
(B) This has two solutions in (π / 2, π ) .
(C) Solutions of this equation belonging to (0, π) form an AP in some order.
(D) This equation has no solutions in (π ,3π / 2).

Solution:
1
2 cos x ⋅ sin x =
2
1 π
sin 2 x =
+ =sin
2 4

Solution Manual for Mathematics For JEE (Main & Advanced), Trigonometry Vector
Algebra Probability, Vol 2,
Copyright©2017 Wiley India Pvt. Ltd. All rights reserved
π 3π 5π 7π
2x = + , , ,
4 4 4 4
π 3π 5π 7π
x= , , ,
8 8 8 8

Answer: (A), (C)

MATRIX-MATCH TYPE QUESTIONS


In each of the following questions, statements are given in two columns, which have to be
matched. The statements in column I are labeled as (A), (B), (C) and (D), while those in column II
are labeled as (p), (q), (r), (s) and (t). Any given statement in column I can have correct matching
with one or more statements in column II. The appropriate bubbles corresponding to the answers
to these questions have to be darkened as illustrated in the following example.

Example: If the correct matches are (A) → (p), (s) , (B) → (q), (s), (t), (C) → (r), (D) → (r), (t) ,
that is if the matches are (A) → (p) and (s); (B) → (q), (s) and (t); (C) → (r); and (D) → (r), (t),
then the correct darkening of bubbles will look as follows:

1. Match the items in Column I with those in Column II.

Column I Column II

(A) If cos 3x − cos 2 x =


sin 3x, then some of the values of x can be −π 3π
(p) ,
4 4

(B) If sin 3x − 4sin x cos 2 x =


0, then x can be (q) –π, π, 2π

(C) If sec2 x − tan 2 x + cot(π / 2 + x) =


cos 2 x sec2 x, then x can be π 5π
(r) ,
4 4

(D) If tan x + sin(π + x) =


2sin 2 x / 2, then x is 2nπ , n ∈ π
(s) (t) (4n − 1) , n ∈
4

Solution:
(A)
cos 3x − cos 2 x =
sin 3x
cos 3x − sin 3x =cos 2 x

Solution Manual for Mathematics For JEE (Main & Advanced), Trigonometry Vector
Algebra Probability, Vol 2,
Copyright©2017 Wiley India Pvt. Ltd. All rights reserved
⇒ 4(sin 3 x + cos3 x) − 3(sin x + cos x) = cos 2 x − sin 2 x
⇒ sin x + cos x = 0
and 7 − 4sin x cos x =−
cos x sin x
⇒ sin x =0 and cos x =1 ⇒ x =2nπ
and sin x + cos x = 0
π
⇒ tan x =−1 ⇒ x =nπ −
4
Answer: (p), (s), (t)
(B) =
sin 3x 2sin 3x − 2sin x
2sin x = sin 3x
=
2sin x 3sin x − 4sin 3 x
sin x (1 − 4sin 2 x) =
0
Either
=
sin x 0,=x nπ
1 π 5π
sin x = ± , x= ,
2 6 6
Answer: (q), (s)

 1 − tan 2 x 
=
1 + − tan ( x)  (1 + tan 2 x)
 1 + tan 2 x 
(C)
 
[1 − tan( x)] =
(1 − tan 2 x)
π 5π −π 3π
1 − tan x = 0, x = , , ,
4 4 4 4
Answer: (p), (r)

(D)
sin x x
(1 − cos x) =2sin 2
cos x 2
2 x
sin (sin x − cos x) = 0
2
x
=sin =
sin (nπ ), x 2nπ
2
π 5π
=
tan x 1,= x ,
4 4
Answer: (r), (s)

2. Some trigonometric equations are given in Column I and some or whole of their solutions
are given in Column II. Match these.

Column I Column II

(A) If 0 ≤ x, y ≤ 2π and α and β are solutions of the equation π


(p)
2, then α + β is
sin x + sin y = 3

Solution Manual for Mathematics For JEE (Main & Advanced), Trigonometry Vector
Algebra Probability, Vol 2,
Copyright©2017 Wiley India Pvt. Ltd. All rights reserved
(B) If 4cos 2 x + 4cos x − 3 =0, then the value of x in the interval 5π
(q)
(0, 2π) is 3

(C) The solution of the equation 2π


(r)
2
0 in the interval (0, π) is
2sin x + (4 − 3)sin x − 2 3 = 3

(D) Solutions of the equation x tan(π / 3 − x) are


3 − tan= π
(s)
2

(t) π

Solution:
(A) sin α + sin β =2
sin α = 1= sin β
π π
α
= , = β , (α + β
=) π
2 2
Answer: (t)

(B) 4 cos 2 ( x) + 6 cos( x) − 2 cos( x) − 3 =


0
2 cos( x)[2 cos( x) + 3] −1(2 cos x + 3) = 0
[2 cos( x) + 3][2 cos( x) −1] = 0
1 π
=
cos( x) = , x
2 3
Answer: (p), (q)

(C)
( 3 − 4) ± 19 − 8 3 + 16 3
sin ( x) =
2 (2)
( 3 − 4) ± 4 + 3
=
4
3
= sin=
( x) ,−2
2
π
x=
3
Answer: (p), (r)

3 − tan x
(D) 3 − tan x =
1 + 3 tan x
( 3 − tan x) ( 3 tan x) =
0

Solution Manual for Mathematics For JEE (Main & Advanced), Trigonometry Vector
Algebra Probability, Vol 2,
Copyright©2017 Wiley India Pvt. Ltd. All rights reserved
π
=
tan x =
3, x
3
= , x π
tan x 0=
Answer: (p), (t)

3. Certain trigonometric equations are given in Column I and some of their solutions are
given in Column II. Match these.

Column I Column II

(A) sin 2 x =
sin 3x + cos x(cos x − 1) 2nπ
(p) , n ∈
3

1 + tan x π
(B) = (sin x + cos x) 2 (q) nπ , (4k − 1)
1 − tan x 4

(C) tan 3x + cos 6 x =


1 π
(r) (2n + 1)
6

(D) cos 4 x + 2cos 2 x =


1 nπ π
(s) , (4k + 1)
3 12

π
(t) (4k + 1)
4

Solution:
(A) sin 2 x =3sin x − 4sin 3 x + cos 2 x − cos x
cos 2 x + sin 3x =cos x
2nπ
x=
3
Answer: (p)

 1 − cos 2 + sin 2 x 
(sin x + cos x)  =
 cos x − sin x 
(B) 0
 
π 
tan  + x  ⋅ 2sin 2 x = 0
4 
Either
=
sin x 0,=x nπ
π 
tan  + x  =0 =nπ
 4 

Solution Manual for Mathematics For JEE (Main & Advanced), Trigonometry Vector
Algebra Probability, Vol 2,
Copyright©2017 Wiley India Pvt. Ltd. All rights reserved
π
=
x (4n −1)
4
Answer: (q)

(C) tan 3x (1 − sin 6 x) =0



=
tan 3x n= π, x
3
π
sin 6=
x 1, = x (4k + 1)
12
Answer: (s)

(D) cos 4 x + cos 2 x =


0
cos 3x ⋅ cos x =
0
 2n + 1  π π
=
x  = (2n + 1)
 3 2 6
Answer: (r)

4. Certain trigonometric equations are given in Column I and their solution sets are given in
Column II. Match these.

Column I Colum II

(A) 1 + sin 4 x =
cos5 x (p) Empty set

(B) sin 4 x= 1 + cos6 x (q) 2nπ , n ∈

(C) sin 2 2 x= 1 + cos 4 x π


(r) 2nπ + , n ∈
6

(D) =
sin 4 x 2sin 2 x + 1 (s) (2n + 1)π / 2, n ∈

Solution:
(A) 1 + sin 4 ( x) = cos5 ( x) = cos5 ( x)
This is possible when sin 4 ( x) = 0
x = 2nπ
Answer: (q)

(B) sin 4 ( x) = 1 + cos5 ( x)


π
This is true when=
x (2n + 1)
2
Answer: (s)

Solution Manual for Mathematics For JEE (Main & Advanced), Trigonometry Vector
Algebra Probability, Vol 2,
Copyright©2017 Wiley India Pvt. Ltd. All rights reserved
(C) cos 2 (2 x) = cos 4 x =
2 cos(2 x) cos(2 x) + 1
2 cos (2 x) = 2 cos x
2
cos (2 x) = 1= Empty
Answer: (p)

(D) =
sin 4
( x) 2sin 2 x + 1
When sin ( x) = 0 empty set
Answer: (r)

5. Match the items in Column I with those in Column II.

Column I Column II

(A) The number of solutions of the equation (p) 0


2cos x in the interval (0, 2π) is
tan x + sec x =

(B) The number of solution of the equation (q) 3


tan 3x − tan 2 x
= 1 in (0, 2π) is
1 + tan 3x tan 2 x

(C) The number of values of x in (0, π) satisfying the (r) 4


equation sin x + sin 3x + sin 5 x =
0 is

(D) The number of values of x in (0,3π / 2) satisfying (s) 2


the equation sin x(2 cos x + 1) =
0 is

(t) 6

Solution:
1 + sin x
(A) = 2 cos ( x)
cos ( x)
1 + sin ( x) = 2cos 2 ( x)
sin ( x) = cos (2 x)
2sin 2 x + sin x −1 =0
[2sin ( x) −1][sin ( x) + 1] =
0
1 π 5π
=
sin ( x) = , x ,
2 6 6

sin ( x) =− 1, x =
2
Answer: (q)

Solution Manual for Mathematics For JEE (Main & Advanced), Trigonometry Vector
Algebra Probability, Vol 2,
Copyright©2017 Wiley India Pvt. Ltd. All rights reserved
(B) tan ( x) =1
π
x=
4
π
But at x =
4
tan 2x = ∞
so x=0
Answer: (p)

(C) sin 3x (2 cos 2 x + 1) =


0
π 2π
x = 0, ,
3 3
Answer: (q)

1
(D) sin x = 0, cos x = −
2
2π 4π
= π, x
x 0,= ,
3 3

Answer: (r)

COMPREHENSION TYPE QUESTIONS

1. Passage: Using trigonometric transformations, answer the following questions.


(i) The number of solutions of the equation sin 2 x ⋅ sin 6 x =
cos x cos 3x in the
interval (0, π / 2) is
(A) 4 (B) 6 (C) 2 (D) 8
(ii) The number of solutions of the equation
tan x + tan( x + π / 3) + tan( x + 2π / 3) =3 in the interval (−π, π) is
(A) 2 (B) 3 (C) 4 (D) 6
(iii) The number of solutions of the equation sin x + sin 7 x =sin 4 x in the interval
(0, π / 2) is
(A) 2 (B) 3 (C) 4 (D) 6

Solution:
(i) 2sin x ⋅ sin 3x ⋅ 2 ⋅ cos x ⋅ cos 3=
x cos x ⋅ cos 3x
(cos x ⋅ cos 3x) (4sin x ⋅ sin 3x − 1) =
0
π
=
cos x 0,= x
2
π π
cos=3x 0= ,x ,
6 2
2 (cos 4 x − cos 2 x) −1 =0
2 (2 cos 2 2 x − cos 2 x −1) −1 =0
4 cos 2 2 x − 2 cos 2 x − 3 =0
2 ± 4 + 48
cos (2 x) =
8

Solution Manual for Mathematics For JEE (Main & Advanced), Trigonometry Vector
Algebra Probability, Vol 2,
Copyright©2017 Wiley India Pvt. Ltd. All rights reserved
2 ± 2 13 1 ± 13
= =
8 4
1  1 + 13  1 −1  1 − 13 
x= ⋅ cos −1   and cos  
2 
 4  2  4 
Answer: (C)

(ii)
sin[2(π / 3)]
tan ( x) + =
3
cos 2 (π / 3) − sin 2 x
3
tan x + =
3
2[(1 / 4) − sin 2 x]
1 2 
 2 − 2sin x  tan x + 3 = 3(1 − 4sin x)
2
 
x=6

Answer: (D)

(iii)
2sin 4 x ⋅ cos 3x =sin 4 x
sin (4 x)[2 cos (3x) − 1] = 0

sin=
(4 x) 0,= 4 x nπ= , x
4
π π
x = 0, ,
4 2
1 π 
cos3x= = cos  
2 3
π
x=
9
π π π
x = 0, , ,
4 9 2
Answer: (C)

c can be solved if | c |≤ a 2 + b 2 , by
2. Passage: Equations of the form a cos x + b sin x =
substituting a = r cos θ and b = r sin θ or a = r sin θ and b = r cos θ. Based on this,
answer the following questions.
(i) General solution set of the equation sin 8 x − cos 6=
x 3(sin 6 x + cos8 x) is

 π   π 
(A)  nπ + : n ∈  ∪  2kπ + : k ∈ 
 2   6 

 π π   π 
(B)  n + : n ∈  ∪  [4k + 1] : k ∈ 
 7 12   4 

Solution Manual for Mathematics For JEE (Main & Advanced), Trigonometry Vector
Algebra Probability, Vol 2,
Copyright©2017 Wiley India Pvt. Ltd. All rights reserved
 π   π 
(C)  nπ + : n ∈  ∪  kπ + : k ∈ 
 6   2 

 π   π 
(D) (2n + 1) : n ∈   ∪ (2k + 1) : k ∈  
 3   7 

(ii) 0 in the interval (0, π)


The number of solutions of the equation cos 5 x + sin 5 x =
is

(A) 4 (B) 5 (C) 6 (D) 8

(iii) The number of solutions of the equation sin x + cos x =( 3 + 1) / 2 in (0, π / 2) is

(A) 4 (B) 3 (C) 2 (D) 1

Solution:

(i)
sin 8 x − cos 6=
x 3(sin 6 x + cos8 x)
sin 8 x − 3 cos8 x = 3 sin 6 x + cos t
 π  π
sin  8 x − =
 sin  6 x + 
 3  6
π  π
⇒ 8 x − = nπ + (−1)n ⋅  6 x + 
3  6
Case I let n = 2m (even)
π π
8 x − = 2mπ + 6 x +
3 6
π
2 x 2mπ +
=
2
π π
x = mπ + = (4m + 1)
4 4
Case II let n = 2m + 1 (odd)
π π
8 x − = (2m + 1)π − 6 x −
3 6
π π
x 2mπ + π +
14 = −
3 6

14 x 2mπ +
=
6
mπ π
x = + ,m∈ z
7 12
Answer: (B)
(ii)
cos 5 x + sin 5 x =
0 ⇒ tan 5 x =
−1
π
x nπ −
5=
4
nπ π
x = − , n∈ z
5 20

Solution Manual for Mathematics For JEE (Main & Advanced), Trigonometry Vector
Algebra Probability, Vol 2,
Copyright©2017 Wiley India Pvt. Ltd. All rights reserved
Since
x ∈ (0, π )
Therefore,
3π 7π 11π 15π 19π
x= , , , ,
20 20 20 20 20
Answer: (B)

(iii)
3 +1  π π
sin x + cos =
x ⇒ cos  x − = cos
2  4  12
π π
⇒ x − = 2nπ ±
4 12
π π
2nπ +
x= , 2nπ +
3 6
Since
 π
x ∈  0, 
 2
Therefore,
π π
x= ,
6 3
Answer: (C)

ASSERTION–REASONING TYPE QUESTIONS


In the following set of questions, a Statement I is given and a corresponding Statement II is given
just below it. Mark the correct answer as:

(A) Both Statements I and II are true and Statement II is a correct explanation Statement I.
(B) Both Statements I and II are true but Statement II is not a correct explanation for Statement
I.
(C) Statement I is true and Statement II is false.
(D) Statement I is false and Statement II is true.
1. Statement I: cos x + sin x = 2 has solutions.

c has solutions if | c |≤ a 2 + b 2 .
Statement II: a cos x + b sin x =

Solution:
(sin x + cos x) ≤ 2
Hence Statement I is false.
Statement II is true.
Answer: (D)

2. Statement I: sin x + sin 2 x + sin 3x =cos x + cos 2 x + cos 3x has solutions.

Statement II: The sets (sin x, sin 2 x, sin 3x) and (cos x, cos 2 x, cos 3x) are equal for all
real values of x.

Solution:
Statement I sin (2 x=
) (2 cos x + 1) cos (2 x) (2 cos x + 1)

Solution Manual for Mathematics For JEE (Main & Advanced), Trigonometry Vector
Algebra Probability, Vol 2,
Copyright©2017 Wiley India Pvt. Ltd. All rights reserved
−1
=tan 2 x 1 =
and cos x
2
π 2π
=x =, x
8 3
Statement II
sin 2 x = cos 2 x
π
For x = only
2
Answer: (C)

3. Statement I: cos x= 1 / 2 ⇒ x= 2nπ ± (π / 3), n ∈  .

Statement II: sin x= 1 / 2 ⇒ x= kπ + (−1)k (π / 6), k ∈  .

Solution:

Statement I is true.
Statement II is true but not the explanation of I.
Answer: (B)

4. Statement I: tan x =1 ⇔ x =nπ + (π / 4), n ∈  .

tan 3x − tan 2 x
Statement II: = tan x for all x ∈ .
1 + tan 3x tan 2 x

Solution:

π
=
For x =, tan 2 x undefined.
4
Answer: (C)

5. Statement I: If y > 0 and 0 ≤ x ≤ π / 2 are such that sin x + cos x = y + 1 / y , then


x = π / 4 and y = 1.

Statement II: | sin x + cos x |≤ 2 and y + (1 / y ) ≥ 2 .

Solution:
max[sin ( x) + cos ( x)]= 2
π
For x =
4
y +1
=2 ⇒
= y 1
y
Answer: (A)

6. Statement I: For 0 ≤ x, y ≤ 2π , sin x + sin y = 2 ⇒ x + y = π .

Statement II: | sin θ |≤ 1 for all θ.

Solution Manual for Mathematics For JEE (Main & Advanced), Trigonometry Vector
Algebra Probability, Vol 2,
Copyright©2017 Wiley India Pvt. Ltd. All rights reserved
Solution:
sin x = 1= sin y
π π
=
x ,=
y , x +=y π
2 2

Answer: (A)

INTEGER ANSWER TYPE QUESTIONS


The answer to each of the questions in this section is a non-negative integer. The appropriate
bubbles below the respective question numbers have to be darkened. For example, as shown in the
figure, if the correct answer to the question number Y is 246, then the bubbles under Y labeled as
2, 4, 6 are to be darkened.

1
X Y Z W
2
ⓞ ⓞ ⓞ ⓞ
① ① ① ①
② ② ② ②
③ ③ ③ ③
④ ④ ④ ④
⑤ ⑤ ⑤ ⑤
⑥ ⑥ ⑥ ⑥
⑦ ⑦ ⑦ ⑦
⑧ ⑧ ⑧ ⑧
⑨ ⑨ ⑨ ⑨

<COMP: Shade ② , ④ , and ⑥ under column Y.>

1. If cos 2 x + a sin x =
2a − 7 has real solutions, then the sum of the minimum and
maximum values of a is _____.

Solution:
1 − 2sin 2 x + 9sin ( x) =
2a − 7
2sin 2 ( x) − a sin ( x) + 2a − 8 =0
a ± a 2 −16a + 64 a ± (a − 8)
sin ( x) = =
4 4
−4 ≤ 2a − 8 ≤ 4
2≤a≤6
Sum of minimum and maximum of a = 2 + 6 = 8
Answer: (8)

2. The number of values of x in the interval (0, 5π) satisfying the equation
3sin 2 x − 7sin x + 2 =0 is_____.

Solution:

Solution Manual for Mathematics For JEE (Main & Advanced), Trigonometry Vector
Algebra Probability, Vol 2,
Copyright©2017 Wiley India Pvt. Ltd. All rights reserved
3sin 2 x − 6sin x − sin ( x) + 2 = 0
[3sin ( x) −1][sin ( x) − 2] = 0
1
sin ( x)= = 0.33
3
x= nπ + (−1) sin −1 (0.33)
n

Answer: (6)

3. The number of solutions of the equation sin 4 x − 2sin 2 x − 1 =0 in the interval (0, 2π)
is_____.

Solution: Let
sin 2 ( x) = t sin 2 ( x) = 1 + 2
t 2 − 2t − 1 =0 but sin ( x) ≤1
(t − 1) =
2
2 hence answer is
(t − 1) =
± 2 no solution.
Answer: (0)

4. Let f=
( x) 2sin x(cos x + cos 3x + cos 5 x + cos 7 x + cos 9 x) . Then the number of values of
x satisfying the equation f (x) = 1 in the interval (0, π) is_____.

Solution:
sin 2x + sin 4x − sin 2x + sin 6x − sin 4x + sin 8x − sin 6x + sin10 x − sin 8 x =1
π 
sin10 x = 1= sin  
2
π
=
10 x (4n + 1)
2
π
=x (4n + 1)
20
π 5π 9π 13π 17π
x= , , , ,
20 20 20 20 20
Answer: (5)

5. The number of values of x satisfying the equation


x 2 + (1 / x 2 ), x ≤ π / 9 is_____.
2[cos 2 ( x / 2)]sin 2 x =

Solution:
1
[1 + cos ( x)] ⋅ sin 2 x =+
x2
x2
2
 1
[1 + cos ( x)] ⋅ sin 2 x = x− x  +2
 
As right side ≥ 2
But left side ≤ 2
Hence x = 0
Answer: (0)

Solution Manual for Mathematics For JEE (Main & Advanced), Trigonometry Vector
Algebra Probability, Vol 2,
Copyright©2017 Wiley India Pvt. Ltd. All rights reserved
6. The number of solutions of the equation esin x − e − sin x =
4 is _____.

Solution: Let
esin ( x ) = y
y 2 −1 =4 y
y2 − 4 y + 4 =
5
( y − 2) =5
y= 2 + 5 , esin ( x ) =
2+ 5
This is not possible for any x.
Answer: (0)

7. The number of values of x satisfying the equation 2cos 2 x + 1 = 3 ⋅ 2− sin


2
x
in the interval [0,
2π] is _____.

Solution: Let
2− sin
2
x
=z
2 ⋅ ( z )2 + 1 =3z
2 z 2 − 3z + 1 =0
(2 z −1) ( z −1) = 0
1
z =1 and
2
2− sin ( x ) = 20 or 2−1
2

sin ( x) = 0 , sin ( x) = ± 1
π 3π
x = 0, π , 2π , ,
2 2
Answer: (5)

8. The number of solutions of the equation sin x + 2sin 2 x = 3 + sin 3x, 0 < x < π is _____.

Solution:
sin ( x) ≤1 and 2sin 2 x ≤ 2
sin ( x) + 2sin (2 x) ≤ 3
so, sin ( x) + 2sin 2 x =3 + sin 3x
is not valid for any x
Answer: (0)

9. Let 0 < x, y < π. Then the number of ordered pairs (x, y) satisfying the system of equations
x − 3sec2 y
= 1 and 22cosec x +
2
5cosec 3|sec y|
= 64 is _____.

Solution:
cos ec2 x = 3sec2 y
π 5π
cos ec (x) = ± 3 sec ( y ) x= ,
6 6

Solution Manual for Mathematics For JEE (Main & Advanced), Trigonometry Vector
Algebra Probability, Vol 2,
Copyright©2017 Wiley India Pvt. Ltd. All rights reserved
π 2π
2 cos ec ( x) + 3 sec ( y ) =
6 y= ,
3 3
2[± 3 sec( y )] + 3 sec ( y ) =
6
2 −6
=
sec ( y ) = or, sec y
3 3
π 3
y= cos y = −
3 6
Answer: (4)
<H1>Answers

<H2>Single Correct Choice Type Questions

1. (D) 17. (D)


2. (C) 18. (A)
3. (D) 19. (A)
4. (B) 20. (B)
5. (A) 21. (C)
6. (B) 22. (A)
7. (D) 23. (C)
8. (A) 24. (D)
9. (B) 25. (C)
10. (A) 26. (B)
11. (B) 27. (A)
12. (C) 28. (D)
13. (B) 29. (B)
14. (D) 30. (A)
15. (D) 31. (D)
16. (B)

<H2>Multiple Correct Choice Type Questions

1. (B), (C) 9. (B), (D)


2. (A), (B) 10. (A), (B), (C)
3. (A), (B), (D) 11. (A), (B), (C), (D)
4. (A), (B), (C), (D) 12. (C), (D)
5. (A), (B), (C) 13. (A), (D)
6. (A), (B), (C), (D) 14. (A), (B)
7. (A), (C), (D) 15. (A), (C)
8. (A), (B), (C), (D)
<H2>Matrix-Match Type Questions

1. (A) → (p), (s), (t); (B) → (q), (s); 4. (A) → (q); (B) → (s); (C) → (p);
(C) → (q), (r), (s); (D) → (r), (s) (D) → (p)
2. (A) → (t); (B) → (p), (q); (C) → 5. (A) → (q); (B) → (p); (C) → (q);
(p), (r); (D) → (p), (t) (D) → (r)
3. (A) → (p); (B) → (q); (C) → (s);
(D) → (r)

Solution Manual for Mathematics For JEE (Main & Advanced), Trigonometry Vector
Algebra Probability, Vol 2,
Copyright©2017 Wiley India Pvt. Ltd. All rights reserved
<H2> Comprehension Type Questions

1. (i) (A); (ii) (D); (iii) (C) 2. (i) (B); (ii) (B); (iii) (C)
<H2>Assertion–Reasoning Type Questions

1. (D) 4. (C)
2. (C) 5. (A)
3. (B) 6. (A)

<H2>Integer Answer Type Questions

1. 8 6. 0
2. 6 7. 5
3. 0 8. 0
4. 5 9. 4
5. 0

Solution Manual for Mathematics For JEE (Main & Advanced), Trigonometry Vector
Algebra Probability, Vol 2,
Copyright©2017 Wiley India Pvt. Ltd. All rights reserved
Solutions to Exercises
Chapter 4

SINGLE CORRECT CHOICE TYPE QUESTIONS

1. In ∆ABC , a =
6, B =
45° and C= 30°. AD is drawn perpendicular to the side BC. A
circle is described on AD as diameter cuts the sides AB and AC in P and Q, respectively.
Then the length of the segment PQ is

3 3 3 3 3
(A) (B) (C) (D) 2 3
2 4 2

Solution:

x
AD = BD = X, tan (30°) =
6− x
 6 
x =  
 3 +1 
Here, x = diameter of circle passes through APQ = 2R. Here ∠PAQ =105°, then

PQ
= sin (105°)
2R

6 ( 3 + 1) 3
PQ= 2 R ⋅ sin (105°)= ⋅ =
( 3 + 1) 2 2 2
Answer: (C)

2. In ∆ABC, the value of

A B C
(b + c) tan + (c + a) tan + (a + b) tan
2 2 2

is

(A) 2( R + r ) (B) 4( R + r )

Solution Manual for Mathematics For JEE (Main & Advanced), Trigonometry Vector
Algebra Probability, Vol 2,
Copyright©2017 Wiley India Pvt. Ltd. All rights reserved
(C) 3( R + r ) (D) 6( R + r )

Solution:
A B C
2 R (sin B + sin C ) tan + 2 R (sin C + sin A) ⋅ tan + 2 R (sin A + sin B ) tan
2 2 2
   B − C   B + C  C−A C+A A− B A + B 
4 R  cos   ⋅ cos    + cos   ⋅ cos   + cos ⋅ cos 
   2   2   2   2  2 2 
 B C A A B C
4 R  cos 2 + cos 2 + cos 2 − sin 2 − sin 2 − sin 2 
 2 2 2 2 2 2
= 4 R + 4r
Answer: (B)

3. In ∆ABC , a =BC , b =CA, c =AB. If a 2 , b 2 , c 2 are in AP (in the given order), then

sin 3B
=
sin B

2 2
 a2 − c2   a2 + c2 
(A)   (B)  
 2ac   2ac 

2 2
 a2 − c2   a2 + c2 
(C)   (D)  
 ac   ac 

Solution:

sin 3B 3sin B − 4sin 3 B


= = 3 − 4sin 2 B
sin B sin B

Now,
2
 a 2 − c 2  a 4 + c 4 − 2a 2 c 2
  =
 2ac  4a 2 c 2
( a 2 + c 2 ) 2 − 4a 2 c 2
=
4a 2 c 2
(4b 4 )
= 2 2 −1 = 3 − 4b 2 = 3 − 4sin 2 B
4a c
Answer: (A)

4. In ∆ABC, the median AD is perpendicular to the side AC at A. Then

 c2 − a2 
cos A cos C = k  
 ca 

where the value of k is

Solution Manual for Mathematics For JEE (Main & Advanced), Trigonometry Vector
Algebra Probability, Vol 2,
Copyright©2017 Wiley India Pvt. Ltd. All rights reserved
3 3 4 2
(A) (B) (C) (D)
2 4 3 3

<DE: Please check solution missing.>

5. In ∆ABC, if BC = 5, CA = 4 and cos( A − B) =


31/ 32, then the area of the triangle is

15 15 15
(A) 5 7 (B) 3 (C) 7 (D) 7
4 3 4

Solution:

1 − tan 2 ( A − B / 2) 31
=
cos ( A − B) =
1 + tan 2 ( A − B / 2) 32

 A− B  1
tan 2  =
 2  63
 A− B   a −b   c  1
=
tan   =  cot  
 2  a+b 2 63
1 c 1
= cot = 2
9   63
c 63
tan   =
2 9
1 − tan 2 (c / 2) 18 1
cos(=
c) = =
1 + tan 2 (c / 2) 144 8
63
sin (c) =
8
1
Area of triangle A = ab sin C
2
1 63 15 7
= 5 ∗4∗ =
2 8 4
Answer: (D)

6. In ∆ABC, if a = 6, b = 3 and cos( A − B) =


4 / 5, then the third side c is equal to

(A) 3 5 (B) 6 (C) 2 5 (D) 6 5

Solution: Since
A − B 1 − cos( A − B)
tan 2 =
2 1 + cos( A − B)
1 − (4 / 5) 1
= =
1 + (4 / 5) 9
Therefore,
A− B 1
tan =
2 3
Now,
Solution Manual for Mathematics For JEE (Main & Advanced), Trigonometry Vector
Algebra Probability, Vol 2,
Copyright©2017 Wiley India Pvt. Ltd. All rights reserved
A− B a−b C
tan = cot
2 a+b 2
1 3 C
= ⋅ cot
3 9 2
C
⇒ cot =1 ⇒ C =90°
2
Therefore,
C 2 = a 2 + b 2 = 45
C =3 5
Answer: (A)

7. In ∆ABC, if cos A = λ cos B, then

 A+ B
tan 2  =
 2 

 a + b 1 + λ   a + b 1 − λ 
(B) 
 a − b   1 + λ 
(A) 
 a − b   1 − λ 

 a − b 1 + λ   a − b  1 − λ 
(C) 
 a + b   1 − λ 
(D)   
 a + b  1 + λ 

Solution: Since
cos A = λ cos B
cos B 1
=
cos A λ
cos B − cos A 1 − λ
⇒ =
cos B + cos A 1 + λ
2sin[( A + B ) / 2]sin[( A − B ) / 2] 1 − λ
=
2 cos[(A + B) / 2]cos[(A − B) / 2] 1 + λ
A+ B A − B 1− λ
= cot
tan ⋅
2 2 1+ λ
A + B tan[( A + B) / 2] 1 − λ
= tan 2 ⋅
2 tan[( A − B) / 2] 1 + λ
sin A + sin B 1 − λ
= ⋅
sin A − sin B 1 + λ
a + b 1− λ
= ⋅
a − b 1+ λ
Answer: (B)

8. In ∆ABC, it is given that

3
cos A + cos B + cos C =
2

If one side of the triangle is of length 4 units, the area of its pedal triangle is

Solution Manual for Mathematics For JEE (Main & Advanced), Trigonometry Vector
Algebra Probability, Vol 2,
Copyright©2017 Wiley India Pvt. Ltd. All rights reserved
3 3
(A) (B) (C) 2 3 (D) 3
4 2

Hint: See Theorem 4.23.

Solution:

Since
3
cos A + cos B + cos C ≤
2
If
3
cos A + cos B + cos C =
2
then
π
A= B= C=
3
3
Area of ∆ABC = (side) 2
4
3
= ⋅ 16 = 4 3
4
1
Since triangle is equilateral. Therefore, area of pedal triangle = (area of ∆ ABC) = 3
4
Answer: (D)

9. In ∆ABC , a = 5, b =4, c =3 and its centroid and circumcentre are, respectively, G and
O. Then the magnitude of OG is

Solution Manual for Mathematics For JEE (Main & Advanced), Trigonometry Vector
Algebra Probability, Vol 2,
Copyright©2017 Wiley India Pvt. Ltd. All rights reserved
5 6 2 3
(A) (B) (C) (D)
6 5 3 2

1
Hint: OG 2 = R 2 − (a 2 + b 2 + c 2 )
9

Solution:
abc −a 2 + b 2 + c 2 −(5) 2 + (4) 2 + (3)
=R = cos ( A) =
4∆ 2cb 24
A= 90°= 0
Now,
1
a  25 50  2
=
2R = 5 OG
=  − 
sin( A)  4 9 
2
5  50  5
R= = =
2  72  6

Answer: (A)

10. In ∆ABC, if

cos A cos B + sin A sin B sin C =


1

then

sin 2 A + sin 2 B + sin 2 C =

3 3
(A) 2 2 (B) (C) 2 (D)
2 2

Solution:
cos A ⋅ cos B + sin A ⋅ sin B + sin A ⋅ sin B ⋅ sin C − sin A ⋅ sin B =
1
cos( A − B) + sin A ⋅ sin B [sin (C ) −1] = 1
sin A ⋅ sin B[sin (C ) −1] =1 − cos( A − B)
Here, L.H.S. ≤ 0, R.H.S. ≥ 0
so,
sin(C ) − 1 = 0

π
c=
2

cos( A − B) = 1 =0
A − B= 0, A= B
1 1
sin 2 A + sin 2 B + sin 2 C = + + 1 = 2
2 2
Answer: (C)

11. In ∆ABC, if

Solution Manual for Mathematics For JEE (Main & Advanced), Trigonometry Vector
Algebra Probability, Vol 2,
Copyright©2017 Wiley India Pvt. Ltd. All rights reserved
a cos A + b cos B + c cos C a + b + c
=
a sin B + b sin C + c sin A 9R

and I1 , I 2 and I 3 are the excentres of ∆ABC, then the area of ∆I1 I 2 I 3 is

(A) 3R 2 (B) 3 3R 2

 3 3 2
(C) 3R 2 (D)  R
 2 

Hint: Area of ∆I1 I 2 I 3 is 2Rs.

Solution:
(sin 2 A + sin 2 B + sin 2C ) 2
= (sin A + sin B + sin C )
2Σ sin A ⋅ sin B 9
4sin A ⋅ sin B ⋅ sin C 2
= (sin A + sin B + sin C )
2Σ sin A ⋅ sin B 9
9sin A ⋅ sin B ⋅ sin C =(sin A + sin B + sin C ) =
(Σ sin A ⋅ sin B)
9 = (sin A + sin B + sin C ) (cosec A + cosec B + cosec)
= 9 3 sin A ⋅ sin B ⋅ sin C ⋅ 3 cosec A ⋅ cosec B ⋅ cosec C
3
=
This is possible for sin =
A sin =
B sin C
2
2S = a + b + c = 3a = 3 ⋅ 2 R sin 60°
Area of =
triangle (3=
3R) R 3 3R 2
Answer: (B)

12. In a circle of 1 unit radius, AB is a chord whose length is also 1. If C is any point on the
major arc of AB, then the maximum value of ( AC )2 + ( BC ) 2 is

(A) 2 + 3 (B) 2(2 + 3)

(C) 2( 3 + 1) (D) 4( 3 + 1)

Hint: BCA= 30° and let BAC = θ .

Solution:

Solution Manual for Mathematics For JEE (Main & Advanced), Trigonometry Vector
Algebra Probability, Vol 2,
Copyright©2017 Wiley India Pvt. Ltd. All rights reserved
Here
1
 2  1 2  2 3
OD =
(1) −    =
  2   2

Hence, ∠AOB
= 180° − 2∠OAB
= 180° − 2(60°)= 60°
Hence, ∠ACB = 30°
then,
3  2+ 3 
CD = 1+ =  
2  2 
 2 
2+ 3  1  8+ 4 3 
( AC )2 + ( BC )2 = 2    + = 2 = 2(2 + 3)
  2  4   4 
   

Answer: (B)

13. In ∆ABC, if a = 2, b = 3 and sec2 A = 8 / 5, then one of the values of the third side C is

(A) 10 (B) 6 (C) 5 (D) 6

Solution:

5 b2 + c2 − a 2 9 + C 2 − 4
cos=
( A) = =
2 2 2bc 6C
⇒ 6 10C =
20 + 4c 2
⇒ 4c 2 − 6 10C + 20 =
0
6 10 ± 360 − 320
=⇒C = 10
8

Answer: (A)

14. In ∆ABC, if p1 , p2 and p3 are the lengths of the altitudes, then 2∆ 2 is equal to

Solution Manual for Mathematics For JEE (Main & Advanced), Trigonometry Vector
Algebra Probability, Vol 2,
Copyright©2017 Wiley India Pvt. Ltd. All rights reserved
(A) Rp1 p2 p3 (B) R 2 p1 p2 p3

3 2
(C) 3R 2 p1 p2 p3 (D) R p1 p2 p3 .
2

where R is the circumradius

Solution:

Let ΔABC is an equilateral triangle

3a
p1 =
2

Now,
3 3a3
p1 ⋅ p2 ⋅ p3 =
8
3  3 
3 3a 9
R ⋅ p1 p2 p3 =  
8  4∆ 
= 2∆ 2
Answer: (A)

15. In ∆ABC, if cot A + cot B + cot C =


3, then the sides are in the ratio

(A) 1:2:3 (B) 1:1: 2

(C) 1: 2 :1 (D) 1:1:1

Solution:
cot A + cot B + cot C =
3
 1  1 1 1
cot A + cot B + cot C = 3  = + +
 3 3 3 3
1 π 
cot=A cot= B cot= C = cot  
3 3
π
A= B= C=
3
sin A : sin B : sin C = 1:1:1
Answer: (D)

Solution Manual for Mathematics For JEE (Main & Advanced), Trigonometry Vector
Algebra Probability, Vol 2,
Copyright©2017 Wiley India Pvt. Ltd. All rights reserved
16. In ∆ABC, if the distances of the vertices from the incentre are x, y and z, and r is its
inradius, then r 2 is equal to

x+ y+ z 1 1 1
(A) (B) + +
xyz x y z

xyz x2 y 2 z 2
(C) (D)
x+ y+ z x+ y+z

Solution:

B
r = y sin
2
Similarly
A C
=r x=
sin z sin
2 2
Since

A B C
=
r 4 R sin ⋅ sin ⋅ sin
2 2 2
r r r
r = 4R ⋅ ⋅ ⋅
x y z
xyz
r2 =
4R
Answer: (C)

17. In ∆ABC, if the altitude, the angle bisector and the median through A, divide angle A into
four equal parts, then angle A is equal to

(A) 120° (B) 90° (C) 115° (D) 75°

Solution:

In ∆ABC, AL, AP and AD are perpendicular, angle bisector and median through A. Since
A
∠BAL = ∠LAP = ∠PAD = ∠DAC =
4

Solution Manual for Mathematics For JEE (Main & Advanced), Trigonometry Vector
Algebra Probability, Vol 2,
Copyright©2017 Wiley India Pvt. Ltd. All rights reserved
Therefore,
∆ABL
A 
+ B = 90° 
4

and in ∆ACL  (1)
3A 
+ C = 90°
4 
Now in ∆ ABD
AD BD
= (2)
sin B sin(3 A / 4)
and in ∆ ACD
AD CD
= (3)
sin C sin( A / 4)
From Eqs. (2) and (3)
BD sin B CD sin C
=
sin(3 A / 4) sin( A / 4)
A 3A
= sin C ⋅ sin
sin B sin
4 4
Using Eq. (1), we get
A A 3A 3A
2sin ⋅ cos = 2sin ⋅ cos
4 4 4 4
A 3A
sin = sin
2 2
A 3A
⇒ =π− ⇒ A = 90°
2 2
Answer: (B)

18. In ∆ABC , r1 + r2 + r3 =r + kR where k is

(A) 2 (B) 4 (C) 6 (D) 8

Solution:

∆ ∆ ∆ ∆
=r1 = , r2 = , r3 = ,r
( s − a) s −b s −c 5

∆[ s ( s − b)( s − c) + s ( s − a)( s − c) + s ( s − a)( s − b) + ( s − a)( s − b)( s − c)]


r1 + r2 + r3 − r =
s ( s − a)( s − b)( s − c)
∆[3s 3 − s 2 (2)(a + b + c) + s (ab + bc + ac) − s 3 + s 2 (a + b + c) − s (ab + bc + ac) + abc]
=
∆2

∆⋅ abc abc
= = = AR
∆2 ∆
Answer: (B)

19. If a, b and γ are the distances of the vertices from its orthocentre, then

Solution Manual for Mathematics For JEE (Main & Advanced), Trigonometry Vector
Algebra Probability, Vol 2,
Copyright©2017 Wiley India Pvt. Ltd. All rights reserved
a b c
+ + =
a b γ

a+b+c abc
(A) (B)
abγ a +b +γ

abc a+b+c
(C) (D)
abγ a +b +γ

Solution:

a b c a bγ + baγ + cab abc


=+ + =
a b γ abγ abγ

Answer: (C)

20. In ∆ABC, if p1 , p2 and p3 are the lengths of the altitudes, then ∆( p1−2 + p2−2 + p3−2 ) =

(A) cos A + cos B + cos C

(B) sin A + sin B + sin C

(C) tan A + tan B + tan C

(D) cot A + cot B + cot C

Solution: Since
1 1 1
=∆ =
ap1 =
bp2 cp3
2 2 2
Therefore,
2∆ 2∆ 2∆
=p1 = , p2 = , p3
a b c
Now,
 1 1 1 
∆ ⋅ 2 + 2 + 2 
p 
 1 p2 p3 
 a 2 + b2 + c2 
∆ ⋅  
 4∆ 2 
a 2 + b2 + c2
=
2∆
(a 2 + b 2 − c 2 ) (b 2 + c 2 − a 2 ) (c 2 + a 2 −
= + +
2∆ 2∆ 2∆

Put 2∆ = ab sin C = bc sin A = ac sin B


= cot A + cot B + cot C
Answer: (D)

Solution Manual for Mathematics For JEE (Main & Advanced), Trigonometry Vector
Algebra Probability, Vol 2,
Copyright©2017 Wiley India Pvt. Ltd. All rights reserved
21. If Q is a point on the altitude AD and is lying inside the triangle such that CBQ = B / 3,
then the length AQ is

B C
(A) 2c sin (B) 2c sin
3 3

B B
(C) 2a sin (D) 2b sin
3 3

Solution:

From ΔABQ,

 B
∠AQB
=  90° + 
 3
2B
∠ABQ =
3

By sine rule in ΔABQ

AQ AB C
= =
sin (2 B / 3) sin (90 + B / 3) cos ( B / 3)

2C ⋅ sin B / 3 ⋅ cos B / 3 sin B


=AQ = 2C
cos B / 3 3
Answer: (A)

22. In ∆ABC, let ma , mb and mc be the lengths of the medians through the vertices A, B and
C, respectively. Then angle A is obtuse if and only if

mb2 + mc2 mb2 + mc2


(A) ma2 < (B) ma2 <
2 3

mb2 + mc2 mb2 + mc2


(C) ma2 < (D) ma2 <
4 5

Hint: Use 4ma2 = 2b 2 + 2c 2 − a 2 , etc.

Solution: Given that 4ma2 = 2b 2 + 2c 2 − a 2 , so,


4mb2 + 4mc2 = 4a 2 + b 2 + c 2

Solution Manual for Mathematics For JEE (Main & Advanced), Trigonometry Vector
Algebra Probability, Vol 2,
Copyright©2017 Wiley India Pvt. Ltd. All rights reserved
Now,
4mb2 + 4mc2 4a 2 + b 2 + c 2
=
5 5

4a 2 + b 2 + c 2
2b 2 + 2c 2 − a 2 <
5

this gives b2 +c2 < a2


Answer: (D)

23. In ∆ABC, if b 2 + c 2 =
(2011)a 2 , then

cot A
=
cot B + cot C

2011 2009
(A) (B) 1005 (C) 1006 (D)
2 2

Solution:
cot A cos A ⋅ sin A ⋅ sin C
=
cot B + cot C sin 2 A
b 2 + c 2 − a 2 bc
⋅ 2
2 bc 4R b2 + c2 − a 2
=
a2 2a 2
4R2
2010a 2
= = 1005
2a 2
Answer: (B)

24. In ∆ABC, if a 2 + b 2 + c 2 = ac + bc 3, then the triangle is

(A) Right angled (B) Isosceles

(C) Equilateral (D) Obtuse angled

Solution:

a 2 + b 2 + c 2 = ac + bc 3

a 2 + b 2 + c 2 − 9c =
bc 3
b 2 + (a + c)2 + ac =
bc 3
a2 + c2 =
b2

Answer: (A)

25. In ∆ABC, if (r2 − r1 )(r3 − r1 ) =


2r2 r3 , then the triangle is

Solution Manual for Mathematics For JEE (Main & Advanced), Trigonometry Vector
Algebra Probability, Vol 2,
Copyright©2017 Wiley India Pvt. Ltd. All rights reserved
(A) Equilateral (B) Isosceles

(C) Right angled (D) Obtuse angled

Solution:
 ∆ ∆   ∆ ∆  2∆ ∆
 − − − = ⋅
 s − b s − a   s − c s − a  s − b ( s − c)
r12 = r1r2 + r1r3 + r2 r3
∆2  1 1 1 
=  + + 
 ( s − a )( s − b) ( s − c)( s − a ) ( s − b)( s − c) 
= s ( s − c ) + s ( s − b) + s ( s − a )
∆2
= = 3s 2 − s (2s ) = s2
( s − a)2
=∆ s( s − a)
s ( s − a) ( s − b)( s − c) = s ( s − a)
s ( s − a)[( s − b)( s − c) −1] = 0
a+b+c≠0
a+c−a= 0
b+c=a

Answer: (C)

MULTIPLE CORRECT CHOICE TYPE QUESTIONS

1. In ∆ABC, if a 2 + b 2 + c 2 = bc + (ac) 3, then

= 60°
(A) A (B) B= 90°

2ab
(C) C= 90° (D) Inradius is
c(3 + 3)

Solution:

a 2 + b 2 + c 2 = bc + (ac) 3
a2 c2 c a c
2
+1+ 2
= + ⋅ 3
b b b b b

tan 2 A + 1 + sec2=
A sec A + tan A ⋅ sec A ⋅ 3
2sec2 A= sec A(1 + tan A ⋅ 3)

Solution Manual for Mathematics For JEE (Main & Advanced), Trigonometry Vector
Algebra Probability, Vol 2,
Copyright©2017 Wiley India Pvt. Ltd. All rights reserved
2  cos A + 3 sin A 
= 
cos A  cos A 
3 1
⋅ sin A + ⋅ cos A =1
2 2
 π π
sin  A +  = sin
 6  2
A= 60° ∠C = 90°
Inradius

∆ 1 abc
(r=) =
S 2 s

ab
=
(a + b + c)
Here,
3
a = c ⋅ sin 60° = C
2
C
b = c ⋅ cos 60° =
2
2ab
r=
C (3 + 3)
Answer: (A), (C), (D)

2. In ∆ABC, if

(a + b) 2 =ab + c 2

and sin A + cos A =


3 / 2 then

(A) =
C 120° (B) A= 15°

(C) B= 45° (D) a : b=


: c ( 3 − 1) : 2 : 6

Solution:
a 2 + b2 − c2 =
+ ab − 2ab =
− ab

a 2 + b2 − c2 1
=− =cos (C )
2ab 2
=
C 120°

3
sin A + cos A =
2

 1 1  3
2 sin A + cos A  =
 2 2  2

Solution Manual for Mathematics For JEE (Main & Advanced), Trigonometry Vector
Algebra Probability, Vol 2,
Copyright©2017 Wiley India Pvt. Ltd. All rights reserved
 π 3 π 
sin  A +  = = sin  
 4  2 3
π π
A = − =15°
3 4
∠B =180 −120 −15=
° 45°

a : b : c = sin A : sin B : sin C

=sin15° : sin 45° : sin120°


= ( 3 −1):2: 6
Answer: (A), (B), (C), (D)

3. In any ∆ABC, which of the following statements are true?

A B C 1
(A) sin sin sin ≤
2 2 2 8

A B C 3
(B) sin 2 + sin 2 + sin 2 ≥
2 2 2 4

A B C 9
(C) cos 2 + cos 2 + cos 2 ≤
2 2 2 4

A B C 3 3
(D) cos cos cos ≤
2 2 2 8

Solution:

A B C 1
2sin ⋅ sin ⋅ sin ≤
2 2 2 4

  A− B  C C 1
cos   − sin  sin ≤
  2  2  2 4
 A+ B   A− B  C 1
2cos   ⋅ cos   − 2sin 2 ≤
 2   2  2 2
 A B C 1
2  cos 2 − sin 2  − 2sin 2 ≤
 2 2 2 2
A B C 3
sin 2 + sin 2 + sin 2 ≥
2 2 2 4
A B C 3
and 1 − cos 2 + 1 − cos 2 + 1 − cos 2 ≥
2 2 2 4
A B C 9
cos 2 + cos 2 + cos 2 ≤
2 2 2 4
Now,
A+ B +C π
=
2 2

Solution Manual for Mathematics For JEE (Main & Advanced), Trigonometry Vector
Algebra Probability, Vol 2,
Copyright©2017 Wiley India Pvt. Ltd. All rights reserved
 A+ B  C C  A+ B 
sin   ⋅ cos + sin ⋅ cos  =1
 2  2 2  2 

A B C B A C A B C A B C
sin ⋅ cos ⋅ cos + sin ⋅ cos ⋅ cos + cos ⋅ cos + sin − sin ⋅ sin ⋅ sin
2 2 2 2 2 2 2 2 2 2 2 2

A B C A B C B A A B C
sin ⋅ sin + sin = sin ⋅ cos ⋅ cos + sin ⋅ cos + cos ⋅ cos ⋅ sin −1
2 2 2 2 2 2 2 2 2 2 2
A B C 1
sin ⋅ sin ⋅ sin ≤
2 2 2 8
A B C 3 3
and cos ⋅ cos ⋅ cos ≤
2 2 2 8

Answer: (A), (B), (C), (D)

4. Let ABC be a triangle. Then


A a
(A) sin ≤
2 b+c
(B) If sin A + sin B + sin C ≤ 1, then min( B + C , C + A, A + B) < 30°
A B B C C A
(C) tan tan + tan tan + tan tan =1
2 2 2 2 2 2
A B C
(D) tan 2 + tan 2 + tan 2 ≥ 1
2 2 2

Solution:
A sin A
(A) sin ≤
2 sin B + sin C
A sin A / 2 ⋅ cos A / 2
sin ≤
2 sin( B + C / 2) ⋅ cos( B − C / 2)
 B −C 
cos   ≤1 (True)
 2 
(B)

 A+ B   A− B  C
2sin   ⋅ cos   + sin ≤1
 2   2  2
 A+ B    A− B   A + B  1
sin   cos   + cos   ≤
 2   2   2  2
 A+ B  A B  1
sin   ⋅ cos   ⋅ cos   ≤
 2   2  2 4
 A+ B   B+C   A+C  1
sin   ⋅ sin   ⋅ sin  ≤
 2   2   2  4
min( B + C , C + A, A + B) ≤ 30°

 A+ B  π C 
(C) tan  =  tan  − 
 2  2 2

Solution Manual for Mathematics For JEE (Main & Advanced), Trigonometry Vector
Algebra Probability, Vol 2,
Copyright©2017 Wiley India Pvt. Ltd. All rights reserved
tan A / 2 + tan B / 2 1
=
1 − tan A / 2 ⋅ tan B / 2 tan C / 2
A C B C A B
tan ⋅ tan + tan ⋅ tan =1 − tan ⋅ tan
2 2 2 2 2 2
A B B C A C
tan ⋅ tan + tan ⋅ tan + tan ⋅ tan = 1
2 2 2 2 2 2
2
 A B C A B C A B
(D)  tan 2 + tan 2 + tan =  tan 2 + tan 2 + tan 2 + 2Σ tan ⋅ tan
 2 2 2 2 2 2
A
= Σ tan 2 + 2
2

2
A  A B C
Σ tan 2 = tan + tan + tan  − 2
2  2 2 2
A B C
tan 2 + tan 2 + tan 2 ≥ 1
2 2 2

Answer: (A), (B), (C), (D)

5. If the lengths of the tangents drawn from the vertices A, B and C of ∆ABC to its incircle
are in AP, then

4c − 3b
(A) a, b, c are in AP (B) cos A =
2c

(C) r1 , r2 , r3 are in AP (D) r1 , r2 , r3 are in HP

Solution:

(A)
a − x + c − z= 2(b − y )
Now,
x+ z=2y
so,
a+c=2b
b + c − a b 2 + c 2 − (2b − c)2 4bc − 3b 2 4c − 3b
2 2 2
(B) =
cos A = = =
2bc 2bc 2bc 2c

Solution Manual for Mathematics For JEE (Main & Advanced), Trigonometry Vector
Algebra Probability, Vol 2,
Copyright©2017 Wiley India Pvt. Ltd. All rights reserved
2r1r3
(D) r2 =
r1 + r3
∆ 2 ⋅∆ ( s − a) ⋅∆ ( s − c) 2∆
= =
s − b ∆ ( s − a) + ∆ ( s − c) 2 s − a − c

∆ 2∆
=
s −b b
⇒ b = 2( s − b) = a + b + c − 2b

Answer: (A), (B), (D)

6. In ∆ABC, D, E and F are the feet of the altitudes drawn from the vertices A, B and C onto
the opposite sides. Let R1 , R2 and R3 be the circumradii of ∆AEF, ∆BDF and ∆DCE,
respectively. Then
(A) 2 R1 = a cot A
(B) 2 R2 = b cot B
(C) 2 R3 = c cot C
(D) The ratio of the perimeter of ∆ABC to the perimeter of ∆DEF is r:R

Hint: See Theorems 4.22 and 4.23.

2b= a + c
Solution:

AP = R1

R1 cos A ⋅ sin A
AP =
sin A

a cot A
= = AP
2
= a=
cot A 2 R1

Solution Manual for Mathematics For JEE (Main & Advanced), Trigonometry Vector
Algebra Probability, Vol 2,
Copyright©2017 Wiley India Pvt. Ltd. All rights reserved
Since AFHE is a cyclic quadrilatered
and ∠AFH =
∠AEH =
90°
Therefore, AH is the diameter, so,
1
R1 =
AH
2
1 1 a
=2 R cos A =⋅ ⋅ cos A
2 2 sin A
⇒ 2 R1 =
a cot A
=
Similarly, 2 R2 b= cot B, 2 R3 c cot C
Perimeter of ∆DEF
a cos A + b cos B + c cos C
Therefore, option (D) is incorrect

Answer: (A), (B), (C)

7. In ∆ABC, which of the following statements are true?

A B C
(A) IA ⋅ IB ⋅ IC =
(abc) tan tan tan
2 2 2

A B C
(B) AI1 ⋅ BI 2 ⋅ CI 3 =
(r1r2 r3 ) cosec cosec cosec
2 2 2

(C) I I1 ⋅ I I 2 ⋅ I I 3 =
16rR 2

(D) ( I I1 ) 2 + ( I 2 I 3 ) 2 =
16 R 2

Solution:
(A)

In ΔAIF,

Solution Manual for Mathematics For JEE (Main & Advanced), Trigonometry Vector
Algebra Probability, Vol 2,
Copyright©2017 Wiley India Pvt. Ltd. All rights reserved
AI A
= cosec
FI 2
Therefore,
Aπ 
=IA r cos ec  −θ 
22 
Similarly
B
IB = r cos ec
2
C
IC = r cos ec
2
Now,
IC ⋅ IB ⋅ IC
A B C
r cos ec ⋅ r cos ec ⋅ r cos ec
2 2 2
A B C
= abc tan tan tan
2 2 2
Since
a sin( B / 2)sin(C / 2)
r=
cos( A / 2)
b sin( A / 2)sin(C / 2) C sin( A / 2)sin( B / 2)
= =
cos( B / 2) cos(C / 2)
(B)

In ∆API1,
I1 A A
= cosec
I1 P 2
A
⇒ I1 A =
r1cosec
2
Similarly
B
I1 B = r2 cosec
2
C
I1C = r3cosec
2
Now,
I1 A ⋅ I1 B ⋅ I1C
A B C
= r1r2 r3cosec cosec cosec
2 2 2
(C)
I=
I1 I1 A − IA

Solution Manual for Mathematics For JEE (Main & Advanced), Trigonometry Vector
Algebra Probability, Vol 2,
Copyright©2017 Wiley India Pvt. Ltd. All rights reserved
A
= (r1 − r ) cos ec
2
Put
A B C
r1 = 4 R sin cos cos
2 2 2
A B C
r = 4 R sin sin sin
2 2 2
A
r1 − r =4 R sin 2
2
Therefore,
A
I I1 = 4 R sin
2
B
I I 2 = 4 R sin
2
C
I I 3 = 4 R sin
2
Now,
I I ⋅ I I 2 ⋅ I I3 =
16rR 2
(D)
( I I1 ) 2 + ( I 2 I 3 ) 2

+ 16 R 2 cos 2 = 16 R
A A 2
= 16 R 2 sin 2
2 2

Answers: (A), (B), (C), (D)

8. In any ∆ABC

C C
(A) (r1 + r2 ) tan =
c (B) (r3 − r ) cot =
c
2 2

C
(C) (r3 − r ) tan =
c (D) r2 r3 + r3 r1 + r1r2 =
∆2
2

Solution:
(A)
A B C
r=
1 4 R sin ⋅ cos ⋅ cos
2 2 2
A A C
=
r2 4 R cos ⋅ sin ⋅ cos
2 2 2

C   A + B  2 C
=(r1 + r2 ) 4 R cos
= sin  4 R cos
2   2   2

C C C C
4 R ⋅ cos 2 ⋅ tan =2 R ⋅ sin ⋅ cos =C
2 2 2 2
(B)
C
(r3 − r ) cot
2

Solution Manual for Mathematics For JEE (Main & Advanced), Trigonometry Vector
Algebra Probability, Vol 2,
Copyright©2017 Wiley India Pvt. Ltd. All rights reserved
 ∆ ∆ C
 −  ⋅ cot
 s −c s  2
∆⋅ C cos C / 2
⋅ =C
s ( s − c) sin C / 2
(C)
C
(r3 − r ) tan ≠C
2

(D)
∆ ∆ ∆ 2 [( s )]
∑ ( s − b) ⋅ ( s − c ) =
( s − b)( s − c)( s − a )
=
∆2

Answer: (A), (B), (D)

9. Two sides of a triangle are 12 and 12 2 and the angle opposite to the shorter side is 30°.
Then
(A) The number of such triangles is two
(B) The angles of the triangles are 30°, 45°, 105° or 30°, 135°, 15°
3 +1 3 −1
(C) Area of the triangle is or
2 2 2 2
(D) Areas are in the ratio ( 3 + 1) : ( 3 − 1)

Solution:
(A) Let third side is (C). Then,

(12 2) 2 + (12) 2 − C 2
cos 30° =
2(12)(12 2)

3 144(3) − C 2
=
2 288 2
=
144 6 144(3) − C 2
C= 144(3 − 6) = 12 3 − 6
(B)
Angles of triangles : 30°:45°:105°
Angles of triangle : 30°:135°:15°
(C)
1
∆ = ab sin C
2
1
= ⋅ 144 ⋅ 2 ⋅ sin C
2
3 +1
= 72 2
2 2
3
or 72 2 ⋅
2 2
(D)

Solution Manual for Mathematics For JEE (Main & Advanced), Trigonometry Vector
Algebra Probability, Vol 2,
Copyright©2017 Wiley India Pvt. Ltd. All rights reserved
Area of triangles is in ratio =
( 3 + 1):( 3 −1)

Answer: (A), (B), (D)

10. In ∆ABC, let

a b c
=cos a = , cos b = and cos γ
b+c c+a a+b

where each of a, b and γ is positive and less than π. Then

a β γ
(A) tan 2 + tan 2 + tan 2 =1
2 2 2

a β γ
(B) tan tan tan = tan A tan B tan C
2 2 2

a β γ A B C
(C) tan tan tan = tan tan tan
2 2 2 2 2 2

(D) tan a tan β tan γ = tan A tan B tan C

Solution:
1 − cos αβ 1 − cos 1 + cos γ
(A) + +
1 + cos αβ 1 + cos 1 − cos γ
b +c −a c + a −b a +b −c
+ +
a+b+c a+b+c a+b+c
a b γ
= 1= tan 2 + tan 2 + tan 2
2 2 2
a β γ
(C) tan ⋅ tan ⋅ tan
2 2 2
(b + c − a)(c + a − b)(a + b − c) A B C
= = tan ⋅ tan ⋅ tan
(a + b + c) ⋅ (a + b + c)
2
2 2 2

Answer: (A), (C)

MATRIX-MATCH TYPE QUESTIONS


In each of the following questions, statements are given in two columns, which have to be
matched. The statements in column I are labeled as (A), (B), (C) and (D), while those in column II
are labeled as (p), (q), (r), (s) and (t). Any given statement in column I can have correct matching
with one or more statements in column II. The appropriate bubbles corresponding to the answers
to these questions have to be darkened as illustrated in the following example.

Example: If the correct matches are (A) → (p), (s), (B) → (q), (s), (t), (C) → (r), (D) → (r), (t) ,
that is if the matches are (A) → (p) and (s); (B) → (q), (s) and (t); (C) → (r); and (D) → (r), (t),
then the correct darkening of bubbles will look as follows:

Solution Manual for Mathematics For JEE (Main & Advanced), Trigonometry Vector
Algebra Probability, Vol 2,
Copyright©2017 Wiley India Pvt. Ltd. All rights reserved
1. In ∆ABC , A is least and its value is 45°. If tan A, tan B and tan C are in AP and the area
of the triangle is 27 square units, then match the items of Column I with those of Column
II.

Column I Column II

(A) The side a is of length (p) 6 2

(B) The side b is equal to (q) 3 2

(C) The side c equals 3 10


(r)
2

(D) Circumradius of ∆ABC is (s) 3 5

(t) 9

Solution:

2 tan B = tan C + tan A = tan A ⋅ tan B ⋅ tan C − tan B


=
3tan B tan B ⋅ tan C
tan B(3 − tan C ) =0

Either tan B = 0 or tan C = 3, C = 71.5 ⇒ B = 180 − 45 − 71.5 = 63.5

Now area of triangle

1 2R2
=A =
bc sin 45° = 27
2 2
27 2 3 10
=R =
2 2

2. For any ∆ABC, match the items of Column I with those of Column II.

Column I Column II

Solution Manual for Mathematics For JEE (Main & Advanced), Trigonometry Vector
Algebra Probability, Vol 2,
Copyright©2017 Wiley India Pvt. Ltd. All rights reserved
(A) rr1r2 r3 is equal to A B C
(p) s 3 tan tan tan
2 2 2

rr1 A
(B) equals (q) tan 2
r2 r3 2

(C) r1r2 r3 is equal to (r) ∆ 2

r1r2 r3 A
(D) ( s − a)( s − b)( s − c) = (s) tan 3
∆ 2

Solution:
∆ ∆ ∆ ∆ ∆4
(A) ⋅ ⋅ ⋅ = =
∆2
s ( s − a ) ( s − b) ( s − c ) ∆ 2

Answer: (r)

( s − a ) tan A / 2 ⋅ s tan A / 2 2  A  s ( s − a )( s − b)( s − c ) A


(B) = tan =   tan 2
∆ /( s − b) ⋅ ∆ /( s − c) 2 ∆ 2
2
Answer: (q)
A B C
(C) r1 ⋅ r2=
⋅ r3 s tan ⋅ s tan ⋅ s tan
2 2 2
A B C
= s 3 tan ⋅ tan ⋅ tan
2 2 2
Answer: (p)

∆3
(D) ( s − a )( s − b)( s − c) =
∆2
∆⋅ ( s − a )( s − b)( s − c)
Answer: (r)

3. Match the items of Column I with those of Column II.

Column I Column II

7 (p) 3
(A) In ∆ABC, if cos A + cos B + cos C = then the value of R / r
4
is

A B−C 4
(B) In ∆ABC, if AC = 2AB, then cot cot = (q)
2 2 3

Solution Manual for Mathematics For JEE (Main & Advanced), Trigonometry Vector
Algebra Probability, Vol 2,
Copyright©2017 Wiley India Pvt. Ltd. All rights reserved
(C) In a right-angled triangle, the hypotenuse is four times the (r) 2 3
length of the altitude drawn from the opposite vertex and 2θ is
the difference of the other two acute angles. Then tan 2 2θ =

(D) In ∆ABC, if A = 30°, b = 2, c = 3 and r is the inradius, then (s)


2
the value of 2r + 1 is 3

(t) 3

Solution:
(A)
7
cos A + cos B + cos C =
4
r 7 r 3
1+ = ⇒ =
R 4 R 4
Therefore,
R 4
=
r 3
Answer: (q)
(B)
Since
b = 2C
Now,
A B−C
cot ⋅ cot
2 2
2sin[( B + C ) / 2] ⋅ cos[( B − C ) / 2]
=
2 cos[( B + C ) / 2] ⋅ sin[( B − C ) / 2]
sin B + sin C b + c
= = = 3
sin B − sin C b − c
Answer: (t)
(C)

A + B = 90°
A − B = 20°
2 A= 90° − 20°
cot 2 A = tan 2θ

Solution Manual for Mathematics For JEE (Main & Advanced), Trigonometry Vector
Algebra Probability, Vol 2,
Copyright©2017 Wiley India Pvt. Ltd. All rights reserved
2
 cot 2 A −1 
= 2
= 2
(2θ ) 
 2 cot A 
cot (2 A) tan
 
2
 15 −1  49
=  = = 3
 2 15  15

Answer: (t)

(D)
b2 + c2 − a 2 3
=
cos A = cos(30 =°)
2bc 2
4 + 3 − a2 3
= =
2(2) 3 2
2(3) = 7 − a 2 , a 2 = 7 − 6 = 1
a =1
1 2 31 3
∆= = bc sin ( A)= =
2 2 2 2
∆ 3 3 3 −1
r= = = =
5 2s 1 + 2 + 3 2
2r + 1 = 3
Answer: (p)

4. In ∆ABC, it is given that a = 5, b = 3 and c = 7. Match the items of Column I with those
of Column II.

Column I Column II

(A) Area of ∆ABC is (p) 5

(B) The circumradius is equal to 3


(q)
2

(C) The inradius equals 7 3


(r)
3

(D) The value of 7 cos B + 3cos C is 15 3


(s)
4

Solution:
(A) Area of triangle

Solution Manual for Mathematics For JEE (Main & Advanced), Trigonometry Vector
Algebra Probability, Vol 2,
Copyright©2017 Wiley India Pvt. Ltd. All rights reserved
15 5 9 1 15 3
=
∆ ⋅ ⋅ ⋅=
2 2 2 2 4

Answer: (s)

(B)
abc 5 ⋅ 3 ⋅ 7 ⋅ 4 7 3
=
R = =
4∆ 4 ⋅ 15 3 3
Answer: (r)

(C)
∆ 15 3 3
γ= = =
5 30 2
Answer: (q)

(D)
 74 − 9   34 −19 
7  + 3 
 70   30 
65 15
= + =5
10 10
Answer: (p)

5. In ∆ABC, match the items of Column I with those of Column II.

Column I Column II

 A+ B  A− B (p) b 2
(A) cot   tan   is
 2   2 

1 − tan( A / 2) tan( B / 2) a−b


(B) is equal to (q)
1 + tan( A / 2) tan( B / 2) a+b

(C) If the angles A, B, C (in this order) are in AP, c


(r)
then a + c − ac is equal to
2 2
a+b

1 + cos( A − B) cos C a 2 + b2
(D) equals (s)
1 + cos( A − C ) cos B a2 − c2

a 2 + b2
(t)
a2 + c2

Solution:

(A)
 A− B   a −b  C
tan   =  cot
 2  a+b 2

Answer: (q)

Solution Manual for Mathematics For JEE (Main & Advanced), Trigonometry Vector
Algebra Probability, Vol 2,
Copyright©2017 Wiley India Pvt. Ltd. All rights reserved
(B)
cos ( A + B / 2) sin C / 2
=
cos ( A − B / 2) cos( A − B / 2)
2sin C / 2 ⋅ cos C / 2 sin C C
= =
2 cos ( A − B / 2) ⋅ sin( A + B / 2) sin B + sin C b + a
Answer: (r)

(C)
A+C =
2B
a + c − ac + 3ac − 3ac
2 2

(a + c) 2 − 3ac =
b2
Answer: (p)

(D)

1 − cos 2 A + sin 2 B a 2 + b2
=
1 − cos 2 A + sin 2 C a2 + c2

Answer: (t)

COMPREHENSION TYPE QUESTIONS

1. Passage: ∆1 , ∆ 2 and ∆ 3 are the areas of the triangles cut off by the tangents drawn to the
incircle of ∆ABC and drawn parallel to the sides BC, CA and AB, respectively. Answer
the following three questions.
∆1 ∆2 ∆3
(i) = = = λ , where λ equals
( s − a) 2
( s − b) 2
( s − c)2

∆ ∆2 ∆ ∆2
(A) (B) (C) (D)
s2 s2 s s

(ii) If R1 is the circumradius of the triangle whose area is ∆1 , then R1 is equal to

B C B C
(A) R cot cot (B) R tan tan
2 2 2 2

R R
(C) (D)
3 r

(iii) If R1 , R2 and R3 are the circumradii of the triangles with areas ∆1 , ∆ 2 and ∆ 3
then R1 + R2 + R3 is

2R 3R 4R
(A) (B) R (C) (D)
3 2 3

Solution Manual for Mathematics For JEE (Main & Advanced), Trigonometry Vector
Algebra Probability, Vol 2,
Copyright©2017 Wiley India Pvt. Ltd. All rights reserved
Solution:
(i)

∆1 R tan B / 2 ⋅ tan C / 2 ∆
= =
(s − a) 2
(s − a)2 s2
Answer: (A)

(ii)
B C
=R1 R tan ⋅ tan
2 2
Answer: (B)

(iii)
 B C C A A B
R1 + R2 =
+ R3 R  tan ⋅ tan + tan ⋅ tan + tan ⋅ tan 
 2 2 2 2 2 2
= R ⋅ 1 =R
Answer: (B)

2. Passage: In ∆ABC, let a, b, c be the sides, s the semiperimeter and ∆ the area. Answer the
following three questions.
(i) Minimum value of a 2 + b 2 + c 2 is

(A) 3 2∆ (B) 4 3∆ (C) 3 3∆ (D) 2 3∆

1 1 1
(ii) Minimum value of 2
+ 2
+ is
a b c2

s 2s 3s
(A) (B) (C) (D) 2∆s
abc abc abc

(iii) Minimum value of

a 2 + b2 b2 + c2 c2 + a 2
+ +
a+b b+c c+a

is

(A) 4s (B) 3s (C) 2s (D) s

Solution Manual for Mathematics For JEE (Main & Advanced), Trigonometry Vector
Algebra Probability, Vol 2,
Copyright©2017 Wiley India Pvt. Ltd. All rights reserved
Solution:
(i)
By cosine law
a 2 = b 2 + c 2 = 2bc cos A
a 2 = b 2 + c 2 = 2bc cos A
a 2 + b 2 + c 2= 2(b 2 + c 2 ) − 2bc cos A
1
We know, ∆ = bc sin A
2
4∆ 3 =
(2bc) 3 sin A
a + b + c − 4∆ =
2 2 2
3 2(b 2 + c 2 ) − 2bc (cos A + 3 − sin A)
π 
= 2(b 2 + c 2 ) − 4bc ⋅ cos  − A 
3 
≥ 2(b 2 + c 2 ) − 4bc
= 2(b − c)2 ≥ 0
a 2 + b 2 + c 2 ≥ 4 3∆
min (a 2 + b 2 + c 2 ) = 4 3∆
Answer: (B)

(ii)
1 1 1
2
+ 2
+
a b c2
 1 1 1 
2
1 1 1  1 
=  + +  = + + +2 2
 bc ca ab  (bc) (ca) (ab)
2 2 2
 (abc) 

2
 a+b+c 
= 2 2 2 
a b c 

 1 1 1  2s
min  2 + 2 + 2  =
 a b c  abc
Answer: (B)

(iii)
a 2 + b 2 (a + b)2 − 2ab 2ab
= = ( a + b) −
a+b a+b ( a + b)
2ab 2bc 2ab
( a + b) − + (b + c) − + (c + a ) −
( a + b) (b + c) ( a + b)
1 1 1
2(a + b + c) − 2  + + 
a b c
= a + b + c = 25
Answer: (C)

3. Passage: For any ∆ABC, answer the following three questions.

Solution Manual for Mathematics For JEE (Main & Advanced), Trigonometry Vector
Algebra Probability, Vol 2,
Copyright©2017 Wiley India Pvt. Ltd. All rights reserved
(i) If the sides a, b, c are in GP (in the given order), then the three numbers
 tan B + tan C   tan C + tan A   tan A + tan B 
(b 2 − c 2 )  , (c 2 − a 2 )  , (a 2 − b 2 ) 
 tan B − tan C   tan C − tan A   tan A − tan B 

are in

(A) AP (B) HP

(C) GP (D) increasing order

(ii) If cos A + 2 cos B + cos C =


2, then

1 1 1
(A) , , are in HP (B) a, b, c are in GP
a b c

1 1 1
(C) a, b, c are in HP (D) , , are in GP
a b c

(iii) If sin A, sin B and sin C are in AP, then the altitudes of the triangle are in

(A) AP (B) GP (C) HP (D) AGP

Solution:
(i)
sin B + C sin( B − C ) ⋅ sin(C )
(b 2 − c 2 ) ⋅ = sin( B + C ) ⋅ = sin 2 (C )
sin B − C sin( B − C )
sin 2 ( A), and sin 2 ( B) and sin 2 (C )
Now, we have b 2 = ac
sin=
2
( B) sin( A) ⋅ sin(C )
=
sin 4
( B) sin 2 ( A) ⋅ sin 2 (C )
sin 2 ( A), sin 2 ( B) and sin 2 (C ) arein G.P.
Answer: (C)

(ii)

2 1 1
= + ⇒ 2ac = bc + ab
b a c

2sin A ⋅ sin C = sin B ⋅ sin C + sin A ⋅ sin B

2[cos ( A + C ) − cos (C − A=
)] cos ( B + C ) − cos (C − B) + cos ( A + B) − cos ( B − A)
− 2 cos B − cos (C − A) =
− cos A − cos (C − B) − cos C − cos ( B − A)

2 cos=
B cos( A) + cos C + cos(C − B) + cos( B − A) − cos(C − A)
Now, for cos A + cos C =2 − 2 cos B
A cos B =2 + cos(C − B) + cos( B − A) − cos(C − A)

Solution Manual for Mathematics For JEE (Main & Advanced), Trigonometry Vector
Algebra Probability, Vol 2,
Copyright©2017 Wiley India Pvt. Ltd. All rights reserved
C−A C+A  C−A
=+
2 2 cos   ⋅ cos  − B  − 2 cos 2   +1
 2   2   2 
 C − A  3B C−A
=3 + 2 cos   − cos
2 
sin
 2  2
4cos B = − 2(cos A + cos C ) + 4
2cos B =
− (cos A + cos C ) + 2
cos A + cos C + 2cos B =
2 (Proved)
Answer: (A)

(iii)
=
2sin B sin A + sin C , then altitude rise in H.P.
Answer: (C)

4. Passage: In ∆ABC, the incircle touches the sides BC, CA and AB at the points D, E and F,
respectively. If the radius of the incircle is 4 and the lengths of BD, CE and AF are
consecutive integers, then answer the following three questions.
(i) ∆ABC is

(A) Equilateral

(B) Right-angled

(C) Right-angled isosceles

(D) The sides of ∆ABC are in AP in some order

(ii) Area of ∆ABC is

(A) 2 21 (B) 2 17 (C) 3 11 (D) 84

(iii) In some order sin A, sin B and sin C are in

(A) AP (B) GP (C) HP (D) AGP

(A)

(i)
Let BD = x
then next consecutive integer x + 1, x + 2

Solution Manual for Mathematics For JEE (Main & Advanced), Trigonometry Vector
Algebra Probability, Vol 2,
Copyright©2017 Wiley India Pvt. Ltd. All rights reserved
CE =
x + 1, AF =
x+2
Now,
BD= BF= x and CD= CE= x + 2
AE= AF= x + 2
AB =AF + BF =2 x + 2
BC = x + x + 1 = 2 x + 1
AC = x + 1 + x + z = 2 x + 3
= AB + AC sides are in A.P.
Here, 2BC
Answer: (D)
(ii)
Now, here x = 6
so, side ‘a’ = x + x + 1 = 13
b = x + 1 + x + 2 = 15
c = x + 2 + x = 14
Now,
13 + 14 + 15
=S = 21
2
so, Area ∆ = r ⋅ S = 4* 21 = 84
Answer: (D)
(iii)
As sides a, b, c are in A.P. in some order
So, sin A, sin B, sin C are also in A.P. in some order
Answer: (A)

ASSERTION–REASONING TYPE QUESTIONS


In the following set of questions, a Statement I is given and a corresponding Statement II is given
just below it. Mark the correct answer as:

(A) Both Statements I and II are true and Statement II is a correct explanation Statement I.
(B) Both Statements I and II are true but Statement II is not a correct explanation for
Statement I.
(C) Statement I is true and Statement II is false
(D) Statement I is false and Statement II is true
a 2 + b2 + c2
1. Statement I: In ∆ABC, the inradius r is less than
3(a + b + c)

1 1 1
Statement II: Area of ∆ABC
= bc sin=
A ca sin=
B ab sin C
2 2 2

Solution: We have
∆ 2∆
r= =
S (a + b + c)
Now,
1 1 1
=∆ =ab sin C = bc sin A ac sin B
2 2 2
ab sin C bc sin A ac sin B
=r = =
(a + b + c) a + b + c a+b+c
ab sin C
3r = ∑ +
a+b+c

Solution Manual for Mathematics For JEE (Main & Advanced), Trigonometry Vector
Algebra Probability, Vol 2,
Copyright©2017 Wiley India Pvt. Ltd. All rights reserved
ab sin C
r= ∑
3(a + b + c)
Now, we know, that
(a + b + c)2 = a 2 + b 2 + c 2 + 2(ab + bc + ca)
a 2 + b 2 + c 2 = (a + b + c)2 − 2(ab + bc + ca)
this whole term
(a + b + c)2 − 2(ab + bc + ca) > ab
so,
a 2 + b 2 + c 2 > abc
so,
a 2 + b2 + c2
r< (Proved)
3(a + b + c)
Answer: (A)

2. Statement I: In an acute-angled ∆ABC, if

8(cos3 A + cos3 B + cos3 C − 3cos A cos B cos C ) + 36(cos A cos B + cos B cos C + cos C cos A) =
27

then ∆ABC is equilateral.

Statement II: In any ∆ABC ,

3
cos A + cos B + cos C ≤
2

where the equality holds if and only if the triangle is equilateral.

Solution:
Statement I
Here
π 
= CyB
CxA = CzC
= cb  
3

 1+1+1  3 1 1 1
8  − ⋅ 8 + 36  + +  =27 (Proved)
 8  8 4 4 4

Statement II

π
A= B= C=
3

1 1 1 3
+ + = (Proved)
2 2 2 3
Answer: (A)

3. Statement I: In ∆ABC , C =
90° and the bisector of the angle C meets the side AB
internally in D. If p = AD and q = BD, then

Solution Manual for Mathematics For JEE (Main & Advanced), Trigonometry Vector
Algebra Probability, Vol 2,
Copyright©2017 Wiley India Pvt. Ltd. All rights reserved
p 1 − tan[( A − B) / 2]
=
q 1 + tan[( A − B) / 2]

Statement II: In a triangle, internal bisector of an angle divides opposite side in the
ratio of the other two sides.
Solution:

p+q =C

q − p tan( A − B / 2) + tan( A − B / 2)
=
q+ p 1+1
q− p  A− B   a −b  C
= tan =
    cot
q+ p  2   a+b  2
a −b q − p p a
= = ,
a+b q+ p q b

Answer: (A)

4. Statement I: The sides a, b, c of ∆ABC are, respectively, 18, 24 and 30. Then its
inradius is 9.

Statement II: The inradius r = ∆ / s, where ∆ is the area and s is the semiperimeter
of the triangle.

Solution:
18 + 24 + 30
=s = 36
2
=∆ 36(36 −18)(36 − 24)(36 − 30)
= 6* 18 **
12 6
= 6 ∗ 6 ∗ 6 = 216

216
r= = 6 False
36

Answer: (D)

Solution Manual for Mathematics For JEE (Main & Advanced), Trigonometry Vector
Algebra Probability, Vol 2,
Copyright©2017 Wiley India Pvt. Ltd. All rights reserved
5. Statement I: In ∆ABC, ‘I’ and ‘O’ denote the incentre and circumcentre,
respectively. If BIO= 90°, then a + c = 2b.

Statement II: In ∆ABC , ( IO)2 =


R 2 − Rr

Solution:

As line connecting incentre and circumcentre makes 90° with vertex B. So, side
lengths of ΔABC are in arithmetic series. Hence,
b−a =c −b
2b − a + c
Answer: (C)

INTEGER ANSWER TYPE QUESTIONS


The answer to each of the questions in this section is a non-negative integer. The appropriate
bubbles below the respective question numbers have to be darkened. For example, as shown in the
figure, if the correct answer to the question number Y is 246, then the bubbles under Y labeled as
2, 4, 6 are to be darkened.

X Y Z W
ⓞ ⓞ ⓞ ⓞ
① ① ① ①
② ② ② ②
③ ③ ③ ③
④ ④ ④ ④
⑤ ⑤ ⑤ ⑤
⑥ ⑥ ⑥ ⑥
⑦ ⑦ ⑦ ⑦
⑧ ⑧ ⑧ ⑧
⑨ ⑨ ⑨ ⑨

<COMP: Shade 2, 4, 6 in column Y>

1. In ∆ABC, if a = 18, b = 24 and c = 30, then the value of cosec C is ______.

Solution:

Solution Manual for Mathematics For JEE (Main & Advanced), Trigonometry Vector
Algebra Probability, Vol 2,
Copyright©2017 Wiley India Pvt. Ltd. All rights reserved
a 2 + b 2 − c 2 324 + 576 − 900
=
cos(C ) = = 0
2bc 2 ⋅18 ⋅ 24

sin(C )= 90°= 1
co sec(C ) = 1
Answer: (1)

2. In ∆ABC, if a = 25, b = 52 and c = 63, then the value of 5 tan( A / 2) + 2 tan( B / 2) is equal
to ______.

Solution:

A ( s − b)( s − c) 18 2 ⋅ 7
=tan =
2 s( s − a) 70 10 5 ⋅ 455

25 + 52 + 63 1
=
s = 70
=
2 5
B ( s − a )( s − c) 45 ⋅ 7 1
=tan = =
2 s ( s − b) 70 10 2 ⋅ 182 2
1 1
5 ⋅ + 2 ⋅ =2
5 2
Answer: (2)

3. In ∆ABC, if a = 6, b = 3 and cos( A − B) =


4 / 5, then its area is ______.

Solution:
 A− B  1 − cos( A − B / 2) 1
=
tan   =
 2  1 + cos( A − B / 2) 3
Now,
 A− B  a−b C
tan   = cot
 2  a+b 2
1 6−3 C
= cot  
3 6+3  2 
C
cot  = 1, C= 90°
2
1 1
∆ = ab = ⋅ 6 ⋅ 3 = 9
2 2
Answer: (9)

( I 2 I3 )2
4. In ∆ABC , is equal to ______.
R(r2 + r3 )

Solution:

Solution Manual for Mathematics For JEE (Main & Advanced), Trigonometry Vector
Algebra Probability, Vol 2,
Copyright©2017 Wiley India Pvt. Ltd. All rights reserved
( I 2 I3 )2 4 R 2 cos 2 A / 2 4 R 2 cos 2 A / 2
= =
R(r1 + r2 ) R(∆ / s − b + s / s − c) ∆(2 s − b − c) /( s − b)( s − c)

4 R ⋅ cos 2 A / 2 ( s − b)( s − c)
= ⋅
1/ 2 R sin A s( s − a)
4 cos A / 2 A
= ⋅ cot= 4
sin A / 2 2
Answer: (4)

5. The circumradius of ∆ABC is ______ times the circumradius of its pedal triangle.

Solution: We know that an pedal triangle DEF

∠D
= 180° − 2 A°, ∠=
E 180° − 2 B°, ∠=
F 180° − 2C °

R = 2 R1
where R1 is the circumradius of pedal triangle
Answer: (2)

6. In ∆ABC, the distances of the circumcentre from the sides BC, CA and AB are,
respectively, p1 , p2 and p3 . Then abc / p1 p2 p3 is ______ times
(a / p1 ) + (b / p2 ) + (c / p3 ) .

Solution:

Let ΔABC is equilateral Δ

BD a
cot (30=
°) =
OD 2 p1

a
= 2 cot=
(30°) 2 3
p1
Similarly
b C
= 2=3
p2 p3
So,
abc
= 24 3
p1 p2 p3

Solution Manual for Mathematics For JEE (Main & Advanced), Trigonometry Vector
Algebra Probability, Vol 2,
Copyright©2017 Wiley India Pvt. Ltd. All rights reserved
So,
24 3 = k (6 3)
k =4
Now,
a b c
+ + =6 3
p1 p2 p3
Answer: (4)

7. If the line joining the incentre and the circumcentre of triangle ABC is parallel to the side
BC, then the value of cos B + cos C is ______.

Solution:

I = incentre
C = circumcentre
As we know,
AI b + c
=
IE a
As we let ΔABC is equilateral Δ. So,
1 1
cos B + cos C = + =1
2 2
Answer: (1)

8. In ∆ABC, the value of

a cos A + b cos B + c cos C r


=
a+b+c kR

where k is equal to ______.

Solution:
k =1

a 2 (b 2 + c 2 − a 2 ) + b 2 (c 2 + a 2 − b 2 ) + c 2 (a 2 + b 2 − c 2 )
2abc(a + b + c)

(a 4 + b 4 + c 4 − 2 a 2b 2 − 2b 2 c 2 − 2c 2 a 2 )
=
2(abc)(a + b + c)

Solution Manual for Mathematics For JEE (Main & Advanced), Trigonometry Vector
Algebra Probability, Vol 2,
Copyright©2017 Wiley India Pvt. Ltd. All rights reserved
16∆ 2 8∆ 2
= =
2(a + b + c)(abc) (abc)(a + b + c)
8⋅∆ ⋅∆ 1 r
= = ⋅r=
(a + b + c) ⋅ (abc) R R
Answer: (1)

9. In ∆ABC, the minimum value of

a b c
+ +
s−a s−b s−c

is ______.

Solution:

a 2a 2a (b + c + a )
= =
s − a b + c − a (b + c) 2 − a 2
2a(b + c + a) 4 S ⋅ 2 R ⋅ sin A / 2 ⋅ cos A / 2 RS A
= = tan
2bc(1 + cos A) 2bc ⋅ 2 cos A / 2
2
bc 2

2 RS A
tan ≥ 2
bc 2
Similarly
b
≥2
s −b
and

c
≥2
s −c

so,
min = 2 + 2 + 2 = 6
Answer: (6)

10. If H is the orthocentre of ∆ABC whose circumradius is 4, then the circumradius of ∆BHC
is ______.

Solution:

Solution Manual for Mathematics For JEE (Main & Advanced), Trigonometry Vector
Algebra Probability, Vol 2,
Copyright©2017 Wiley India Pvt. Ltd. All rights reserved
Circumradius R = 4
Here, we took ΔABC as equilateral triangle
So, BH = CH = 4
Now, 4n ∆BHC , BH ⊥ CH
So, circumradius of ΔBHC is point H and so, circumradius of ΔBHC = BH = 4
Answer: (4)

<H1>ANSWERS

<H2>Single Correct Choice Type Questions

1. (C) 14. (A)


2. (B) 15. (D)
3. (A) 16. (C)
4. (D) 17. (B)
5. (D) 18. (B)
6. (A) 19. (C)
7. (B) 20. (D)
8. (D) 21. (A)
9. (A) 22. (D)
10. (C) 23. (B)
11. (B) 24. (A)
12. (B) 25. (C)
13. (A)

<H2>Multiple Correct Choice Type Questions

1. (A), (C), (D) 6. (A), (B), (C)


2. (A), (B), (C), (D) 7. (A), (B), (C), (D)
3. (A), (B), (C), (D) 8. (A), (B), (D)
4. (A), (B), (C), (D) 9. (A), (B), (D)
5. (A), (B), (D) 10. (A), (C)
<H2>Matrix-Match Type Questions

1. (A) → (s), (B) → (p), (C) → (t), (D) → 2. (A) → (r), (B) → (q), (C) → (p), (D) →
(r) (r)

Solution Manual for Mathematics For JEE (Main & Advanced), Trigonometry Vector
Algebra Probability, Vol 2,
Copyright©2017 Wiley India Pvt. Ltd. All rights reserved
3. (A) → (q), (B) → (t), (C) → (t), (D) → 5. (A) → (q), (B) → (r), (C) → (p), (D) →
(p) (t)
4. (A) → (s), (B) → (r), (C) → (q), (D) →
(p)

<H2> Comprehension Type Questions

1. (i) (A); (ii) (B); (iii) (B) 3. (i) (C); (ii) (A); (iii) (C)
2. (i) (B); (ii) (B); (iii) (C) 4. (i) (D); (ii) (D); (iii) (A)
<H2>Assertion–Reasoning Type Questions

1. (A) 4. (D)
2. (A) 5. (C)
3. (A)

<H2>Integer Answer Type Questions

1. 1 6. 4
2. 2 7. 1
3. 9 8. 1
4. 4 9. 6
5. 2 10. 4

Solution Manual for Mathematics For JEE (Main & Advanced), Trigonometry Vector
Algebra Probability, Vol 2,
Copyright©2017 Wiley India Pvt. Ltd. All rights reserved
Solutions to Exercises
Chapter 5

SINGLE CORRECT CHOICE TYPE QUESTIONS


    
1. In a regular hexagon ABCDEF, vector AB + AC + AD + AE + AF is equal to
   
(A) 2AD (B) 3AD (C) 4AD (D) 6AD
Solution:

Here
AB = − DE

and AF = CD
Now,
 
AE − DE = AD
 
and AC + CD = AD
so,
AB + AC + AD + AE + AF =
3 AD
Answer: (B)

2. ABCD is a quadrilateral and not a parallelogram described in this order. Let P, Q, R and S
be the mid-points of the sides AB, BC, CD and AD, respectively. Then PQRS is a
(A) Parallelogram (B) Rectangle
(C) Square (D) Rhombus
Solution:

   d
Let position vector of A = a that of B, C and D are b, c and d
Now, position vector of P,
dd
d b + a d c + b
= OP = , OQ ,
2 2
d d + c a +d
= OR = , OS
2 2
Now,
c +d  b +a  c +d b +a  c +d −b −c 
PQ =  − = − =  
 2   2  2  2   2 
 a + b  c + d  a + b − c − d c+d −a −b 
Rs =  −  = = − 
 2  2  2  2 
Here
Solution Manual for Mathematics For JEE (Main & Advanced), Trigonometry Vector
Algebra Probability, Vol 2,
Copyright©2017 Wiley India Pvt. Ltd. All rights reserved
d
PQ =
− RS and QR =
− PS
Hence, PQRS is parallelogram.
Answer: (A)
  
3. Let A and B be points whose position vectors are, respectively, i + 3 j − 2k and
  
3i + j − 2k . The bisector of the angle AOB (‘O’ is the origin) meets the segment AB

internally in C. Then the vector OC is equal to
     
(A) i + j + k (B) 2(i + j − k )
     
(C) i + j − k (D) 4(i + j + k )
Solution: Here point ‘C’ is bisector point of line AB so,
     
 i + 3 j − 2k + 3 j + j − 2k   
OC= = 2(i + j − k )
2
Answer: (B)
  
4. Let a, b, c be non-coplanar vectors. If
           
a + 3b + 4c = x(a + 5b − 2c) + y (a − 2b + 3c) + z (6a + 14b + 4c)
then
(A) x = −3, y = 2, z = 1 (B) x = 3, y = −2, z = 1
(C) x = −3, y = −2, z = 1 (D)= x 3,= y 2,= z 1
  
Solution: As a , b , c , be non-coplanar vectors, so, their sector coefficients are equal
Here,
     
a + 3b + 4c =( x + y + 6 z )a + (5 x − 2 y + 14 z )b + (− 2 x + 3 y + 4 z )c
x + y + 6 z = 1, 5 x − 2 y + 14 z = 3, − 2 x + 3 y + 4 z = 4
7 x=+ 26 z 5 and 11x= + 34 z 17
x= − 3, z = 1, y = −2
Answer: (C)
  
5. a is a vector in the Cartesian plane in which OX and OY are coordinate axes having
components 2p and 1. The axes are rotated in the same plane about origin, through the

angle 45° in the counterclock sense. The components a with respect to the new system
axes are 2 p + 1 and 1. Then the value of p is
1 1
(A) 0 (B) –1 or (C) − or 1 (D) 1 or –1
3 3

Solution: Magnitude of a is remain same in both conditions, so
(2 p 2 ) + 1= ( p + 1) 2 + 1
4 p2 = p2 + 2 p + 1
3 p2 − 2 p − 1 =0
−1
( p − 1)(3 p + 1) = 0 ⇒ p =1,
3
Answer: (C)
     
6. Let the points A, B and C be represented by the vectors i + j , − 4i + 2 j and 2i + 3 j ,
respectively, in the Cartesian plane. Then the length of the median through B of ∆ABC is
5 2 11 11
(A) (B) (C) (D)
2 5 3 2
Solution Manual for Mathematics For JEE (Main & Advanced), Trigonometry Vector
Algebra Probability, Vol 2,
Copyright©2017 Wiley India Pvt. Ltd. All rights reserved
Solution:

Position vector of mid-point E of side AC



 i + 2i j + 3 j 
 , 
 2 2 
 3 
 i, 2 j 
2 
Now,
3  dd 11  
side BE=  − (−4)  i + (2 −= 2) j   i + 0
2  2
Side
2
 11  11
=BE =
 
 2 2
Answer: (D)

7. ABCD is a tetrahedron and O is a point inside the tetrahedron. The lines AO, BO, CO and
DO meet the opposite faces in P, Q, R and S, respectively, in P, Q, R and S. Then
AP BQ CR DS
(A) + + + = 1
OP OQ OR OS
OP OQ OR OS
(B) + + + = 1
AP BQ CR DS
AO BO CO DO
(C) + + + = 1
OP OQ OR OS
OP OQ OR OS
(D) + + + = 1
OA OB OC OD
Solution:

1
Here, we take point ‘O’ as centroid then AP is a median for ΔABZ, such that OP = AP
3
2
and OA = AP
3
so,

Solution Manual for Mathematics For JEE (Main & Advanced), Trigonometry Vector
Algebra Probability, Vol 2,
Copyright©2017 Wiley India Pvt. Ltd. All rights reserved
OP 1
=
AP 3
Similarly
OQ 1 OR 1
= = and
BQ 3 CR 3
So,
OP OQ OR 1 1 1
+ + = + + =1
AP BQ CR 3 3 3
Answer: (B)
  
8. The line passing through the point 2a + 3b and parallel to the vector c cuts the plane
        
r = a − b + t ( a + b − c ) s ( a + c − b)
where t and s are scalars in the point whose position vector is
     
(A) 2a + 3b + 4c (B) 2a − 3b + 4c
     
(C) 2a − 3b − 4c (D) 2a + 3b − 4c
 
Solution: Any vector parallel to vector C = SC
   
and let O be origin, and A and B are 2 points withP.V. OA = a and OB = b let ‘L’ be line
      
passing through 2OA + 3OB = 2a + 3b and line AB= a − b
  
Now, line is passing through 2a + 3b and parallel to C is
        
r = a − b + s (a + b − c) + (a − b + c)
Answer: (D)

9. P is the mid-point of the side AD of the parallelogram ABCD. The line BP meets the
diagonal AC in Q and the line CD in R. Then RQ:QB is equal to
(A) 2:1 (B) 1:2 (C) 3:1 (D) 1:3
Solution:

  d d
Let ‘A’ is origin and AB = b and AD = d
As ABCD is a parallelogram
so,
dd d
AC= b + d
d
d d
and AP =
2
Let point Q divide AC in μ:1 and BP in λ :1 then,
d d dd
µ (b + d ) λ (d / 2) + 1(b)
=
µ +1 λ +1
µ 1 λ
= =
µ +1 λ +1 2(λ +1)
From this we get
Solution Manual for Mathematics For JEE (Main & Advanced), Trigonometry Vector
Algebra Probability, Vol 2,
Copyright©2017 Wiley India Pvt. Ltd. All rights reserved
λ 2 − λ −1= 0
λ= 2
Hence, point Q divides BR in 1: 2
Answer: (B)
       
10. The three points i − 2 j + 3k , 2i + 3 j − 4k and −7 j + 10k
(A) are collinear
(B) form the vertices of a right-angled triangle
(C) form an isosceles triangle
(D) are non-collinear
Solution: If given points are collinear then
1 −2 3
2 3 4 =2 − 2 (− 20) + 3(−14) =0
0 −7 10
Hence given points are collinear.
Answer: (A)

11. OXYZ is a rectangular coordinate system. Keeping Z-axis fixed, the X- and Y-axes are
rotations in their plane through angle 45° in the anti-clockwise sense. If the components
of a vector with respect new system are 2 2, 3 2 and 4, then the components of the
same vector with respect to the original systems are
(A) 5, –1, 4 (B) –1, 5, 4
(C) −1, − 5, 4 2 (D) −1, 5, 4 2
Solution Let the component of vector with respect +0 old system are x, y, z
Since system is rotating about z axis. Therefore, components of vector with respect to
new system are x′, y ′, z − 2 2,3 2, 4
x y
2 2 1 1
2 2
3 2 −1 1
2 2

x y
2=
2 +
2 2
x y
and 3 2=
− +
2 2
Solving x and y, we get x = −1 and y = 5.
Answer: (B)
       
12. In ∆ABC , AB = 2i + 4 j + 4k and AC = 2i + 2 j + k . Then the length of the median
through A is
77
(A) (B) 5 (C) 5 2 (D) 10 2
2
Solution:

Solution Manual for Mathematics For JEE (Main & Advanced), Trigonometry Vector
Algebra Probability, Vol 2,
Copyright©2017 Wiley India Pvt. Ltd. All rights reserved
Point D:→

(2 + 2)  (4 + 2)  (4 + 1)k
= i+ j+
2 2 2
  5
= 2i + 3 j + k
2
2
5 77
AD = (2)2 + (3) 2 +   =
 
2 2
Answer: (A)
  
13. a , b , c are non-zero vectors such that they are pairwise non-collinear. It is given that
        
a + 2b is collinear with c and b + 3c is collinear with a. Then the vector a + 2b + 3c is
 
(A) equal to 0 (B) parallel to b
 
(C) parallel to a (D) parallel to c
<AQ: Solution missing. Please check>
          
14. Let a , b , c be non-coplanar vectors and x =2a + 3b − c, y =a − 2b + 2c and
         
z =−2a + b − 2c. Then the vector 3a − b + 2c in terms of x, y and z is
     
(A) 5 x + 2 y + 3z (B) 3x + 5 y + 2 z
     
(C) 2 x + 5 y + 3z (D) 5 x + 3 y + 2 z
Solution:
  
2 x + 5 y + 3z
        
2(2a + 3b − c) + 5(a − 2b + 2c) + 3(2a + b − 2c)
  
= 3a − b + 2c
Answer: (C)
          
15. Let a, b and c be three non-coplanar vectors and p = 5a + 6b + 7c, q = 7a − 8b + 9c
      
and r =3a + 20b + 5c . If =r x p + yq where x and y are scalars, then
(A) x= 1= y (B) x + y =
1
3
=
(C) x = ,y 1 (D) x = 2, y = −1
2
Solution:
5x + 7 y = 3 x = 2, y = −1
6x − 8 y =20
Answer: (D)

16. A, B, C and D are four points whose position vectors are, respectively,
            
4i + 5 j + k , − i − k , 5i + 9 j + 4k and −i + k . If =
AB x AC + y AD, then

Solution Manual for Mathematics For JEE (Main & Advanced), Trigonometry Vector
Algebra Probability, Vol 2,
Copyright©2017 Wiley India Pvt. Ltd. All rights reserved
−2 2
=
(A) x = ,y
3 3
2 −2
(B)=x = ,y
3 3
3 2
(C)=x = ,y
2 3
(D) Real values of x and y do not exist
Solution:
   
AB = − 5i − 5 j − 2k
   
AC = i + 4 j + 3k
   
AD = − 5i − 5 j + o
− 5 = x − 5 y , − 2 = 3 x, − 5 = 4 x − 5 y
This is not satisfied for real x and y.
Answer: (D)
 
17. Let A and B be two points whose position vectors are 5i and 5 j , respectively. A point P
divides the line joining A and B in the ratio AP : PB = λ :1. If the position vector of P has
magnitude less than or equal to 37, then
−1 1 −1
(A) ≤λ≤ (B) λ ≤ −6 or λ ≥
6 6 6
(C) λ ≥ 0 (D) 0 < λ < 1
Solution:


 λ 5 j + 5i
OP =
1+ λ
 5 1 + λ 2
OP =
1+ λ
Now,
2
 5 1+ λ 2 
  ≤ ( 37) 2
 1+ λ 
 
12λ 2 + 37λ + 12 ≥ 0
−1
λ ≤ − 6, and λ ≥
6

Solution Manual for Mathematics For JEE (Main & Advanced), Trigonometry Vector
Algebra Probability, Vol 2,
Copyright©2017 Wiley India Pvt. Ltd. All rights reserved
Answer: (B)

18. In ∆ABC, E is the mid-point of the median AD. The line BE meets the side AC in F. Then
AC equals
(A) 3 AF (B) 2 AF
3
(C) 3 CF (D) CF
2
Solution:

Let ‘A’ is the origin,


   
and AB = b and AC = C
   
 b + c  b + c
=D = ,ε
2 4
 
   b+c 
BE =(1 − r ) b + r  
 4 
  
=AF S= AC Sc
Position vector F must satisfy both equations. Hence, equating the corresponding
coefficients we get,
r r
1 − r += 0, = S
4 4
3r r
1 − = 0, = S
4 4
4 4 1 1
r= , S= × =
3 3 4 3
 1   
=AF = AC , AC 3 AF
3

Answer: (A)

19. ABCD is a quadrilateral. P, Q, R and S are the mid-points of the sides AB, BC, CD and
AD, respectively. The lines PR and QS intersect in E. If ‘O’ is any point other than any
   
point mentioned above, then the vector OA + OB + OC + OD is equal to
 
(A) 8OE (B) 6OE
 
(C) 4OE (D) 3OE
Solution:

Solution Manual for Mathematics For JEE (Main & Advanced), Trigonometry Vector
Algebra Probability, Vol 2,
Copyright©2017 Wiley India Pvt. Ltd. All rights reserved
Now,
   
(OA + OB) + (OC + OD)
 
= 2OP + 2OR
 
= 2(OP + OR)
 
= 2(2 = OE ) 4OE
Since E is the mid-point of PR and QS
Answer: (C)

20. ABCDE is pentagon in which AB, CD are parallel and AE, BC are parallel. The lines BE
and AD intersect in X. Then the ratio AX:XD is equal to
(A) 2:5 (B) 5:2 (C) 2:3 (D) 3:2
Solution:

   
Let ‘A’ is the origin point and=
AB b= , BC c
As
   
AE BC , AE = c
   
AB CD , CD = b
Now, let point X, divide AD into λ:1 and BE into μ:1, so,
ddd ddd
AD = c + b and BC = c −b
   
AE = e= c − b
    
µ (c − b ) + b λ (c + b )
=
µ +1 1+ λ
µ λ 1− µ
= =
µ +1 1 + λ 1 + µ
1
µ=
2
so,
λ 1 2
= = , λ
1+ λ 3 5

Answer: (A)

Solution Manual for Mathematics For JEE (Main & Advanced), Trigonometry Vector
Algebra Probability, Vol 2,
Copyright©2017 Wiley India Pvt. Ltd. All rights reserved
<H1>ANSWERS
<H2>Single Correct Choice Type Questions

1. (B) 11. (B)


2. (A) 12. (A)
3. (B) 13. (A)
4. (C) 14. (C)
5. (C) 15. (D)
6. (D) 16. (D)
7. (B) 17. (B)
8. (D) 18. (A)
9. (A) 19. (C)
10. (A) 20. (A)

Solution Manual for Mathematics For JEE (Main & Advanced), Trigonometry Vector
Algebra Probability, Vol 2,
Copyright©2017 Wiley India Pvt. Ltd. All rights reserved
Solutions to Exercises
Chapter 6

SINGLE CORRECT CHOICE TYPE QUESTIONS


          
1. If a =i + 2 j + k and =
b 2i − j , then the angle between the vectors a − b and a + b
is

1 1
(A) cos −1   (B) cos −1  
 13   11 

π π
(C) (D)
3 4

Solution
         
a − b =i + 2 j + k − 2i + j =− i + 3 j + k
         
a + b =i + 2 j + k + 2i − j =3i + j + k
Angle between them, θ
     
[(−i + 3 j + k ) ⋅ (3i + j + k )]
cos(θ ) =
(12 + 32 + 12 )(32 + 12 + 12 )
−3 + 3 + 1
=
11
1
θ = cos −1  
 11 
Answer: (B)
   
3 and | b |= 4, then the scalar product of
2. If the angle between a and b is 120°, | a |=
   
3a − 2b and a + 2b is

(A) 61 (B) 41 (C) −41 (D) −61

Solution
         
(3a − 2b ) ⋅ (a − 2b=) 3 | a | + 6 a ⋅ b − 2 a ⋅ b − 4 | b |2
= 3(3) 2 + 4(− 6) − 4(4) 2
=27 − 24 − 64 =−61

Answer: (D)
       
3. Let a = i + j, b = i − j . Suppose x and y are two vectors satisfying the two
      
equations 2x + y = a and x + 2 y = b . If θ is the angle between the two vectors x and

y, then cos θ is equal to

1
(A) −1 (B) 0 (C) (D) 1
2

Solution Manual for Mathematics For JEE (Main & Advanced), Trigonometry Vector
Algebra Probability, Vol 2,
Copyright©2017 Wiley India Pvt. Ltd. All rights reserved
Answer: (A)

2bx + by =
1
bx + 2by =
−1
bx = 1, by = −1
 1   1 
x =i + j , y =
− i−j
3 3

<solution missing>
     
4. Let a = 6i − 8 j − (15 / 2)k . If b is collinear with a , making a obtuse angle with z-axis
 
and | b |= 50, then b is equal to
     
(A) −24i + 32 j + 30k (B) 24i + 32 j − 30k
     
(C) 24i − 32 j − 30k (D) 24i − 32 j − 15k

Solution
 
As b collinear with a
   15 
b= λ (6i − 8 j − k ).
2
d d
As, angle between a and b is obtuse, so
 
a ⋅b <0
   15      15  
 6i − 8 j − 2 k  ⋅ λ  6i − 8 j − 2 k  < 0
   
 225 
λ  36 + 64 + <0
 4 
625
λ < 0, hence λ < 0
4

Now, | b | = 50
 25 
so, λ   = 50, λ = − 4
 2 
   
b=− 24i + 32 j + 30k
Answer: (A)

    
5. a , b , c are three non-zero vectors such that no two are collinear. If a + b is collinear
      
with c and b + c is collinear with a , then a + b + c is equal to
    
(A) 0 (B) 2(a + c ) (C) 2b (D) 3b

Solution
As it is given that no two vector are collinear, so these sum are also not collinear.

Solution Manual for Mathematics For JEE (Main & Advanced), Trigonometry Vector
Algebra Probability, Vol 2,
Copyright©2017 Wiley India Pvt. Ltd. All rights reserved
Answer: (A)

6. If A (3, 2, −3), B (5, 1, −1) and C (1, −2, 1) are the vertices of a triangle, then the
external angle at the vertex A is

2 4
(A) π − cos −1   (B) π − cos −1  
9 9

4 2
(C) cos −1   (D) cos −1  
9 9

Solution

   
AB = 2i − j + 2k
   
AC = −2i − 4 j + 4k
− 4+ 4+8 4
= cos(θ ) =
3.6 9
 4 
External angle= π − cos −1  
9
Answer: (B)
     
7. In a trapezium ABCD, BC = λ AD. If the vector=
x AC + BD is collinear with AD
 
and x = µ AD, µ equals

(A) λ − 1 (B) 1 − λ (C) 2 + λ (D) λ + 1

Solution

  


=
AC AB + BC
  
=
BD AD − AB

Solution Manual for Mathematics For JEE (Main & Advanced), Trigonometry Vector
Algebra Probability, Vol 2,
Copyright©2017 Wiley India Pvt. Ltd. All rights reserved
    
AC + BD =BC + AD =(λ + 1) AD
µ= λ + 1
Answer: (D)
  
=
8. In a parallelogram ABCD, it is given that | AB | a=
, | AD | b and | BD |= c, then the
 
scalar product of AC and AD is equal to

a 2 + 3b 2 − c 2 3a 2 + b 2 − c 2
(A) (B)
2 2

a 2 + 2b 2 + c 2 c2 + b2 − a 2
(C) (D)
2 2

Solution

d d d
AC= a + b
d d
AC d c
and = a+
2 2
d dd
AC= 2a + c
 
Now, AC ⋅ AD
 
=| AC | ⋅ | AD | ⋅ cos θ
  
= (2a + c ) ⋅ (b ) ⋅ cos θ
a 2 + 3b 2 − c 2
=
2
Answer: (A)
   
9. ABCD is a parallelogram in which AB = p and AD = q. DM is drawn perpendicular

to AB. Then DM is equal to
     
 ( p ⋅ q) p ( p ⋅ q) p 
(A) q −  2 (B)  −q
| p| | p |2
     
(q ⋅ p)q   (q ⋅ p) p
(C)  −p (D) q − 
| p| | p|

<solution missing>

Solution Manual for Mathematics For JEE (Main & Advanced), Trigonometry Vector
Algebra Probability, Vol 2,
Copyright©2017 Wiley India Pvt. Ltd. All rights reserved
      
10. In ∆ABC , C =
90° and | AB |= c. Then the sum AB ⋅ AC + BC ⋅ BA + CA ⋅ CB equals

(A) 0 (B) 2c 2 (C) 3c 2 (D) c 2

Solution

   


AB ⋅ AD = | AB | ⋅ | AC | ⋅ cos θ
   
BA ⋅ BC = | BA | ⋅ | BC | sin θ
 
CA ⋅ CB = 0
   
sum =AC ⋅ (| AB | ⋅ cos θ )+ | ( BC ) || BA sin |
 
=| AC |2 + | BC |2 = c 2
Answer: (D)

11. A(1, y1 ) and B( x2 , 1) are points on the curve y = x 2 − 4 x + 5 in the rectangular


Cartesian coordinate system. If “O” is the origin of the axes, then the scalar product
 
OA and OB is

(A) 4 (B) 2 (C) 6 (D) 8

Solution
Point A(11 y1) satisfy = y x2 – 4x + 5
y=
1 1 – 4 + 5= 2
Point A = (1, 2)
Point B (x2, 1) satisfy 1 = x2 2 − 4 x2 + 5
x2 = 2
Point B = (2, 1)
 
OA ⋅ OB = 1.2 + 2.1 = 4
Answer: (A)

12. In a quadrilateral ABCD, angle A is equal to 120° and the diagonal AC is the bisector
  
of the angle A.= =
If | AC | (1/ 5) | AB | (1/ 3) | AD | and θ is the angle between the
 
vectors BA and CD, then cos θ is equal to

1 2 3 4
(A) (B) (C) (D)
7 7 17 17

Solution

Solution Manual for Mathematics For JEE (Main & Advanced), Trigonometry Vector
Algebra Probability, Vol 2,
Copyright©2017 Wiley India Pvt. Ltd. All rights reserved
 
Angle between CD and BA
 
| CD | ⋅ | AB |
cos(θ ) =  
| CD | ⋅ | AB |
  3 1 
   AB :  − 
AB ⋅ ( AD − AC ) 5 5
= =   
  2  
| AB | ⋅ | AD − AC |
| AB | ⋅   AB
5
2
=
7
Answer: (B)
 
13. If a and b are non-collinear vectors and
    
[( x − 2)a + b ] × [(2 x + 1)a − b ] =
0

then the scalar x is equal to

1 1 1 1
(A) (B) − (C) − (D)
2 2 3 3

Solution
As cross product = 0
So, (x − 2) (2x + 1) = 0
Either x − 2 = 0, x = 2
1
Or, 2x + 1 = 0, x = −
2
Answer: (B)

14. ABCD is a rhombus having sides of one unit length. If the angle BAD is 30°, then

| AC | is equal to

(A) 2− 3 (B) 2+ 3

(C) 1 + 3 (D) 3 −1

Solution

Solution Manual for Mathematics For JEE (Main & Advanced), Trigonometry Vector
Algebra Probability, Vol 2,
Copyright©2017 Wiley India Pvt. Ltd. All rights reserved
 
From | AC=
|2 | AB + BC |2
= 1 + 1 + 2 ⋅1 ⋅1 ⋅ cos θ (150)
= 2+ 3

AC= 2+ 3
Answer: (B)
   
15. The vectors a and b are non-zero vectors of equal magnitudes. If the vectors a + 2b
   
and 5a − 4b are orthogonal to each other, then the angle between a and b is

π π π 2π
(A) (B) (C) (D)
6 4 3 3

Solution
   
(a + 2b ) ⋅ (5a − 4b ) = 0
2 2  
5 | a | − 8| b | + 6a ⋅ b = 0
5 − 8 + 6 cos(θ ) =0
3
cos(θ ) =
6
θ= 60°
Answer: (C)

16. In a triangle, the vertices are at points A (3, 2, −3), B (5, 1, −1) and C (1, −2, 1). Then
the angle at the vertex A is

4 2
(A) cos −1   (B) cos −1  
9 9

π 2π
(C) (D)
3 3

Solution
 
AB =(2, − 1, 2), AC =(−2, − 4, 4)
− 4+ 4+8 4
=
cos( A) =
3.6 9
4
( A) = cos − 1  
9
Answer: (A)

Solution Manual for Mathematics For JEE (Main & Advanced), Trigonometry Vector
Algebra Probability, Vol 2,
Copyright©2017 Wiley India Pvt. Ltd. All rights reserved
     
17. The vector a + 3b is perpendicular to the vector 7a − 5b and the vector a − 4b is
   
perpendicular to the vector 7a − 2b . Then the angle between a and b is

2π π 3π π
(A) (B) (C) (D)
3 3 4 4

Solution
   
(a + 3b ) ⋅ (7 a − 5b ) = 0
2 2  
7| a | − 15| b | +16 a ⋅ b = 0
7 − 15 + 16 cos θ = 0
d d
(Here we letting ‘θ ’ is angle between a and b )
−8 + 16 cos( θ ) = 0
8 π
=cos θ = , θ
16 3
Answer: (B)
   
18. a and b are two vectors which include angle 120°. If a + λ b is orthogonal to the
   
vectors a − b and | a |= (1/ 2) | b |, then the value of λ is

4 3 2 1
(A) (B) (C) (D)
5 5 5 5

Solution
   
(a + λ b)(a − b) = 0
 2   
| a | + λ a ⋅b − λ | b | =0
  2 
| a | + λ | a | (−1) − λ 4| a |2 =
0
− 5λ + 2 = 0,
2
λ=
5
Answer: (C)
            
19. If a + b +=
c 0, =
| a | 3, =
| b | 1 and | c |= 4, then the value of a ⋅ b + b ⋅ c + c ⋅ a is
equal to

(A) 12 (B) −12 (C) 13 (D) −13

Solution
       
(a + b + c )2= | a |2 + | b |2 + | c |2 +2 ε a ⋅ b
 
0 = (3) 2 + (1) 2 + (4) 2 + 2 ε a ⋅ b
     
a ⋅ b + b ⋅ c + c ⋅ a = −13
Answer: (D)

      
20. The angle between the vectors (a ⋅ c )b − (a ⋅ b )c and a is

Solution Manual for Mathematics For JEE (Main & Advanced), Trigonometry Vector
Algebra Probability, Vol 2,
Copyright©2017 Wiley India Pvt. Ltd. All rights reserved
π π π 5π
(A) (B) (C) (D)
4 2 3 12

Solution
     
( a ⋅ c ) b − (a ⋅ b ) ⋅ c
    
=a (b × c ) =a ⋅ (A vector along a )
=0
π
=
cos(θ ) 0,=θ
2
Answer: (B)

21. ABCD is a quadrilateral and R is the radius of the circle circumscribing the
 
quadrilateral. If | AB |2 + | CD |2 =
4 R 2 , then the angle between the diagonals AC and
BD is

π 5π 2π π
(A) (B) (C) (D)
2 12 3 3

Solution

Let A is origin.
  
= AB + BC
AC
d dd d d d
=
Let AB b= , AC c= , AD d
d 2 d 2
Now, ( AB) + (CD) = 4R2
dd d
| a | + (d − c )2 = 4R2
ddd dd
| a |2 + | d |2 + | c |2 − 2c ⋅ d =
4R2
d d d dd
Now, AC ⋅ BD =c ⋅ (d − b )
dd dd
= c ⋅ d −c ⋅b = 0
Hence, AC ⊥ BD.
Answer: (A)

22. In ∆ABC, a line is drawn parallel to the side AB meeting in AC in E and BE in F. If


AF = BE, then ∆ABC is

(A) right angled

(B) right-angled isosceles

Solution Manual for Mathematics For JEE (Main & Advanced), Trigonometry Vector
Algebra Probability, Vol 2,
Copyright©2017 Wiley India Pvt. Ltd. All rights reserved
(C) isosceles

(D) equilateral

Solution

4t is given that AF = BE.


So, from S–S−S Theorem,
AE = BF
Now, AB is parallel to EF
so, CE = CF,
Hence AC = BC, triangle is isosceles.
Answer: (C)

23. The area of ∆ABC is 6 square units. K is a point on the side AB is such that
AK:KB = 2:3. L is a point on the side AC is such that AL:LC = 5:3. P is the point of
intersection of the lines BL and CK inside the triangle. If the distance of P from the
side of AB is 3/2 units, then the length of the side AB is equal to

(A) 6 (B) 5 (C) 4 (D) 3

<solution missing>

24. OAB is a triangle right angled at the vertex O. Squares OALM and OBPQ on the sides
OA and OB, respectively, are constructed externally. The lines AP and BL meet in H.
 
Then the angle between OH and AB is

(A) 90° (B) 60° (C) 45° (D) 75°

Solution

Solution Manual for Mathematics For JEE (Main & Advanced), Trigonometry Vector
Algebra Probability, Vol 2,
Copyright©2017 Wiley India Pvt. Ltd. All rights reserved
   
=
Let OA a= , OB b
  
AB= b − a
Now, since OA is parallel to LM and PQ is parallel to OB point of intersection BL and AP
gives H. OH is ⊥ AB.
Answer: (A)
     
25. a , b and c are unit vectors such that a is perpendicular to both b and c . If the
    
angle between b and c is π / 3, then the value of | a + b + c | is

5 7
(A) 2 (B) 3 (C) (D)
2 2

Solution
       
| a + b + c |2= | a |2 + | b |2 + | c |2 +2ε a ⋅ b
 π
= 3 + 2 ⋅ 1 ⋅ 1 ⋅ cos  = 4
 3
  
a +b +c = 2.
Answer: (A)
     
26. Let i + 2 j + 3k and 2i − j − k be the position vectors of the points A and B,
   
respectively. Then the projection of AB on the vector i + j + k is

   7   
(A) 2(i + j + k ) (B) − (i + j + k )
3

7      
(C) (i + j + k ) (D) −2(i + j + k )
3

Solution

Solution Manual for Mathematics For JEE (Main & Advanced), Trigonometry Vector
Algebra Probability, Vol 2,
Copyright©2017 Wiley India Pvt. Ltd. All rights reserved
      
AB= (2i − j − k ) − (i + 2 j + 3k )
  
= (i − 3 j − 4 k )
       
AB ⋅ (i + j + k )= 12 + 12 + 12 (Proj of AB on i + j + k )
     
(i − 3 j − 4 k ) (i + j + k )
Proj. = 2
= −2
3
  
Hence proj. vector = − 2 (i + j + k )
Answer: (D)
      
27. AB = 3i − 2 j + 2k and BC= i − 2k represent the adjacent sides of a parallelogram.
Then the acute angle between the diagonals is

π 3
(A) (B) cos −1
6 10

3 2
(C) cos −1 (D) cos −1
5 5

Solution

  


= AB + BC
AC
    
= (3i − 2 j + 2k ) + (i − 2k )
  
AC= 4i − 2 j
     
BD =(i − 2k ) − (3i − 2 j + 2k )
  
= − 2i + 2 j − 4k
 
Now, BD ⋅ BD
    
= (4i − 2 j ) ⋅ (− 2i + 2 j − 4k =) 20 ⋅ 24 ⋅ cos
−8− 4 −3 − 3
=
cos(θ) = =
4 30 30 2
θ 150°
=
π
Acute angle =
6
Answer: (A)

28. Let Π be a plane and P a point not in Π . Q is the foot of the perpendicular drawn
from P onto Π . QM is drawn perpendicular to a line L in Π . Then the angle between
the lines PM and L is

Solution Manual for Mathematics For JEE (Main & Advanced), Trigonometry Vector
Algebra Probability, Vol 2,
Copyright©2017 Wiley India Pvt. Ltd. All rights reserved
π π
(A) (B)
2 3

 3  2
(C) cos −1   (D) cos −1  
5  3 

(This result is called “Theorem of three perpendiculars” in solid geometry.)

<solution missing>
            
29. Let a = i + j + k , b = 2i + 4 j − 5k , c = xi + 2 j + 3k . If the scalar product of a with a
 
unit vector parallel to b + c is unity, then the value of x is equal to

(A) 5 (B) −1 (C) 3 (D) 1

Solution
    
b + c = (2 + x) i + 6 J − 2k
 
Unit vector along b + c =
  
(2 + x) i + 6 J − 2k
(2 + x)2 + (6)2 + (−2)2

Now, scalar product with a
1
[(2 + x) + 6 − 2] =
1
(2 + x) 2 + 40
( x + 6) 2 =(2 + x) 2 + 40
12 x + 36 = 4 x + 44
x =1
Answer: (D)

        
30. Let a = 3i + j − 2k and b =i − 3 j + 4k . Then the area of the parallelogram with a

and b as diagonal vectors is

(A) 3 5 (B) 3 (C) 5 3 (D) 5

Solution
Area of parallelogram
1
A= Vector product of diagonal
2
1      
= (3i + j − 2k ) × (i − 3 j + 4k )
2
= 5 3
Answer: (C)

31. Let A (2, 3, 5), B (−1, 3, 2) and C (x, 5, y) be the vertices of ∆ABC. If the median AD
is equally inclined to the coordinate axes, then

Solution Manual for Mathematics For JEE (Main & Advanced), Trigonometry Vector
Algebra Probability, Vol 2,
Copyright©2017 Wiley India Pvt. Ltd. All rights reserved
= =
(A) x –10, y 7 =
(B) =
x –7, y –10

(C)=x 7,=y 10 =
(D) =
x 10, y 7

Solution
 ( x − 1) y + 2
Position vector of D =  , 4,
 2 2 
  5 − x 8− y 
=
 AD , −1,
 2 2 

As AD is equally included to the 3-axis
so,
 5− x 8− y 
 2 , −1, 2  =(−1, −1, −1)
 
5− x
= −1, x = 7,
2
8− y
= −1, y = 10
2
Answer: (C)
                   
32. Let a = 2i − 3 j + 4k , b = i + 2 j − 2k , c = 3i − j + k and x = a + b + c , y = a − b + c
      
and z = a + b − c . Then the volume of the parallelopiped with x , y and z as
coterminus edges is

(A) 24 (B) 6 (C) 28 (D) 7

(Hint: Use part (2) of Quick Look 11.)

Solution
   
x = 6i − 2 j + 3k
   
y = 4i − 6 j + 7 k
  
z =0 − 2 j + k
6 −2 3
Volume v = 4 −6 7 = 6(+8) + 2(4) + 3(−8) = 28
0 −2 1
Answer: (C)

33. Let
           
a = λ (i + 2 j − 3k ), b = (2λ + 1)i + (2λ + 3) j + (λ + 1)k , c = (3λ + 5)i + (λ + 5) j + (λ + 2)k .
  
Then a , b , c are

(A) coplanar for all real values of λ

(B) non-coplanar for all real values of λ

(C) non-coplanar for all non-zero real values of λ

(D) coplanar for all non-zero real values of λ

Solution Manual for Mathematics For JEE (Main & Advanced), Trigonometry Vector
Algebra Probability, Vol 2,
Copyright©2017 Wiley India Pvt. Ltd. All rights reserved
Solution:

λ 2λ −3λ
2λ + 1 2λ + 3 λ + 1 ≠ 0
3λ + 5 λ + 5 λ + 2
λ[(2λ + 3)(λ + 2) − (λ + 5)(λ + 1)] + 2λ[(λ + 1)(3λ + 5) − (2λ + 1)(λ − 3λ )] − 3λ[(2λ + 1)(λ + 5) − (2λ + 3)(3λ + 5)] ≠ 0
λ ≠ 0 λ ∈R

Answer: (C)
      
34. b and c are non-collinear vectors. a is a vector such that a ⋅ b + a ⋅ c =4 and

    
a × (b × c ) = ( p 2 − 2 p + 6)b + (sin q)c

where 0 < q ≤ π . Then the point (p, q) is

π   π
(A) (1, π) (B)  , 1 (C) (1, 1) (D) 1, 
2   2

Solution
  
a.b ⋅+ a ⋅ c = 4
        
a × (b × c ) = (a ⋅ b ) ⋅ c + (a ⋅ c )b
 
= ( p 2 − 2 p + 6)b + (sin q)c
 
a⋅b = sin q
 
a ⋅ c = p2 − 2 p + 6
4= p 2 − 2 p + 6 + sin q
p2 − 2 p + 6 =3

p2 − 2 p + 3 =0
p
=
p 1,=
q
2
Answer: (D)
   
35. Let L be the line of intersection of the planes r ⋅ (3i + 3 j + 2k ) =
0 and
   
r ⋅ (i + 2 j + 3k ) =
0, then the angle between the line L and the unit vector along the
positive X-axis is

 5   8 
(A) cos −1   (B) cos −1  
 83   83 

 7   7 
(C) π − cos −1   (D) cos −1  
 83   83 

Solution

Solution Manual for Mathematics For JEE (Main & Advanced), Trigonometry Vector
Algebra Probability, Vol 2,
Copyright©2017 Wiley India Pvt. Ltd. All rights reserved
i j k
3 3 2 = i (5) + j (−7) + k (3)
1 2 3
Angle along x – axis is
 5  −1  5 
cos −1   = cos  
 2  83 
 5 +7 +3
2 2

Answer: (A)

         
36. Let a = i + j − k and b= i + 3k . If c is a unit vector such that [a b c ] is maximum,

then c can be

1    1   
(A) (3i + 4 j + k ) (B) (3i − 4 j − k )
26 26

1    1   
(C) (4i + 3 j − k ) (D) (i − 3 j + 4 k )
26 26

<solution missing>
    
37. Let p = p1i + p2 j + p3 k where p1 , p2 and p3 are rational numbers and | p | is also
     
rational. If p makes an angle π / 4 with the vector a = 2i + 3 2 j + 4k , then p lies
in

(A) XY-plane

(B) ZX-plane

(C) YZ-plane
 
(D) along the bisector of the angle between k and − a.

Solution
2 p1 + 3 2 p2 + 4 p3
cos(45°) =
| p |2 ⋅6
1 2 p1 + 3 2 p2 + 4 p3
=
2 6| p|
3 | p |=p1 + 3 p2 + 2 2 p3

As p is rational, so p3 terms = 0

Hence p lies in xy-plane.

Answer: (A)
  
38. a , b , c are non-coplanar vectors and

Solution Manual for Mathematics For JEE (Main & Advanced), Trigonometry Vector
Algebra Probability, Vol 2,
Copyright©2017 Wiley India Pvt. Ltd. All rights reserved
      
p = x(b × c ) + y (c × a ) + z (a × b )

   
where x, y, z are scalars. If p is perpendicular to the vector a + b + c , then

(A) xy + yz + zx > 0 (B) x3 + y 3 + z 3 =


3xyz

(C) x= y= z (D) x 2 + y 2 + z 2 =
xyz

Solution
  
As a , b and c are non-coplanar vectors
so,
        
b ×=
c a, c × =a b, a ×=
b c
   
p =xa + yb + zc
   
Now, given that p is ⊥ to a + b + c
Hence x + y + z = 0
so, x3 + y 3 + z 3 − 3xyz =
0
x3 + y 3 + z 3 =
3xyz
Answer: (B)
          
39. If a and b are orthogonal vectors, then {[(a × b ) × a ] × a} × a − | a |2 (a ⋅ b )a is equal to

   
(A) − | a |4 b (B) | a |4 b

    
(C) | a |2 | b |2 a (D) | b |4 a

Solution
        
(a × b ) × a = (a ⋅ a ) ⋅ b − (a ⋅ b ) ⋅ a
 
=| a |2 b
Now,
       
| a |2 b × a =| a |3 ⋅b × a =− | a |4 ⋅b
Answer: (A)
     
40. If a and b non-collinear unit vectors, then | (a − b ) × (a + b ) | is equal to

   
(A) 2 1 − (a ⋅ b )2 (B) 2 − (a ⋅ b )2

   
(C) 4 − (a ⋅ b )2 (D) 4 − 2(a ⋅ b ) 2

Solution
We have
         
| (a − b ) × (a + b ) | = | (a × (a + b ) − b × (a + b ) |
   
= 2 a × b = 4 (1 − (a ⋅ b )2
 
= 2 1− ( a ⋅ b )2
Answer: (A)
Solution Manual for Mathematics For JEE (Main & Advanced), Trigonometry Vector
Algebra Probability, Vol 2,
Copyright©2017 Wiley India Pvt. Ltd. All rights reserved
dd d  
41. Let b , c , d be three non-coplanar vectors. Then for any vector a ≠ 0, the vector
dddddd
dddddd
(a × b ) × (c × d ) + (a × c ) × (d × b ) + (a × d ) × (b × c ) is equal to
dddd dddd
(A) 2[b c d ]a (B) 2[c b d ]a
 dddd
(C) 0 (D) [a b d ]c

Solution
We have
dddddd
dddddd
(a × b ) × (c × d ) + (a × c ) × (d × b ) + (a × d ) × (b × c )
dd d
[acd ]b − [bcd ]a + [add ]c − [cdb]a + [abc]d
−[dbc]a
d
= 2[cbd ]a
Answer: (B)
   
42. If the volume of tetrahedron having coterminus edges −12i + λ k , 3 j − k and
  
2i + j − 15k is 91, then one of the values of λ is

(A) −6 (B) 5 (C) 4 (D) −3

Solution
−12 0 λ
1
=
Volume 0 =3 −1 41
6
2 1 −15
⇒ 528 + λ (−6) = 91 ∗ 6
6λ =528 − 546 = −18
−18
λ= = −3
6
Answer: (D)
  
43. Let a , b and c be non-coplanar vectors and
     
 b × c  c × a  a×b
=a′ = 
  , b ′ = 
  , c ′   
[a b c ] [a b c ] [a b c ]

        
Then [a b c ](a′ × b′ + b′ × c′ + c′ × a′) equals
   
(A) 0 (B) a + b + c
       
(C) [a b c ](a + b + c ) (D) 3(a + b + c )

Solution

Solution Manual for Mathematics For JEE (Main & Advanced), Trigonometry Vector
Algebra Probability, Vol 2,
Copyright©2017 Wiley India Pvt. Ltd. All rights reserved
       
[ab c ][a × b + b × c + c × a ]
           
[(b × c ) × (c × a ) + (c × a ) × (a × b ) + (a × b ) × (b × c )]
 
[abc ]
    
[a + b + c ][ab c ]   
=   = [a + b + c ]
[abc ]
Answer: (B)

44. On the sides AB and BC of ∆ABC, points P and Q are taken, respectively, such that
AP:PB = 2:3 and CQ:QB = 3:2. The lines AQ and CP intersect in R. If the area of
∆BRC is 2 square units, then the area of ∆ABC is

9 7 17 19
(A) (B) (C) (D)
19 4 4 9

<solution missing>
              
45. Let a= i + j and b= j + k . r is a vector such that r × a = b × a and r × b = a × b .

Then r is equal to
     
(A) i + j − 2k (B) i − j + 2k
     
(C) i − j + k (D) i + 2 j + k

Solution
   
r =xi + y j + z k
   
r ×a =b×a
        
b × a =( j + k ) × (i + j ) =−k + j − i
        
a × b = (i + j ) × ( j + k ) = k − j + i
      
r × a= ( x i + y j + z k ) × (i + j )
     
=( x − y ) k + z j − z i =−k + j − i
From, z =
1, x − y =−1
       
(x i + y j + z k ) × ( j + k ) = k − j + i
     
xk − xj + ( y − z ) i = k − j + i
x = 1, y − z = 1, z = 1, y = z
   
r =i + 2 j + k

Answer: (D)

MULTIPLE CORRECT CHOICE TYPE QUESTIONS

1. Let A (−1, 3, 7), B (2, −1, 0) and C (0, 1, −5) be three points. Then,

(A) | AB |= 74
   
(B) (2 AB − CB) ⋅ (2 BC + BA) =−19

Solution Manual for Mathematics For JEE (Main & Advanced), Trigonometry Vector
Algebra Probability, Vol 2,
Copyright©2017 Wiley India Pvt. Ltd. All rights reserved

(C) | BC |= 33
   
(D) ( AB − CB) ⋅ ( BC + AB) =
19

Solution

(A) We have
=
AB (2 + 1)2 + (−1 − 3)2 + (0 − 7)=
2
74
(B) (2 AB + BC ) ⋅ (2 BC − AB)
= 4 AB ⋅ BC + 2 | BC |2 − BC ⋅ AB − 2 | AB |2
= −19
(C) | BC=| (2) + 22 + 5=
2 2
33
Answer: (A), (B), (C)
   
2. If the vectors a (3, x, − 1) and b = (2, 1, y ) are orthogonal vectors and | a |=| b |, then

31 41
(A) x = (B) y = −
12 12

31 41
(C) x = − (D) y =
12 12

Solution
 
a ⋅b =0
6+ x− y =0
y−x= 6
d d
and | a | =| b |, x 2 + 10 =5 + y 2
y 2 − x2 = 5
5
( x + y) =
6
x− y= −6
5 − 36 − 31 41
=x = = ,y
12 12 12
Answer: (C), (D)
     
3. Let a , b be two vectors such= that | a | 2=
2, | b | 3 and the angle between a and b
  
is π / 4. Then, the lengths of the diagonals constructed with =p 5a + 2b and
  
q= a − 3b as adjacent sides of a parallelogram and its area are

(A) 15 (B) 693

(C) 593 (D) 120

Solution

Solution Manual for Mathematics For JEE (Main & Advanced), Trigonometry Vector
Algebra Probability, Vol 2,
Copyright©2017 Wiley India Pvt. Ltd. All rights reserved
    
AC =AB + BC =P + Q
 
= 6a − b
    
BD = AD − AB =− 4a − 5b
1
Now area = Product of diagonals
2
1    
| (6a − b ) × (−4a − 5b ) |
2
1    
| −30 a × b − 4 a × b |
2
102

Answer: (A), (C), (D)

       
4. Consider the parallelopiped with OA = i + j + k , OB = 2i + 4 j − k and
   
OC = i + j + 3k as edges. Then
  
(A) [OA OB OC ] = −4
  
(B) [OA OB OC ] = 4
(C) the length of the altitude CM drawn from the corner C onto the base of
 
parallelogram having OA and OB as adjacent sides is 4 / 38
2
(D) CM =
3
Solution
(B) Volume of parallelepiped
1 1 1
= 2 4=−1 4
1 1 3

(C) Length of altitude CM drawn from

Solution Manual for Mathematics For JEE (Main & Advanced), Trigonometry Vector
Algebra Probability, Vol 2,
Copyright©2017 Wiley India Pvt. Ltd. All rights reserved
  
OA × OB = −5i + 3 j + 2 k
Perpendicular and distance
     
(−5i + 3 j + 2k ) ⋅ ( i + j + 3 k )
CM =
−(5) 2 + (3) 2 + (2) 2
−5 + 3 + 6 4
= =
38 38

Answer: (B), (C)


            dd dd
5. Let a = i + j + k , b = i − j + k , c = i + j − k and d =−i + j + k . Then

dd dd d
(A) (a × b ) × (c × d ) =−8 j

dd ddd
(B) (a × c ) × (b × d ) =
8k

dd dd
(C) (a × b ) ⋅ (c × d ) =
0

dd dd
(D) (a × c ) ⋅ (b × d ) =
0

Solution
(A)
We have
i j k
d dd d
a × b = 1 1 1 = 2i − 2 k
1 −1 1
d dd 1 1 −1 d
c×d = = 2i + 2 k
−1 1 1
dd dd d
(a × b ) × (c × d ) =−8 j
(B)
dd 1 1 1 dd
a×c = =−2i + 2 j
1 1 −1
dd 1 −1 1 dd
b×d = = −2i − 2 j
−1 1 1
dd ddd
(a × c ) × (b × d ) =8k

(C)
We have
dd dd
( a × b ) × (c × d ) =
0
(D)
We have
d ddd
(a × b ) × (b × d ) =
0

Answer: (A), (B), (C), (D)


   d
6. Let a , b , c and d be four vectors. Then

Solution Manual for Mathematics For JEE (Main & Advanced), Trigonometry Vector
Algebra Probability, Vol 2,
Copyright©2017 Wiley India Pvt. Ltd. All rights reserved
dddd
dddddd
(A) [(a × b ) × (a × =
c )] ⋅ d [a b c ](d ⋅ a )

dddddddddd
(B) [(a × b ) × (b × c=
)] ⋅ d [a b c ](b ⋅ d )

ddddddd dddddddd
(C) [(a × c ) × (b × =
d )] ⋅ d [a c d ](b ⋅ d ) + [a b c ](c ⋅ d )

dddddddddd
(D) [(a × d ) × (c × =
d )] ⋅ d [a c d ](d ⋅ d )

Solution
We have
dddd
dddddd
(A) [(a × b ) × (a × c )] ⋅ d = [a ⋅ b ⋅ c ](d ⋅ a )
dddddddddd
(B) [(a × b ) × (b × c )] d = [a b c ](b d )
dddddd
dddd
(D) [(a × d ) × (c × d= )] d [a cd ](d ⋅ d )
This is direct from theorem

Answer: (A), (B), (D)

7. Consider the four points A (0, 1, 2), B (3, 0, 1), C (4, 3, 6) and D (2, 3, 2). Then,
(A) area of ∆ABC is 3 10
(B) the altitude drawn from A on to the side BC has magnitude 6 / 7 14
(C) volume of the tetrahedron ABCD is 6
(D) the shortest distance between the lines AB and CD is 6
Solution
(A)
AB = (3, −1, −1), BC = (1,3,5)
1
Area of ∆ABC = | AB × BC |
2
1
= | (− 2, −16,10) |
2
1
= × 6 10 = 3 10
2
(C) AB = (3, −1, −1), AD = (2, 2,0)
Volume of tetrahedron
1
=
v | AB × AD=| 6
12
(D) ⊥ distance between AB and CD = 6

Answer: (A), (C), (D)

8. Consider the line L whose equation is

x − 1 y + 1 z + 10
= =
2 −3 3

Which of the following are correct?

(A) The direction cosines of L are (2, −3, 8).

Solution Manual for Mathematics For JEE (Main & Advanced), Trigonometry Vector
Algebra Probability, Vol 2,
Copyright©2017 Wiley India Pvt. Ltd. All rights reserved
(B) The foot of the perpendicular drawn from the point P (1, 0, 0) onto the line is (3,
−4, −2).
(C) The image of the point P (1, 0, 0) in the line L is (5, −8, −4).
(D) The distance of the point P (1, 0, 0) from the line L is 2 6.
Solution

(A) direction cosines = (2, −3, 8)


(B) Foot of perpendicular = 2 ∗1 + 1, − 3 ∗1, 8 ∗1 −10
= (3, −4, −2)
(C) The single point
= (2 ∗ 3 − 1, −4 ∗ 2, −2 ∗ 2)
= (5, −8, −4)
(E) Distance D = (2) 2 + (4) 2 + (2) 2 =24
=2 6
Answer: (A), (B), (C), (D)
9. Let a, b, c be non-zero real numbers such that a + b = 0 and c = 2b. Then,
       
(A) the vectors p = ai + bj + ck and r =−i + j + 2k are collinear vectors
    
(B) the number of vectors p = ai + bj + ck such that | p | is equal to 6 is infinite
   
(C) the magnitude of p = ai + bj + ck is 2 6 if 2a + 3b + 6c = 26
       
(D) the vectors p = ai + bj + ck and q = 4i + 2 j + k are perpendicular to each other
Solution
(A)
   
p= − bi + bj + 2bk
   
= b( − i + j + 2k ) = b( r )
 
Hence, p and r are collinear
(D)
   
p = b( −i + j + 2k )
       
p ⋅ q = b(−i + j + 2k ) ⋅ (4i + 2 j + k )
b(−4 + 2 + 2) =0
 
Hence p ⊥ q
Answer: (A), (D)

10. Let A, B and C be the points (1, 1, 1), (1, −1, 1) and (−1, −3, −5), respectively. Then,
(A) equation of the plane ABC is 3 x – z – 2 = 0
 
(B) the vector 3i − k is normal to the plane ABC
 3 19 
(C) foot of the perpendicular drawn from the point (1, 1, 2) is  , 1, 
 10 10 
(D) image of (1, 1, 2) in the plane ABC is (1, 1, 3)
Solution
x −1 z −1
(A) = ⇒ 3x − =z−2 0
1 − (−1) 1 − (−5)
 
(B) Equation of plane of ABC = 3i − k = 0
   
Now (3i − k ) × (3i − k ) =
0

Solution Manual for Mathematics For JEE (Main & Advanced), Trigonometry Vector
Algebra Probability, Vol 2,
Copyright©2017 Wiley India Pvt. Ltd. All rights reserved
 
Hence 3i − k is normal
(C) The food of the perpendicular
 3 19 
= , 
 10 10 

Answer: (A), (B), (C)

MATRIX-MATCH TYPE QUESTIONS


In each of the following questions, statements are given in two columns, which have to be
matched. The statements in column I are labeled as (A), (B), (C) and (D), while those in
column II are labeled as (p), (q), (r), (s) and (t). Any given statement in column I can have
correct matching with one or more statements in column II. The appropriate bubbles
corresponding to the answers to these questions have to be darkened as illustrated in the
following example.

Example: If the correct matches are (A) → (p), (s) , (B) → (q), (s), (t), (C) → (r), (D) → (r), (t) ,
that is if the matches are (A) → (p) and (s); (B) → (q), (s) and (t); (C) → (r); and (D) → (r), (t),
then the correct darkening of bubbles will look as follows:

1. Match the items of Column I with those of Column II.

Column I Column II

(A) In the rectangular Cartesian coordinate (p) 1


plane A and B are points on the curve xy
   
= 6 such that OA ⋅ i =−2 and OB ⋅ i = 3
where O is the origin of the coordinate
 
axes. Then | 2OA + 3OB | is equal to

 
(B) a = (2, 3, − 5), b = (3, 0, 1) and (q) –1
 dd dd
c (4, − 3, 2). If d = 3a + b − c , then
=
1 d
| d | is
17

Solution Manual for Mathematics For JEE (Main & Advanced), Trigonometry Vector
Algebra Probability, Vol 2,
Copyright©2017 Wiley India Pvt. Ltd. All rights reserved
(C) A (–3, –2, 0), B (3, –3, 1) and C (5, 0, 2) (r) 5
are three consecutive vertices of a
parallelogram. Then the sum of the
coordinates of its fourth vertex is

 
(D) If= =
a ( x, − 2, 1) and b ( x, x, − 3) (s) –3
are orthogonal vectors, then possible
values of x are

(t) 3

Solution
  
(A) | 2(−2)i + 3(3) i |= | 5i |= 5

Answer: (r)
d
(B) =
d 3(2,3, −5) + (3,0,1) − (4, −3, 2)
= (5,12, −16, )
Now,
d
| d | 5 17
= = 5
17 17

Answer: (r)

(C) Mid-point of diagonal AC


 −3 + 5 −2 + 0 0 + 2 
=
2 
, ,
 2 2
= (1, − 1,1)

Now, let D = (x, y, z)


So,
 x + 3 y − 3 z +1
 2 , 2 , 2 =  (1, −1,1)
 
( x, y, z ) = (−1, 1,1)
x+ y+z= 1

Answer: (p)

(D)
x2 − 2 x − 3 =0
x 2 − 3x + x − 3 =0
x= 3, −1

Solution Manual for Mathematics For JEE (Main & Advanced), Trigonometry Vector
Algebra Probability, Vol 2,
Copyright©2017 Wiley India Pvt. Ltd. All rights reserved
Answer: (q), (t)

2. Match the items of Column I with those of Column II.

Column I Column II
     
π
(A) If a + b +=c 0, = | a | 3, =
| b | 5 and (p)
   2
| c |= 7 then the angle between a and b
is

 
(B) If a and b are unit vectors such that (q) π
  4
| a − b | is also unit vector, then the angle
 
between a and b is

(C) In a regular tetrahedron (i.e., all edges are (r) π


equal) the angle between opposite pair of 6
edges is

 
(D) If a and b are two non-collinear vectors (s) π
    3
such that | a + b |=| a − b |, then the angle
 
between a and b is

Solution
(A) Let
       
a= x, i + y , j , b = x2 i + y2 j , c =x3i + y3 j
Now, ( x1 + x2 + x3 )i + j ( y1 + y2 + y3 ) = 0
( x1 + x2 ) 2 =
(− x3 ) and ( y1 + y2 ) 2 =
(− y3 ) 2
2 x1 x2 + 2 y, y2 = x 23 − ( x 21 + x 2 2 ) + y32 − ( y12 + y2 2 )
= 49 − 9 − 25 = 15
15
x1 x2 + y1 y2 =
2
 
Angle between a and b =
15 / 2
3∗5
1 π
=
cos θ = ,θ
2 3
Answer: (s)

 
(B) a ( x1 y1 ), b = ( x2 y2 )

Solution Manual for Mathematics For JEE (Main & Advanced), Trigonometry Vector
Algebra Probability, Vol 2,
Copyright©2017 Wiley India Pvt. Ltd. All rights reserved
Now, x12 +=
y12 1 and x2 2 +=
y2 2 1,
x1 y2 + y1 y2 1 π
cos θ , θ=
x12 y12 2
x2 y2 2 2 3
Answer: (s)

360°
(C) The angle between opposite edge = = 90°
4∗2
Answer: (p)

(D) ( x1 + y2 )2 + ( y1 + y2 )2 = ( x1 − x2 )2 + ( y1 − y2 )2
x1 x2 + y1 y2 =
0
  π
Angle between a and b =
2
Answer: (p)

3. Match the items of Column I with those of Column II.

Column I Column II

(A) The line p) 2

x −1 y − 2 z + 3
= =
2 3 4

meets the plane 2 x + 4 y − z − 1 =0 in


the point (a, b, c), then the value of a
+ b + c is

(B) The square of the reciprocal of the q) 3


distance of the point (1, –2, 3) from
the plane x − y + z − 5 = 0 is

(C) The distance of the line r) 7

x −1 y + 2 z −1
= =
3 −2 2

from the plane 2 x + 2 y − z =0 is

(D) Let V be the volume of a tetrahedron σ s) 1


and V′ be the volume of the
tetrahedron

Solution Manual for Mathematics For JEE (Main & Advanced), Trigonometry Vector
Algebra Probability, Vol 2,
Copyright©2017 Wiley India Pvt. Ltd. All rights reserved
formed by the centroids of the faces of
σ, then V / 9V ′ is equal to

<solution missing>
    
4. Let a , b , c be three vectors such that [a b c ] = 2. Match the items of Column I with
those of Column II.

Column I Column II
    
(A) The value of [a + b b + c c + a ] is (p) 2

    
(B) The value of [b × c c × a a × b ] equals (q) 6

    
(C) [(a − b ) × (a + b )] ⋅ c is equal to (r) 4

(D)       (s) 8
a ⋅a a ⋅b a ⋅c
     
b ⋅ a b ⋅ b b ⋅ c equals
     
c ⋅a c ⋅b c ⋅c

(t) 12

Solution
       2
(A) [a + b b + c c +=
a ] [a b =
c] 4

Answer: (r)
         
(B) [a × b c × a a × b ] = (a × b ) × (a × c )

= [a= b c] 4

Answer: (r)
    
(C) [(a − b ) × (a + b )]c
 
= [a=b c ][a b c ] 4

Answer: (r)
      
(D) ε (a )2 (b × c − b ⋅ c × c ⋅ b ) =4

Answer: (r)

COMPREHENSION TYPE QUESTIONS

Solution Manual for Mathematics For JEE (Main & Advanced), Trigonometry Vector
Algebra Probability, Vol 2,
Copyright©2017 Wiley India Pvt. Ltd. All rights reserved
        
1. Passage: Let a =
i − j, b =j − k and c= k − i . Answer the following three
questions.
dd ddd d d
(i) If a ⋅ d = 0 = [b c d ] and | d |= 1, then d equals

1    1   
(A) ± (i + j − 2 k ) (B) ± (i + j − k )
6 3

1    
(C) ± (i + j + k ) (D) ± k
3
   
(ii) (a × b ) × (a × c ) is equal to
    
(A) i + j + k (B) −2i + 2 j
  
(C) 2i + k (D) 0
    
(iii) [b × c c × a a × b ] is equal to

(A) 0 (B) 1 (C) 2 (D) 4

Solution
(i)
d d
D = xi + yj + zk , A ⋅ D = 0, x − y = 0
0 1 −1
and −1 2 1 = 0, x − z + y = 0, z = x + y
x y z
1
D= (i + j − 2ky )
6

Answer: (A)

(ii)
       
[(i − j ) × ( j − k )] × [(i − j ) × (k − i )]
     
= (k + j + i ) × (− j − i − k )= 0

Answer: (D)

(iii)
        
(i + k + j i + j + k i + j + k ) =
0

Answer: (A)
           
2. Passage: Let a = i + 2 j + k , b = i − j + k , c = i + j − k . Answer the following
questions.
 
(i) A vector coplanar with a and b whose projection is 1/ 3 is
     
(A) 4i − j + 4k (B) 3i + j − 3k

Solution Manual for Mathematics For JEE (Main & Advanced), Trigonometry Vector
Algebra Probability, Vol 2,
Copyright©2017 Wiley India Pvt. Ltd. All rights reserved
     
(C) 2i + j − 2k (D) 2i + j + 2k

    
a ⋅a a ⋅b a
    
(ii) b ⋅ a b ⋅ b b equals
    
c ⋅a c ⋅b c

   
(A) 9(i − k ) (B) 9(i − j )
   
(C) 18(i − k ) (D) 18(2 j − k )

(Hint: Use part (2) Quick Look 11.)


   
(iii) The area of the parallelogram for which a + b and b + c are adjacent
sides is

(A) 2 2 (B) 2 (C) 3 2 (D) 4 2

Solution
(i)
   
(i )r = (1 + λ )i + (2 − λ ) j + (1 + λ )k
 
r .c 1
 =
|c | 3
(1 + λ ) + 2 − λ − 1 − λ 1
= = ,λ 1
3 3
   
r = 2i + j + 2k

Answer: (D)

(ii)

6 0 a
   
0 3 b = −6(b + 3c ) + a (−6)

2 −1 c
    
=− 6(a + b + 3c ) = 18(i − k )

Answer: (C)

1     1  
(iii) | (a + b ) × (a − b=
)| | 4 j − 4=
k| 2 2
2 2

Answer: (A)
   dd dd
3. Passage: If a , b , c are three non-coplanar vectors and d = xa + yb + zc , then
ddddd
dddd
[b c d ] [c a d ] [a b d ]
= x = ddd
dddddd ,y = ,z
[a b c ] [a b c ] [a b c ]

Solution Manual for Mathematics For JEE (Main & Advanced), Trigonometry Vector
Algebra Probability, Vol 2,
Copyright©2017 Wiley India Pvt. Ltd. All rights reserved
           
Now let a = 2i − j + k , b = i + 2 j − k and c = i + j − 2k . Answer the following three
questions
     
(i) If i + j + k = xa + yb + zc , then

3 9 7
(A) x = , y= , z= −
8 8 8

3 9 7
(B) x =
− , y= , z=

8 8 8

3 9 7
=
(C) x =,y = ,z
8 8 8

3 9 7
(D) x =
− , y=
− , z=
8 8 8
        
(ii) If i − j + k= x(b × c ) + y (c × a ) + z (a × b ), then

1 1 1
(A) x =
− , y=
− , z=
2 4 4

1 1 2
=
(B) x =,y = ,z
2 4 8

1 1 1
(C) x =
− , y= , z=
2 4 4

1 1 2
(D) x =
− , y=
− , z=
8 8 8
     
   (b × c ) (c × a ) (a × b )
(iii) j − k x    + y    + z    , then
i +=
[a b c ] [a b c ] [a b c ]

(A) x = 0, y = 2, z = −2 (B)=x 0,=y 4,=z 4

(C) x =
0, y =
−4, z =
−2 (D) x =
0, y =
−4, z =
4

Solution
(i)
2x + y + z =
1 (i)
− x+ 2y + z =1 (ii)
x − y − 2z =
1 (iii)
From Eqs. (ii) and (iii), y − z = 2 and 3x − z = 2
3 9 7
From that x = , y = , z =
8 8 8

Answer: (A)

Solution Manual for Mathematics For JEE (Main & Advanced), Trigonometry Vector
Algebra Probability, Vol 2,
Copyright©2017 Wiley India Pvt. Ltd. All rights reserved
(ii) Similarly we can find
1 1
x=
− , y= =
z
2 4

Answer: (C)

(iii) x = 0, y = z = 4

Answer: (B)
 
4. Passage: Let a and b be two non-zero non-collinear vectors. Then the projection
 
   b ⋅a  
vector of b on a is  | a |2  a and the vector component of b perpendicular to the

   b ⋅ a  
direction of a is b −   2  a. Answer the following three questions.
|a | 
  
(i) Let a , b , c be non-zero and non-coplanar vectors and

   b ⋅ a  
b1= b −   2  a
| a | 

   b ⋅ a  
b2= b +   2  a
| a | 
   
   c ⋅a    b ⋅c  
c1 =c −   2  a +   2  b1
| a |  |c | 
   
  c ⋅a    b ⋅c   
c2 =−   2  a −   2  b1 + c
|b | 
| a |   1 
   
   c ⋅a    b ⋅c  
c3 =c −   2  a +   2  b1
|c |  |c | 
   
   c ⋅a    b ⋅c  
c4= c −   2  a =   2  b1
|c |  |b | 

Then which of the following triads are orthogonal?


  
(A) (a , b1 , c3 )

  
(B) (a , b1 , c2 )

  
(C) (a , b1 , c1 )

  
(D) (a , b2 , c2 )

           
(ii) Let a = i + 2 j + k , b = i − j + k and c = i + j − k . Then the projection
  
vector of c on a × b is
 
(A) 2(i − k )

Solution Manual for Mathematics For JEE (Main & Advanced), Trigonometry Vector
Algebra Probability, Vol 2,
Copyright©2017 Wiley India Pvt. Ltd. All rights reserved
 
(B) 3 2(i + 2k )

2 2  
(C) (i − k )
3
 
(D) i − k
  
(iii) The component of a perpendicular to the direction of (b × c ) where
  
a , b and c are same as in (ii) is
  
(A) 2i + j − k

1   
(B) (2i + j − k )
2
 
(C) j − k

1  
(D) ( j − k)
2

Answer: B
(i) < solution missing>
  
(ii) a × b = 3i − 3k
 
   (3 + 3) ⋅ (i − k )
Projection of c on a × b =
6

Answer: (D)

 a ⋅ (b × c )   
(iii) −    2  ⋅ (b × c ) = ( D)
 (b × c ) 

    4 + 2 2   1   
(i + 2 j + k ) −   ( j + k=
) (2i + j + k )
 2  2

Answer: (B)

ASSERTION–REASONING TYPE QUESTIONS


In the following set of questions, a Statement I is given and a corresponding Statement II is
given just below it. Mark the correct answer as:

(A) Both Statements I and II are true and Statement II is a correct explanation Statement I
(B) Both Statements I and II are true but Statement II is not a correct explanation for
Statement I
(C) Statement I is true and Statement II is false
(D) Statement I is false and Statement II is true

Solution Manual for Mathematics For JEE (Main & Advanced), Trigonometry Vector
Algebra Probability, Vol 2,
Copyright©2017 Wiley India Pvt. Ltd. All rights reserved
           
1. Statement I: Let a = a1i + a2 j + a3 k , b = b1i + b2 j + b3 k , c = c1i + c2 j + c3 k where
3
ar , br , cr (r = 1, 2, 3) are non-negative real numbers and ∑ (ar + br + cr ) =
L. If V is
r =1
  
the volume of the parallelopiped with a , b and c as coterminus edges, then V ≤ L3 .

Statement II: The Arithmetic mean of non-negative real numbers is greater than or
equal to their Geometric mean.

Solution
Here A.M.≥G.M.
d ddddd dd
L = a + b + c and v = a × b × c
d
L d3
so, ≥ (v )
3
d3 d
( L ) ≥ v proved
Answer: (A)

2. Statement I: A, B, C and D are four points such that for any point P in the space
   
AP ⋅ CP ≠ BP ⋅ DP. Then ABCD is a parallelogram which cannot be a rectangle.

Statement II: If the points A, B, C and D are the vertices of a rectangle, then for any
       
point Q, QA ⋅ QC = QB ⋅ QD and | QA |2 + | QC |2 =| QB |2 + | QD |2

(Hint: See multiple correct choice type question 7 in Worked-Out Problems.)

Solution
In a rectangle ABCD, for any inside point Q dot product of its distance from opposite
vertex are always equal. Also its sum of square of its distance be equal.
Answer: (A)

3. Statement I: The angle at the vertex A of ∆ABC is π / 6. H is the orthocentre and M


is the mid-point of BC. If T is a point on the line HM produced such that HM = MT,
 
then | AT |= 2 | BC | .

Statement II: In ∆ABC, if O is the circumcentre and H is the orthocentre, then


   
OH = OA + OB + OC.

Solution
We know that vector distance of circumcenter and orthocentre is always equal to the sum
of circum radius from 3 vertex of the triangle.
Answer: (A)

4. Statement I: The sum of the squares of the sides of any quadrilateral equals the sum
of the squares of its diagonals together with four times the square of the distance
between the middle points of the diagonals.
       
Statement II: If a and b are two vectors, then | a − b |2= | a |2 −2(a ⋅ b )+ | b |2 .

Solution

Solution Manual for Mathematics For JEE (Main & Advanced), Trigonometry Vector
Algebra Probability, Vol 2,
Copyright©2017 Wiley India Pvt. Ltd. All rights reserved
This is from expansion of formula (a − b)2 = a2 − 2ab + b2. Here we take dot product of
   
(a − b ) ⋅ (a − b ).
Answer: (A)

5. Statement I: If the incident ray on a surface is along unit vector v , the reflected ray
 
along the unit vector w and the normal is along the unit ray a outward, then
    
w= v − 2(a ⋅ v )a.

Statement II: The incident vector, the reflected vector and the normal vector at the
point of incidence are coplanar and the angle of incidence is equal of reflection.

See the below figure.

Assertion–reasoning type question 5.

Solution
From reflection law of rays this is valid fully.
Answer: (A)

INTEGER ANSWER TYPE QUESTIONS


The answer to each of the questions in this section is a non-negative integer. The appropriate
bubbles below the respective question numbers have to be darkened. For example, as shown
in the figure, if the correct answer to the question number Y is 246, then the bubbles under Y
labeled as 2, 4, 6 are to be darkened.

Solution Manual for Mathematics For JEE (Main & Advanced), Trigonometry Vector
Algebra Probability, Vol 2,
Copyright©2017 Wiley India Pvt. Ltd. All rights reserved
X Y Z W
ⓞ ⓞ ⓞ ⓞ
① ① ① ①
② ② ② ②
③ ③ ③ ③
④ ④ ④ ④
⑤ ⑤ ⑤ ⑤
⑥ ⑥ ⑥ ⑥
⑦ ⑦ ⑦ ⑦
⑧ ⑧ ⑧ ⑧
⑨ ⑨ ⑨ ⑨

  
1. The volume of a parallelopiped with a , b , c as coterminus edges is 3. Then the
     
volume of the parallelopiped with a + b , b + c and c + a as coterminus edges is
____.

Solution
   
Let a =x1i + y, j + z1k
   
b =x2 i + y2 , j + z2 k
   
c =x3i + y3 , j + z3 k
x1 y1 z1

=
[a b c ] x=2 y2 z 2 3
x2 y3 z3
     
Now [a + b b + c c + a] =

x1 + x2 y1 + y2 z1 + z2
x2 + x3 y2 + y3 z2 + z3
x3 + x1 y3 + y1 z3 + z1
x1 y1 z1 x2 y2 z2
= x2 y2 z2 + x3 y3 z3 = 3 + 3 = 6
x3 y3 z3 x1 y1 z1

Answer: (6)
  
2. The volume of a parallelopiped with coterminus edges a , b , c is 3. Then the value of
the determinant
     
a ⋅a a ⋅b a ⋅c
     
b ⋅a b ⋅b b ⋅c
     
c ⋅a c ⋅b c ⋅c

is _____.

Solution
Solution Manual for Mathematics For JEE (Main & Advanced), Trigonometry Vector
Algebra Probability, Vol 2,
Copyright©2017 Wiley India Pvt. Ltd. All rights reserved
   
aa ab ac
   
[a b c] = 3, ba bb bc
   
ca cb cc
 
= [a b c ][a b c ] = 3 ⋅ 3 = 9
Answer: (9)
  
3. If e1 , e2 , e3 are mutually perpendicular unit vectors forming a right-handed system,
      
then the value of [e1 + e2 + e3 e1 + e2 e2 + e3 ] is _____.

Solution
     
Hence= c1 s=
, e2 j=
, e2 K
      
[i + j + k i + j j + k ]
1 1 1 
= =1 1 0  1
0 1 1 
Answer: (1)
        
4. If | a |= 2 and a ⋅ b =0, then a × {a × [a × (a × b )]} is nb where n is _____.

Solution
Here a⋅b = 0 a ⊥ b
     
Now, a ×[9 × a (a × b)]= | a |4 ⋅b

= (2)4 b ,
N = 16, n = 4.
Answer: (4)
          
5. Let a , b , c be three non-coplanar vectors and p =−
a 2b + 3c , q =−2a + 3b − 4c and
     
r =a − 3b + 5c . Then [ p q r ] equals _____.

Solution
 1 −1 3 
  
[ p q r ] = −2 3 −4  =1(3) − 2(6) + 3(3)
 1 −3 5 
=0
Answer: (0)

6. A (1, 1, 1), B (1, −1, 1) and C (0, 0, 1) are the vertices of a triangle. Then the area of
∆ABC in square units is ____.

Solution
0 0 1
1
1 −1 1 =
1
2
1 1 1
Answer: (1)

Solution Manual for Mathematics For JEE (Main & Advanced), Trigonometry Vector
Algebra Probability, Vol 2,
Copyright©2017 Wiley India Pvt. Ltd. All rights reserved
7. M and N are the mid-points of the sides AC and AB, respectively, of ∆ABC. Through
the vertex C, a line is drawn parallel to the side AB meeting the line BM in P. Then
the area of ∆ABC is equal to λ times the area of ∆PMN where λ equals _____.

Solution

Area of triangle
1
A= base × height
2
Let ΔABC is equilateral triangle
BC
So, MN = .
2
Also, height as = 2AQ.
1 1 BC AS
So, ∆PMN = ⋅ MN ⋅ AQ = ⋅ ⋅
2 2 2 2
∆ABC =
4∆PMN
Answer: (4)

8. D, E and F are points on the sides BC, CA and AB, respectively, such that
BD:DC = CE:EA = AF:FB = 2:1. Then m times the area of ∆DEF is equal to n times
the area of ∆ABC where m/n is equal to ____.

Solution

Let A as origin

 b + 2c
then, AD =
3

Solution Manual for Mathematics For JEE (Main & Advanced), Trigonometry Vector
Algebra Probability, Vol 2,
Copyright©2017 Wiley India Pvt. Ltd. All rights reserved
 c
AE =
3

 2b
AF =
3
1  
DDEF = OE × DF
2
   
1 − b − c b − 2c
= ×
2 3 3
1    
= 2
2(b × c ) + (1) 2 (b × c )
2(3)
22 − 2 + 1  
= b ×c
2*9
3 1
= (Area of DABC ) = DABC.
9 3
DABC
Hence =3
DDEF
Answer: (3)

9. If V is the volume of the tetrahedron whose vertices are (2, −1, −3), (4, 1, 3), (3, 2,
−1) and (1, 4, 2), then the integer part of V is _____.

<solution missing>
           
10. If the vectors a = i + j + k , b = 4i + 3 j + 4k , c = i + a j + b k are coplanar and

| c |= 3, then αβ2
+ is _____.

Solution
Here 1 + αβ
2
+ =
2
( 3)=
2
3
αβ
2
+ 2=
2
 
Also, as ab and c are columnar
1 1 1
So 4 3 4 = 0
1 αβ
4αββα
−3 +4−4 +4 −3=0
αβ
=
so, αβαβ
2
= 1,= 2 , = = 1
so, αβ
2
+ 2 = 12 + 1 = 2
Answer: (2)

<H1>Answers

<H2>Single Correct Choice Type Questions

Solution Manual for Mathematics For JEE (Main & Advanced), Trigonometry Vector
Algebra Probability, Vol 2,
Copyright©2017 Wiley India Pvt. Ltd. All rights reserved
1. (B) 24. (A)
2. (D) 25. (A)
3. (A) 26. (D)
4. (C) 27. (B)
5. (A) 28. (A)
6. (B) 29. (D)
7. (D) 30. (C)
8. (A) 31. (C)
9. (B) 32. (C)
10. (D) 33. (C)
11. (A) 34. (D)
12. (B) 35. (A)
13. (D) 36. (B)
14. (B) 37. (A)
15. (C) 38. (B)
16. (A) 39. (A)
17. (B) 40. (A)
18. (C) 41. (B)
19. (D) 42. (D)
20. (B) 43. (B)
21. (A) 44. (D)
22. (C) 45. (D)
23. (C)

<H2>Multiple Correct Choice Type Questions

1. (A), (B), (C) 6. (A), (B), (D)


2. (B), (D) 7. (A), (C), (D)
3. (A), (C), (D) 8. (B), (C), (D)
4. (B), (C) 9. (A), (D)
5. (A), (B), (C), (D) 10. (A), (B), (C)

<H2>Matrix-Match Type Questions

1. (A) → (r); (B) → (r); (C) → (p); (D) 3. (A) → (q); (B) → (q); (C) → (s); (D)
→ (q), (t) → (q)
2. (A) → (s); (B) → (s); (C) → (p); (D) 4. (A) → (r); (B) → (r); (C) → (r); (D)
→ (p) → (r)

<H2> Comprehension Type Questions

1. (i) (A); (ii) (D); (iii) (A) 3. (i) (A); (ii) (C); (iii) (B)
2. (i) (D); (ii) (C); (iii) (A) 4. (i) (B); (ii) (D); (iii) (B)
<H2>Assertion–Reasoning Type Questions
Solution Manual for Mathematics For JEE (Main & Advanced), Trigonometry Vector
Algebra Probability, Vol 2,
Copyright©2017 Wiley India Pvt. Ltd. All rights reserved
1. (A) 4. (A)
2. (A) 5. (A)
3. (A)
<H2>Integer Answer Type Questions<Here no parentheses required>

1. 6 6. 1
2. 9 7. 4
3. 1 8. 3
4. 4 9. 7
5. 0 10. 2

Solution Manual for Mathematics For JEE (Main & Advanced), Trigonometry Vector
Algebra Probability, Vol 2,
Copyright©2017 Wiley India Pvt. Ltd. All rights reserved
Solutions to Exercises
Chapter 7

SINGLE CORRECT CHOICE TYPE QUESTIONS


1. A bag contains 8 white, 6 black, 2 red and 4 green balls. Two balls are picked at random
at a time. The probability that they are of different colours is
7 9 14 12
(A) (B) (C) (D)
19 19 19 19
Solution: Total probability of selecting 2 balls of same colour
8
c2 + 6 c2 + 2 c2 + 4 c2 5
= p (ε ) = 20
c2 19
Now, {[1 − [ P(ε )]} gives total number of ways of selecting 2 balls of different colour
1 − 5 19 − 5 14
= =
19 19 19
Answer: (C)

2. An urn contains 6 white, 4 red and 10 green marbles. Three are drawn one after other
without replacement. The probability that they are of different colours is
2 3 6 2
(A) (B) (C) (D)
9 19 19 57
Solution: Total marbles in use =10 + 6 + 4 = 20

3 marbles selected from 20 in 20 c3 = 20 × 19 × 18


Now, number of ways of selecting 1 white of 6 = 6
1 red out of 4 = 4 ways
1 green out of 10 = 10 ways
6 × 4 × 10 2
= =
Required probability
20 × 19 × 18 57
Answer: (D)

3. A bag contains 4 black and 7 white balls. Two balls are drawn randomly. The probability
that at least one of them is black is
(A) 0.71 (B) 0.618 (C) 0.728 (D) 0.628
Solution: Total number of ways are
1black × 1white + 2 black
=
2 drawn out of 11
4 × 7 + [(4 × 3) / 2] 34
= = = 0.618
11 × 5 55
Answer: (B)

4. Three fair coins are tossed at a time. The probability of getting at least one head is
7 1 3 4
(A) (B) (C) (D)
8 8 8 8
1
Solution: Probability of getting no head= (T ⋅ TT ) = 8
1 7
so, probability of getting at least 1 head =1 − =
8 8
Answer: (A)

5. There are 9 books of which one is very popular. The books are arranged in a row at
random. The probability that the popular book is always at the middle place is
Solution Manual for Mathematics For JEE (Main & Advanced), Trigonometry Vector
Algebra Probability, Vol 2,
Copyright©2017 Wiley India Pvt. Ltd. All rights reserved
2 1 (4!) 2 2(4!)2
(A) (B) (C) (D)
9 9 9! 9!
Solution:

Popular book must be at fifth position; so, remaining 8 books placed in 8 positions in 8!
Total number of ways of arrangement = 9!
8! 1
Required probability= =
9! 9
Answer: (B)

6. The letters of the word PECULIAR are arranged at random. The probability that all the
vowels are together is
3 1 1 3
(A) (B) (C) (D)
7 7 14 14
Solution: Total vowels words = A, E , I ,U
Total words = 8
When all vowels come together
Total word = 8 − 4 + 1 = 5
5!× 4! 5 × 4 × 3 × 2 ×1 1
= =
8! 82 × 7 × 6 × 5 14
Answer: (C)

7. A and B are two students among six students. If all the six students are arranged in a row,
the probability that B immediately succeeds A is
1 1 2 2
(A) (B) (C) (D)
5 6 5 3
Solution: As B is always after A, so, total number of ways of arrangement =
(6 −1)!=
5!
Total arrangement = 6!
Required probability is
5! 1
=
6! 6
Answer: (B)

8. An ellipse of eccentricity 2 2 / 3 is inscribed in a circle. A point inside the circle is


selected at random. The probability that the point lies outside the ellipse is
2 1 1 3
(A) (B) (C) (D)
3 3 2 4
Solution: Area of ellipse
b a
= A 4 ∫ a 2 − x 2 ⋅ dx
a 0

Solution Manual for Mathematics For JEE (Main & Advanced), Trigonometry Vector
Algebra Probability, Vol 2,
Copyright©2017 Wiley India Pvt. Ltd. All rights reserved
a
4b  a 2  sin 2θ 
=  θ + 
a 2  2  θ
Here,
x = a sinθ
dx = a cosθ
a
 
 x x a −x
2 2
= 2ab sin −1   + ⋅ 
 a a a 
 o
A = abπ
Here
  2 2 2  a 2
a 1 − 
b =
2 2
 =
  3   9
 
a
b=
3
a π a2
= A a= π
3 3
(π a 2 / 3) 1
Probability that point lies inside=
ellipse =
π a2 3
1 2
So, probability that point, outside ellipse =1 − =
3 3
Answer: (A)

9. Two subsets A and B are chosen at random from a set containing 6 elements one after
another with replacement. The probability that A and B contain equal number of elements
is
131 231 331 101
(A) (B) (C) (D)
1021 1021 1021 1021
Solution: Number of subset of n-element set = 2n

Total number of choice = 2n ∗ 2n = 4n


Here n = 6
26 ∗ 26 = 4096
Now, here sets A and B are equal sets, total number of equal sets = 924

924 231
=
Required probability =
1084 1021
Answer: (B)

10. A lot contains 12 cell phones out of which 4 are defective. If two cell phones are drawn
from the lot at random, the probability that one is defective and the other is non-defective
is
16 15 13 17
(A) (B) (C) (D)
33 13 33 33
Solution: Number of ways selecting 1 defective at: 4 = 4
Number of ways 1 correct phone 8 = 8

Solution Manual for Mathematics For JEE (Main & Advanced), Trigonometry Vector
Algebra Probability, Vol 2,
Copyright©2017 Wiley India Pvt. Ltd. All rights reserved
4 × 8 16
=
Required probability =
66 33
Answer: (A)

11. On the real line (i.e., x-axis) points p and q are selected at random such that −2 ≤ q ≤ 3
and 0 ≤ p ≤ 3. The distance between p and q is greater than 3 with probability
2 1 1 3
(A) (B) (C) (D)
3 3 4 4
Solution: Point ‘q’ is at − 2 , then
1 3 1
1≤ p ≤ 3 1 ≤ p ≤ , p =
3 3 3
Answer: (B)

12. A and B are two events such that


3 1 1
P=( A) , P=( B) and P( A ∩=B)
8 2 4
Then P( A ∩ B ) is equal to
1 1 1 1
(A) (B) (C) (D)
8 3 16 4
Solution:
3 1 1
P( A ∩ B ) = P( A) − P( A ∩ B) = − =
8 4 8
13. Answer: (A)
14. A bag contains 4 red, 3 white, 2 blue and 1 green marbles. Four marbles are drawn from it
at random. The probability that the draw contains one each from the given colours is
1 6 4 31
(A) (B) (C) (D)
7 7 35 35
Solution:
4 × 3× 2 ×1 4
=
10 × 9 × 8 × 7 / 4 × 3× 2 ×1 35
Answer: (C)

15. A gardener plants 3 apple trees, 4 neem trees and 5 mango trees in a row at random. The
probability that no two mango trees are next to one another is
7 1 8 1
(A) . (B) . (C) . (D) .
99 9 99 11
Solution:
Apple + neem = 4 + 3 = 7
so, total place, where mango trees plant = 7 + 1 = 8
8
c5 8× 7 7
=
Hence, required probability = =
c5 11 × 9 × 8 99
12

Answer: (A)

16. The first 11 letters of the English alphabet are arranged in all possible ways at random.
The probability that there are exactly 4 letters in between A and B is
12 14 7 6
(A) (B) (C) (D)
55 55 55 55
Solution:
A B
Position 1 6
Solution Manual for Mathematics For JEE (Main & Advanced), Trigonometry Vector
Algebra Probability, Vol 2,
Copyright©2017 Wiley India Pvt. Ltd. All rights reserved
2 7
3 8
4 9
5 10
6 11
so, A can position in 6 ways again A and B interchange their position, so probability is
2×6 6
=
11 × 10 55
Answer: (D)

17. Two positive real numbers x and y such that x + y = 12 are selected at random. The
probability that xy is greater than or equal to 3/4 times their greatest product is
1 1 2 3
(A) (B) (C) (D)
3 2 3 4
Solution:
x + y = 12, y = (12 − x)
Now, greatest product of xy, when xy = 6 so,
xy = 36
Now, given that
3
xy ≤ × 36
4
xy ≤ 27
x(12 − x) ≤ 27 3 ≤ x ≤ 9
x 2 −12 x + 27 ≤ 0
But we have, x ≤ 6
( x − 3) ( x − 9) ≤ 0
so,
3 1
3 ≤ x ≤ 6 p( E ) = =
6 2
Here,
=
n( E ) 3,=
n( S ) 6
Answer: (B)

18. From 15 consecutive positive integers, 3 are selected at random. The probability that they
are in AP is
3 7 9 11
(A) (B) (C) (D)
65 65 65 65
Solution:
Common difference (d)
d = 1, number of terms = (1, 2, 3,) (2, 3, 4) (3, 4, 5)
n
= =
d 2, number =
of terms (1,3,5) (2, 4, 6)  11 X i
i =1
= =
d 3................................. 9
= =
d 4................................ 7
= =
d 5................................ 5
= =
d 6............................... 3
= =
d 7............................... 1
Total n( E ) = 13 + 11 + 9 + 7 + 5 + 3 + 1 = 49

Solution Manual for Mathematics For JEE (Main & Advanced), Trigonometry Vector
Algebra Probability, Vol 2,
Copyright©2017 Wiley India Pvt. Ltd. All rights reserved
n( S=) 15
c=
2 65 × 7
49 7
=
p( E ) =
65 × 7 65

Answer: (B)

19. Five numbers are selected at random from the first 50 natural numbers and arranged in the
increasing order x1 < x2 < x3 < x4 < x5 . The probability that x3 is 30 is
29
c2 × 20 c2 39
c2 × 19 c2
(A) 50
(B) 50
c5 c5
30
c2 × 20
c2 55
(C) 50
(D)
c5 1029
Solution:
Number of ways of selecting 2 numbers that are less then 30 = 29 c2
and number of ways of selecting 2 numbers between 31 and 50 = 20 c2
29
c2 ×20 c2
Required probability = 50
c5
Answer: (A)

20. Two different coloured dice are rolled. A is the event that the sum of the faces appeared is
odd and B is the event that one of the faces turned up shows face 1. Then P( A ∩ B ) is
1 2 1 3
(A) (B) (C) (D)
3 3 4 4
Solution:
P( A ∩ B )= P( A) − P( A ∩ B)
Here
18 1
P( A=) sum is odd = =
36 2
6 1
P( A ∩ B) = (1, 2), (1, 4) (1, 6) (6,1), (4,1) (2,1) = =
36 6
1 1 1
− =
2 6 3
Answer: (A)

21. Two dice are thrown at a time. The probability that the sum is greater than 8 is
1 1 1 5
(A) (B) (C) (D)
6 9 3 18
Solution:

n(ε ) = (5, 4), (5,5), (5, 6), (6,3), (6, 4) (6,5)


(6, 6) = 7
n( S ) = 36
7
p( E ) =
36
Answer <AQ>Solution is not matching.</AQ>

Solution Manual for Mathematics For JEE (Main & Advanced), Trigonometry Vector
Algebra Probability, Vol 2,
Copyright©2017 Wiley India Pvt. Ltd. All rights reserved
22. A problem in combinatorics is given to three students whose probabilities of solving it
are, respectively, 1/2, 3/4 and 1/4. They try the problem independently. The probability
that majority of them solving the problem is
1 1 2 3
(A) (B) (C) (D)
3 2 3 4
Solution:
P( A ∩ B ∩ C ) + P( A ∩ B ∩ C ) + P( A ∩ B ∩ C ) + P( A ∩ B ∩ C )
P( A) ⋅ P( B)[ P(C )] + P( A) P( B) P(C ) + P( A) P ( B ) ⋅ P (C ) + P ( A) ⋅ P ( B ) ⋅ P (C )
1 3 3 1 1 1 1 3 1 1 3 1
= ∗ ∗ + ⋅ ⋅ + ⋅ ⋅ + ⋅ ⋅
2 4 4 2 4 4 2 4 4 2 4 4
16 1
= =
32 2
Answer: (B)

23. A lot contains 15 cameras out of which 4 are defective. Cameras are drawn one after
another to identity the defective pieces. The process continues till all the defective pieces
are identified. The probability that the procedure ends at the ninth testing is
11 13 8 9
(A) (B) (C) (D)
195 195 195 195
Solution: Probability of selecting 4 defective out of
=
15 =
15
c4 1365
It is given, in 9th test, so,
8× 7× 6
=n( E ) 8=
C3 = 56
3 × 2 ×1
56 8 8
P(=E) = =
1365 195 195
Answer: (C)

24. The probability that man aged x years to die in five years is 1/3. Out of persons
P1 , P2 , P3 , P4 and P5 , each aged x years, P1 will die in 5 years and he is the first person
to die in 5 years is
211 311 111 1
(A) (B) (C) (D)
1215 1315 1115 5
<solution missing>
25. The probabilities of three mutually exclusive events are
1+ 3p 1− p 1− 2 p
, ,
3 4 2
Then
1 1 1
(A) 0 < p ≤ (B) ≤ p ≤
3 3 2
1 1 1
(C) < p < 1 (D) ≤ p ≤
2 5 4
Solution: Given that, events are mutually exclusive,
so,
P( A ∪ B ∪ C=) P( A) + P( B) + P(C )
1 + 3P 1 − P 1 − 2 P
1= + +
3 4 2
12= 13 − 3P

Solution Manual for Mathematics For JEE (Main & Advanced), Trigonometry Vector
Algebra Probability, Vol 2,
Copyright©2017 Wiley India Pvt. Ltd. All rights reserved
1
=
3P 1,=
P
3
Also, for event probability ≤1 , so,
1− 2P
≤1
2
1
− 2 P ≤1, P ≤
2
Hence
1 1
≤P≤
3 2
Answer: (B)

26. Two events A and B are such that P(A) = 0.7, P(B) = 0.4 and P( A ∩ B ) =
0.5. Then
P[ B /( A ∪ B )] is equal to
1 1 1 2
(A) (B) (C) (D)
2 3 4 3
Solution:
P( A ∩ B)= P( A) − P( A ∩ B)= 0.5
P( A ∩ B) = 0.7 − 0.5 = 0.2
Now,
 B  P[ B ∩ ( A ∪ B)]
P =
 A∪ B  P( A ∪ B)
P( B ∩ A) ∪ P( B ∩ B) 0.2 1
= = =
P( A) + P( B) − P( A) + P( A ∩ B) 0.6 + 0.2 4
Answer: (C)

27. A fair die is rolled four times. Out of the four face values obtained, the probability that the
minimum face value is greater than or equal to 2 and the maximum value is less than or
equal to 5 is
16 1 8 65
(A) (B) (C) (D)
81 81 81 81
Solution:
4 2
A= (2,3, 4,5), P( A)= =
6 3
Total probability
4
 2  16
= = 
 3  81
Answer: (A)

28. A purse contains 4 half-rupee coins and 3 rupee coins. Another purse contains 6 half-
rupee coins and 2 rupee coins. One of the purses is selected random and a coin is drawn
from it. The probability that it is a half-rupee coin is.
35 25 37 27
(A) (B) (C) (D)
56 56 56 56
Solution:
A = Selecting first purse
B = Selecting 2nd purse
C = Selecting 50 paise coin

Solution Manual for Mathematics For JEE (Main & Advanced), Trigonometry Vector
Algebra Probability, Vol 2,
Copyright©2017 Wiley India Pvt. Ltd. All rights reserved
1
P=
( A) P=
( B)
2
C = ( A ∪ B) ∩ C
C =( A ∩ C) ∪ (B ∩ C)
P(C )= P ( A ∩ C ) ∪ P( B ∩ C )
C  C 
= P( A) ⋅ P   + P( B) ⋅ P  
 A B
3
1 42 1 6
= ⋅ + ⋅
2 7 2 8
2 3 16 + 21 37
= + = =
7 8 56 56
Answer: (C)

29. If each of a, b and c takes values from the set {1, 2, 3, 4, 5, 6}, then the probability that
the equation ax 2 + bx + c = 0 has real roots is equal to
53 43 23 63
(A) (B) (C) (D)
216 216 216 216
Solution:
Sample space = n( S ) = 6 × 6 × 6 = 216
For real roots, b 2 ≥ 4ac
This is true for b = {2,3, 4,5, 6}
and a, c = {1, 2,3, 4,5, 6}
n( E ) = 43
43
p( E ) =
216
Answer: (B)

30. In a regular hexagon ABCDEF, if three vertices are selected at random, then the
probability that they form the vertices of an equilateral triangle is
1 1 2 1
(A) (B) (C) (D)
5 3 3 10
Solution: A hexagon has six vertices out of which three selected in 6 c3 = 20 ways.

To form equilateral ∆1 , we connect opposite vertices.


Here, 2 opposite vertices from A is E and D·
Then selected in 2 c1 = 2

Solution Manual for Mathematics For JEE (Main & Advanced), Trigonometry Vector
Algebra Probability, Vol 2,
Copyright©2017 Wiley India Pvt. Ltd. All rights reserved
2 1
Required probability= =
20 10
Answer: (D)

31. From the set {1, 2, 3, …, 15}, seven numbers are selected at random one after another
with replacement. The probability that the largest of these is 9 is
7 7 7 7
 8  9  7  3
(A)   (B)   (C)   (D)  
 15   10   10   5
Solution:
E = number is less or equal to 9
n( E ) = 9
9 3
n( S=) 15, p( E=) =
15 5
Total probability
7
3
P( E ) =  
5

Answer: (D)

32. Two cards are drawn at random from a deck of 52 playing cards one after another,
replacing the earlier drawn card. The probability that the first card is diamond and the
second card is queen is
3 4 5 1
(A) (B) (C) (D)
13 13 52 52
Solution:
A = Card is diamond,
1
P( A) =
4
B = Card is queen,
1
P( B) =
13
Now, first card is diamond and second is queen
Probability
 B  P( A) ⋅ P( A ∩ B) 1
= P( A) ⋅ P  = = P( A ∩ B)=
 
A P ( A) 52
Answer: (D)

33. Let S be a four-element set. If a two-element subset of S is selected at random, the


probability that it belongs to a partition class of S is
2 1 3 4
(A) (B) (C) (D)
9 5 5 5
Hint: See Figure 1.5 and Single Correct Choice Type Question 13, Chapter 1, Vol. 1,
page 60.
Solution: See Definition 1.28
Answer; (A)

34. Let w ≠ 1 be a cube root of unity. A fair die is thrown three times. If r1 , r2 and r3 are the
numbers that appeared on the faces, the probability that wr1 + wr2 + wr3 =
0 is
1 1 2 1
(A) (B) (C) (D)
18 9 9 36

Solution Manual for Mathematics For JEE (Main & Advanced), Trigonometry Vector
Algebra Probability, Vol 2,
Copyright©2017 Wiley India Pvt. Ltd. All rights reserved
<solution missing>
35. “A” is one of the six horses entered for a race and it is to be ridden by one of the two
jockeys B and C. If B rides A, then all the horses are equally likely to win. If C rides A,
the chances of A’s win will be trebled, then, the odds in favour of A is
(A) 1:2 (B) 2:1 (C) 1:3 (D) 3:1
Solution:
1
A = One horse out of six =
6
A
= Horse ridden by B·
B
A
= Horse ridden by C
C
A =A ∩ ( B ∪ C )

A = ( A ∩ B) ∪ ( A ∩ C )
P( A)= P( A ∩ B) + P( A ∩ C )
 A  A
P( A) = P( B) ⋅ P   + P(C ) ⋅ P  
B C 
1 1 1 1
= ⋅ + ⋅3⋅ 
2 6 2 6
1
=
3
1 2
P( A) =1 − P( A) =1 − =
3 3

Odds in favour

P( A)  1  1/ 3 1
= = =  =
P( A)  3  2 / 3 2
Answer: (A)

36. A and B are two persons. A speaks truth 2 out of 3 times, while B speaks truth 3 out of 4
times. They are independent witnesses in an accident case which is in a police court. The
probability that they have given identical statements is
5 1 7 2
(A) (B) (C) (D)
12 3 12 3

Solution:

2 3
=
P( A) =, P( B)
3 4

2 1 3 1
P( A) =1 − = , P( B) =1 − =
3 3 4 4

P(ε ) = P ( A) ⋅ P( B) + P( A) ⋅ P( B)

2 3 1 1 7
= ⋅ + ⋅ =
3 4 3 4 12

Solution Manual for Mathematics For JEE (Main & Advanced), Trigonometry Vector
Algebra Probability, Vol 2,
Copyright©2017 Wiley India Pvt. Ltd. All rights reserved
Answer: (C)

37. There are five pairs of socks in a cupboard. If 4 socks are drawn at random, the
probability that the draw contains exactly one matching is
8 2 5 13
(A) (B) (C) (D)
21 7 7 21

Solution: One matching sock selected out of 5 in 5 c1

Now, 2 selected socks are unmatched

5
c1 ∗ 2(4 c1 ∗ 3c1 ) =
120

Total ways of selection =

10 × 9 3 × 8 × 7
=10
c4 = 210
4 × 3 × 2 ×1

120 4
(ε )
P= =
210 7

<AQ: Asnwer and option in Question not matching in Question 36.>Answer

38. In a bolt-producing factory, machines A, B and C give 25%, 35% and 40% of the total
output, respectively. It is also known that 5%, 4% and 2% of them are defective. If a bolt
of the factory is selected at random then the probability that it is defective is
69 63 73 83
(A) (B) (C) (D)
2000 2000 2000 2000

Solution:

25 35 40
=
P( A) = , P( B) = P(C )
100 100 100

E = for defective product


E 5 E 4 E 2
= P  =,P  = P 
 A  100  B  100  C  100
e  ee  
P(e ) =P( A) ⋅ P   + P( B) ⋅ P   + P(c) ⋅ P  
 
A  
B C 
25 5 35 4 40 2
= ⋅ + ⋅ + ⋅
100 100 100 100 100 100
345 69
= =
10000 2000
Answer: (A)

39. A and B are two politicians who settle disputes between parties independently. The
probabilities of their settling disputes amicably are, respectively, 4/7 and 7/15. A firm is
engaged for a settlement of dispute between the firm and the employees. The probability
that neither of them settle the dispute is

Solution Manual for Mathematics For JEE (Main & Advanced), Trigonometry Vector
Algebra Probability, Vol 2,
Copyright©2017 Wiley India Pvt. Ltd. All rights reserved
73 32 8 16
(A) (B) (C) (D)
105 105 25 25
Solution:
4 3
=
P( A)=, P( A)
7 7
7 8
=
P( B) = , P( B)
15 15
3 8 8
P( A ∩ B) =P( A) ⋅ P( B) = ∗ =
7 15 35
Answer: (C)

40. S is a five-element set. A subset P of A is selected at random. After inspecting the


elements of P, the elements are restored to S. Again a subset Q of S is selected at random.
The probability that P and Q form a partition of S (i.e., P ∪ Q = S and P ∩ Q = φ ) is
1 1 1 1
(A) (B) (C) (D)
2 16 8 32
Solution:
Here number of subset for a 5 element set= 2=
5
32
Now, as P( P ∪ θ ) =
ς and ( P ∪= Q) ς and P ∩
= Q d
1
P( P ∪ Q) =
32
Answer: (D)

41. A set X has 10 elements. Subset A of X is selected at random and then the elements of A
are returned to X. Again a subset B of X is chosen at random. The probability that B is a
subset of A is
10 5
1 1  3  3
(A) (B) (C)   (D)  
210 25  4  4
Solution:
Total subset = 210
Total number of element = 210 ∗ 210 = 220 = 410
Now, let n is the total number of elements in set A
then set B has 2n − r elements
n−r
Now, for ‘r’ element of set A· 2 , elements of set B
n n n n n
∑ Cr (2)n−r =C0 2n + C1 2n−1 + C0 20
r =0

=(2 + 1)n =(3)n


Hence, required probability
10
(3) n  3 
= =  
4n  4 
Answer: (C)

42. Three groups of workers contain 3 women, 1 man; 2 women and 2 men and 3 men and 1
woman. One worker from each group is selected for a work at random. The probability
that the selection constitutes 2 men and 1 woman is
9 3 1 13
(A) (B) (C) (D)
32 32 32 32

Solution Manual for Mathematics For JEE (Main & Advanced), Trigonometry Vector
Algebra Probability, Vol 2,
Copyright©2017 Wiley India Pvt. Ltd. All rights reserved
Solution: 2 men and 1 woman selected in
3 2 3 1 2 3 1 2 1
⋅ ⋅ + ⋅ ⋅ + ⋅ ⋅
4 4 4 4 4 4 4 4 4
26 13
= =
64 32
Answer: (D)

43. Two persons A and B each tosses a fair coin thrice. The probability that they get same
number of heads is
5 11 7 9
(A) (B) (C) (D)
16 16 16 16
<AQ: Solution missing. Please check.>

Answer: (A)
44. If A and B are two independent events in a random experiment then
P ( A ∪ B ) + P( A ∩ B ) is equal to
1 2
(A) 0 (B) (C) 1 (D)
2 3
Solution:
P( A ∪ B)= P( A) + P( B) − P( A) ⋅ P( B)
So, A and B are independent events. So,
P( A ∩ B)= P( A) ⋅ P( B)
= [1 − P( A)][1 − P( B)]
so,
P( P)( A ∪ B) + P( A ∩ B)
= P( A) + P( B) − P( A) ⋅ P( B) − P( A) − P( B) + P( A) ⋅ P( B) + 1 = 1
Answer: (C)

45. Three bags have the following combination of balls.


Bag I: 7 black, 3 white
Bag II: 4 black, 6 white
Bag III: 2 black, 8 white
The probabilities of selecting bags are, respectively, 1/5, 3/5 and 1/5. One of the bags is
chosen at random and two balls are drawn from it, one after another without replacement.
The probability that both balls are black is
4 1 41 8
(A) (B) (C) (D)
45 9 45 45
Hint: Total probability.
Solution: Let X = selecting 2 black balls
B1 = selecting bag I
B2 = selecting bag II
B3 = selecting bag III
X  X  X 
P   , P   and P  
 B1   B2   B3 
Selecting balls form bags I, II and III

Solution Manual for Mathematics For JEE (Main & Advanced), Trigonometry Vector
Algebra Probability, Vol 2,
Copyright©2017 Wiley India Pvt. Ltd. All rights reserved
 x   X  X 
P( X ) = P( B1 ) ⋅ P   + P( B2 ) ⋅ P   + P( B3 ) ⋅ P  
 B1   B2   B3 
1 21 3 6 1 1 8
=⋅ + ⋅ + ⋅ =
5 45 5 45 5 45 45
Answer: (D)

46. Five men out of 100 and 25 women out of 1000 have colour blindness. If a colour blind
person is selected at random, the probability that the person is a man is
1 2 1 1
(A) (B) (C) (D)
3 3 2 4
Hint: Use Bayes’ theorem.
Solution:
A= Person is man
B = Person is woman
C = Colour blindness
C 5 1
P =  =
 A  100 20
 C  25 1
P =  =
 B  1000 40
Now
1
P=
( A) P=
( B)
2
so, by Baye’s theorem
 A P( A) ⋅ P(C / A)
P =
 C  P( A) ⋅ P(C / A) + P( B ) ⋅ (C / B)
(1 / 2) ⋅ (1 / 20) 2
=
(1 / 2) ⋅ (1 / 20) + (1 / 2) ⋅ (1 / 40) 3
Answer: (B)

47. A fair coin is tossed (m + n) times where m > n. The probability for m consecutive heads
is
n+2 n +1 m+2 m +1
(A) m +1 (B) m +1 (C) n +1 (D) n +1
2 2 2 2
 
1
Solution: Probability for head =  
2
Now, probability for m consecutive head
m+n m +1
1
C ⋅ 
m 2

m+n−2 n+2
= =
2m + 1 2m + 1
Answer: (A)

48. Three identical bags contain the following composition of balls.


Bag I: 2 white, 1 black
Bag II: 1 white, 2 black
Bag III: 2 white, 2 black

Solution Manual for Mathematics For JEE (Main & Advanced), Trigonometry Vector
Algebra Probability, Vol 2,
Copyright©2017 Wiley India Pvt. Ltd. All rights reserved
The probabilities of selecting the bags are equal. One of the bags is selected and a ball is
drawn from it. The drawn ball is found to be black and it is kept outside. Then the
probability of a black ball again is
1 2 2 3
(A) (B) (C) (D)
3 3 5 5
Solution: Here probability of selecting a black ball in 1st trial
B  B   B
P( B) = P( B1 ) ⋅ P   + P( B2 ) ⋅ P   + P( B3 ) ⋅ P  
 B1   B2   B3 
1  1 2 2  1 18 1
=  + + = ⋅ =
3  3 3 4  3 12 2
Now, on second ball drawn:
1  S   S 
P( S=)  P( B2 ) ⋅ P   + P( B3 ) ⋅ P    +
2   B2   B3  
2 S   S   S 
 P( B1 ) ⋅ P   + P ( B2 ) ⋅ P   + P( B3 ) ⋅ P   
3   B1   B2   B3  
2
=
3
so,
1
P( B) ⋅ P( S ) =
3
Answer: (A)

49. A fair die is rolled. The probability that first 1 appears at even number of trials is
6 1 5 5
(A) (B) (C) (D)
11 6 36 11
<solution missing>
50. An experiment has 10 equally likely outcomes. Let A and B be two non-empty events of
the experiment. If A consists of 4 outcomes, the number of outcomes B should have such
that A and B are independent is
(A) 2, 4 or 3 (B) 3, 6 or 9
(C) 4 or 8 (D) 5 or 10
<solution missing>
51. If the mean and variance of a Binomial variate X are, respectively, 2 and 1, then P (X > 1)
is
11 5 7 9
(A) (B) (C) (D)
16 16 16 16
Solution: Here number of trial = n, so,
= 2 np = and 1 npq
1 1
q = , p =1 − q =
2 2
1
2= n ⋅  
2
So, n = 4 . Now
P (X > 1) = 1 − P (X = 1) − P (X = 0)
11
P=
16
Answer: (A)

Solution Manual for Mathematics For JEE (Main & Advanced), Trigonometry Vector
Algebra Probability, Vol 2,
Copyright©2017 Wiley India Pvt. Ltd. All rights reserved
MULTIPLE CORRECT CHOICE TYPE QUESTIONS
1. If A and B are independent events, then
(A) A and B are independent
(B) A and B are independent
(C) A and B are independent
(D) A and B are exclusive events
Solution:
(A) Given that P( A ∩ B=) P ( A) ⋅ P( B)
P( A ∩ B)= P( A) − P( A ∩ B)
Now,
P( A) ⋅ P( B)= P( A)[1 − P( B)]
= P( A) − P( A) ⋅ P( B)
= P( A) − P( A ∩ B) (Proved)
(B)
P( A ∩ B)= P( B) − P( A ∩ B)
P( A) ⋅ P( B) = P( B) − P( A) ⋅ P( B)
= P( B) − P( A ∩ B) (Proved)
(C) P( A ∩ B ) =
1 − P ( A) − P ( B ) + P ( A ∩ B )
[1 − P( A)][1 − P( B)] =
1 − P( A) − P( B) + P( A) ⋅ P( B)
=
1 − P( A) − P( B) + P( A ∩ B)
Answer: (A), (B), (C)

2. Two different coloured fair dice are thrown at a time. Then


1
(A) (Sum of the faces is odd) =
2
11
(B) P(At least one face is odd) =
36
1
(C) P(Sum of the faces is 5) =
9
25
(D) P(Neither of the faces is 1) =
36
Solution:
(A)
Out of 36 outputs, 18are odd and 18 are even
so,
18 1
=P =
36 2
(B) (1, 1), (1, 2), (1, 3), (1, 4), (1, 5), (1, 6), (2, 1), 2, 3), (2, 5), (3, 1), (3, 2), 3, 4), (3, 5),
(3, 6), (4, 1), (4, 3), (4, 5), (5, 1), (5, 2), (5, 3), (5, 4), (5, 5), (5, 6), (6, 3), (6, 5)
27 3
= =
36 4
(C)
(1, 4) (2,3) (3, 2), (4,1)
4 1
=
P =
36 9
(D)
Total output with true 1 = 11, so,

Solution Manual for Mathematics For JEE (Main & Advanced), Trigonometry Vector
Algebra Probability, Vol 2,
Copyright©2017 Wiley India Pvt. Ltd. All rights reserved
11 25
P =−
1 =
36 36
Answer: (A), (C), (D)

3. Let P be a probability function on a samples S and B a fixed non-empty subset of S. For


any subset A of S, define
PB ( A) = P( A / B)
Then
(A) PB ( S ) = 1
P( A ∩ B ∩ C )
(B) PB ( A ∩ C ) =
P( B)
(C) PB ( A ∪ C=
) PB ( A) + PB (C ) − PB ( A ∩ C )
(D) PB ( A ∩ C ) =
PB ( A) PB (C ) , whenever A and C are independent.
Solution:
(A)
Here
n( E ) = n( S )
n( S )
B= (S ) = 1
P
n( S )
(B)
 A ∩ C  P( A ∩ B ∩ C ) P[( A ∩ C ) ∩ B]
=
P  =
 B  P( B) P( B)
(C)
PB ( A ∪ C )= P ( A) + P (C ) − P ( A ∩ C )
(D)
PB ( A ∩ C=) PB ( A) ⋅ PB (C )

Answer: (A), (B), (C), (D)

4. Let A, B, C be mutually exclusive and exhaustive events of a random experiment with


sample space S. Suppose P is a probability function on S. Which of the following are not
correct?
=
(A) =
P( A) 0.24, =
P( B) 0.4, P(C ) 0.2
1 1 1
=
(B) P( A) = , P( B) = , P(C )
4 4 2
=
(C) =
P( A) 0.1, =
P( B) 0.2, P(C ) 0.6
(D) P=
( A) 0.6, P ( A ∩
= B ) 0.5

(A)

P( A) + P( B) + P(C ) ≠ 1 True
P( A) + P( B) + [ P(C )]
1 1 1
= + + =1
4 4 2
P( A) + P( B) + (C ) ≠ 1
(D) As A, B, C are mutually exclusive
So,
P( A) + P( B) + P(C ) = 1

Solution Manual for Mathematics For JEE (Main & Advanced), Trigonometry Vector
Algebra Probability, Vol 2,
Copyright©2017 Wiley India Pvt. Ltd. All rights reserved
P( A ∩ B)= P( A) − P( A ∩ B)
= P( A)
But
0.6 ≠ 0.5
Answer: (A), (B), (C), (D)
=
5. Let A and B be two events of a random experiment. Suppose P( A) p=
1 , P( B) p2 and
P( A ∩ B) =
p3 . Then
(A) ( A ∪ B) =1 − p1 + p3
(B) P( A ∩ B ) =1 − p1 − p2 + p3
(C) P[ A ∪ ( A ∩ B)] = p1 + p2 − p3
(D) P( A ∪ B) = p1 + p2 − p3
Solution:

(A)
P( A ∪ B) = 1 − P( A) + P( B) − P( B) + P ( A ∩ B)
=1 + P( B) − P( A ∪ B)
=−
1 P3 + P2

(B)
P( A ∩ B=) P ( A) + P( B) − P( A ∪ B)
=1 − P( A) + 1 − P ( B) − P( A) + P ( B) − 2 P( A ∩ B),
2 − 2[ P( A) + P( B)] + 2[ P( A)] + P( B) − P( A ∩ B)
=2 − 2 P ( A ∪ B)
= 2 − 2 P3

(C)
P[ A ∪ ( A ∩ B)]
= P( A) + P( A ∩ B) − P( A ∩ A ∩ B)
= P( A) + P( B) − P( A ∩ B)
P( A ∪ B) =
P3
(D)

P( A ∪ B) =−
1 ( A ∪ B)
= 1 − P3
Answer: (A), (B), (C)

6. Two fair dice are thrown at a time. Let A, B, C be the following events:
A = [(x, y) | x is odd]
B = [(x, y) | y is odd]
C = [(x, y) | x + y is odd]
Then
(A) P( A ∩ B) = P( A) P( B)
(B) P( B ∩ C ) = P( B) P(C )
(C) P(C ∩ A) = P(C ) P( A)
(D) P( A ∩ B ∩ C ) = P( A) P( B) P(C )
Solution:

Solution Manual for Mathematics For JEE (Main & Advanced), Trigonometry Vector
Algebra Probability, Vol 2,
Copyright©2017 Wiley India Pvt. Ltd. All rights reserved
n( A) 18 1
P( A=) = =
n( s ) 36 2
Here
n( B ) 1
P=
( B) =
n( s ) 2
1
P(C ) =
2
(A)
P( A ∩ B)= P( A) ⋅ P( B)
9 1 1
= ⋅ True
36 2 2
(B)
P( B ∩ C )= P( B) ⋅ P(C )
9 1 1
= ⋅ True
36 2 2
(C)
P(C ∩ A=) P(C ) ⋅ P( A)
9 1 1
= ⋅ = True
36 2 2
(D)
P( A ∩ B ∩ C=) P( A) ⋅ P( B) ⋅ P(C )
1 1 1
0= ⋅ ⋅ False
2 2 2
Answer: (A), (B), (C)

7. The probabilities of the events A ∩ B, A, B and A ∪ B of a random experiment are in


AP in the given order, with the common different as P(A). Assume that P(A) > 0. Then A
and B are
(A) Mutually exclusive.
(B) Independent.
(C) Exhaustive and dependent.
(D) Such that one is twice as likely as the other.
Solution:
P( A)= P( A ∩ B) + P( A) ⇒ P( A ∩ B)= 0
(A) P(A) = P(B) are mutually exclusive
(B) False
(C) False
(D) P( A ∪ B)= P( A ∩ B) + 3P( A)
and P( B)= P( A ∩ B) + 2 P( A) ⇒ P( B)= 2 P( A)

Answer: (A), (D)

8. The letters of the word PROBABILITY are arranged in a row at random. Let E1 be the
event that the two Is are together and E2 is the event that two Bs are together. Then
2 18
(A) P( E1 ∩ E2 ) = (B) P( E1 ∪ E2 ) =
55 55

Solution Manual for Mathematics For JEE (Main & Advanced), Trigonometry Vector
Algebra Probability, Vol 2,
Copyright©2017 Wiley India Pvt. Ltd. All rights reserved
1 2
(C) P( E1 / E2 ) = (D) P( E2 / E1 ) =
5 5
Solution:
(A) n( E1 ∩ E=
2 ) 2, n( E2 ∪ E=
1 ) 55
2
P( E1 ∩ E2 ) =
55

18
(B) P(ε E1 ∪ E2 ) =
55

 E1  p( E1 ∩ E2 ) 1 2 / 55
(C)   = =
 E2  p( E ) 5 2 /11

Answer: (A), (B), (C)

9. The probabilities of 12th standard student passing the subjects Maths, Physics and
Chemistry are, respectively, m, p and c, the probability of passing at least one subject is
3/4, the probability of passing at least two subjects is 1/2 and the probability of passing
exactly two subjects is 2/5. Which of the following relations are true?
19 27
(A) p + m + c = (B) p + m + c =
20 20
1 1
(C) pmc = (D) pmc =
10 4

Solution:

3
P(k − m) (1 − C ) + m (1 − P)(1 − C ) + (1 − m) (1 − P)(C ) =
4
3
P + m + C − 2( pm + mc + pc) − 3 pcm =
4

3 1
(1 − p)(1 − m) (1 − c) =1 − =
4 4

1
1 − ( P + m + c) + ( Pc + mc + pm) − pcm = (1)
4

1
( P + m + c) − 2( Pm + mc + Pc) − 2 pcm = (2)
2

2
( P + m + c) − 2( Pm + mc + Pc) − 3 pcm = (3)
5

From Eqs. (1)–(3)

1 2 1
pcm = − =
2 5 10

Solution Manual for Mathematics For JEE (Main & Advanced), Trigonometry Vector
Algebra Probability, Vol 2,
Copyright©2017 Wiley India Pvt. Ltd. All rights reserved
From Eqs. (1) and (2)

2 − ( P + c + m) − 4 pcm =
1

1 27
2 − 4  p + c + m =
 10  20
Answer: (B), (C)

10. A and B are independent events of a random experiment. If P( A ∩ B) =


1/ 6 and
P( A ∩ B ) =
1/ 3. Then
1 1
(A) P( A) = (B) P( A) =
2 3
1 1
(C) P( A) = (D) P( B) =
3 2

Solution:

1
P( A ∩ B)= P( A) ⋅ P( B)=
6

P( A ∩ B)= P( A) ⋅ P( B)

1
1 − P( A) − P( B) + P ( A) ⋅ P ( B) =
3

1 1
1 − P( A) − =
6 P( A) 6

6 P( A)2 − 5P( A) + 6 =0

1 1
P( A) = ,
2 3

1 1 1
=
P( B) = ,
6 P( A) 2 3

Answer: (A), (B), (C), (D)


MATRIX-MATCH TYPE QUESTIONS
In each of the following questions, statements are given in two columns, which have to be
matched. The statements in column I are labeled as (A), (B), (C) and (D), while those in column II
are labeled as (p), (q), (r), (s) and (t). Any given statement in column I can have correct matching
with one or more statements in column II. The appropriate bubbles corresponding to the answers
to these questions have to be darkened as illustrated in the following example.

Solution Manual for Mathematics For JEE (Main & Advanced), Trigonometry Vector
Algebra Probability, Vol 2,
Copyright©2017 Wiley India Pvt. Ltd. All rights reserved
Example: If the correct matches are (A) → (p), (s) , (B) → (q), (s), (t), (C) → (r), (D) → (r), (t) ,

that is if the matches are (A) → (p) and (s); (B) → (q), (s) and (t); (C) → (r); and (D) → (r), (t),
then the correct darkening of bubbles will look as follows:

1. A, B and C are three events of a random experiment and P is a probability function. If


P=( A) 0.6, P=( B) 0.4, P=(C ) 0.5, P( A ∪= B) 0.8, P( A ∩=C ) 0.3, P( A ∩ B ∩= C ) 0.2
and P( A ∪ B ∪ C ) = 0.85, then match the items of Column I with those of Column II.

Column I Column II
(A) P( A ∩ B) is (p) 0.35
(B) P( B ∩ C ) is (q) 0.8
(C) P( A / B) is equal to (r) 0.2
(D) P( A ∪ B ) equals (s) 0.3
2
(t)
3

Solution:

(A) P( A ∩ B)= P( A) + P( B) − P( A ∪ B)

= 0.6 + 0.4 − 0.8 = 0.2


Answer: (r)
(B) P ( B ∩ C )= P( B) + P(C ) − P( B ∪ C )

0.85 = 0.6 + 0.4 + 0.5 − 0.2 − P( B ∩ C ) − 0.3 + 0.2


P( B ∩ C ) = 1.2 − 0.85 = 0.35
Answer: (p)
 A  P( A ∩ B) 1 − P( A ∩ B)
(C) =
P  =
B P( B) P( B)

0.2 0.2
=
1− = = 0.5
0.4 0.4
Answer: (t)

2. A, B, C are three events such that


3 1
P=
( B) , P( A ∩ B ∩=
C)
4 3

Solution Manual for Mathematics For JEE (Main & Advanced), Trigonometry Vector
Algebra Probability, Vol 2,
Copyright©2017 Wiley India Pvt. Ltd. All rights reserved
1
and P( A ∩ B ∩ C ) =. Match items of Column I with those of Column II.
3
Column I Column II
(A) P( B ∩ C ) is 1
(p)
12

(B) P( B ∩ C ) is 4
(q)
9

(C) P(( A ∩ C ) / B) is 1
(r)
3

(D) P[( A ∩ C ) / B] is 2
(s)
3

Solution:

2
(A) P( B ∩ C )= P( B) − P( B ∩ C )=
3

1
P( A ∩ B ∩ C=) P( B) − P( A ∩ B) − P( B ∩ C ) + P ( A ∩ B ∩ C=)
3
1
P( A ∩ B ∩ C )= P( A ∩ B) − P( A ∩ B ∩ C )=
3
2
P( B) − P( B ∩ C ) =
3
3 2
− P( B ∩ C ) =
4 3

Answer: (s)

<AQ: Explanation of (B) is missing here.>

A∩ B ∩C 4
(C) =
P( B) 9

Answer: (q)

P( A ∩ B ∩ C ) 4
(D) =
P( B) 9
Answer: (q)
3. A and B are two independent events. It is given that P( A ∩ B) =
1/12 and
P( A ∩ B ) = 1/ 2. Match the items of Column I with those of Column II.
Column I Column
II
(A) P(A) is equal to 1
(p)
3

Solution Manual for Mathematics For JEE (Main & Advanced), Trigonometry Vector
Algebra Probability, Vol 2,
Copyright©2017 Wiley India Pvt. Ltd. All rights reserved
(B) P(B) is equal to 1
(q)
6
(C) P( A ∩ B ) is 1
(r)
2
(D) P( A ∩ B) is 1
(s)
4

Solution:

(A)

1
P( A ∩ B)= P( A) ⋅ P( B)=
12
1
P( A ∩ B) = 1 − P( A) − P( B) + P( A) ⋅ P( B ) =
2
1 1
1 − P( A) − P( B) + =
12 2
1 1 12 + 1 − 6 7
P( A) + P( B) =1 + − = =
12 2 12 12
1 7
P ( A) + =
12 P( A) 12

Let

P( A) = x,
12( x 2 + 1) − 84 x =
0
12 x 2 − 7 x + 1 =0
4 x(3x −1) −1(3x −1) =
0
1 1
x= ,
3 4

Answer: (p), (s)

<AQ: Please check explanation of (B) is missing here.>

1 1
(C) P( A) − P ( A ∩ B) =,
6 4

Answer: (q), (s)

1 1
(D) P( B) − P ( A ∩ B) =,
6 4
Answer: (q), (s)
4. Match the items of Column I with those of Column II.
Column I Column II

Solution Manual for Mathematics For JEE (Main & Advanced), Trigonometry Vector
Algebra Probability, Vol 2,
Copyright©2017 Wiley India Pvt. Ltd. All rights reserved
(A) If two fair dice are rolled, then the (p) 1
probability of sum of the faces is 7 is 4
(B) If a card is picked from a deck of 52 (q) 1
playing cards, then the probability of 2
getting a red card is
(C) P(x) is a polynomial satisfying the (r) 3
relation P( x) + P(2 x) = 5 x 2 − 18 for 8
all real x. Now, each coefficient a, b
and c of the quadratic equation
ax 2 + bx + c is one of the roots of the
equation P(x) = 0. The probability that
ax 2 + bx + c =0 has real roots is

(D) In class X of a school 75% are boys and (s) 1


25% are girls. Probability of boy 6
getting first class is 1/3 while girl
getting first class is 1/2. If one
candidate is selected at random, the
probability of the candidate getting first
class is

Solution:

(A) E = (1, 6), (2,5), (3, 4), (4,3)(5, 2)(6,1)

6 1
P( E=) =
36 6
Answer: (s)

26 1
(B) P( E=) =
52 2
Answer: (q)

F F
(C) =P( F ) P( B) P   + P(G ) P  
 
B G

3 1 1 1 3
= ⋅ + ⋅ =
4 3 4 2 8
Answer: (r)
(D) <explanation missing>
5. The letters of the word MATHEMATICS are written in a row at random. Match the items
of Column I with those of Column II.

Column I Column II
(A) Probability of the two Ms together (p) 2
is 11

Solution Manual for Mathematics For JEE (Main & Advanced), Trigonometry Vector
Algebra Probability, Vol 2,
Copyright©2017 Wiley India Pvt. Ltd. All rights reserved
(B) The probability that two Ms and two (q) 4
As are together is 55

(C) The probability that the vowels are (r) 6


together is 55

(D) The probability of beginning with (s) 1


M and end with M is 55

Solution:

(A) total word =11

when 2 Ms together,

2!10! 2
P=
(E) =
11! 11
Answer: (p)
(B) When 2 Ms and 2 As are together,
2!2!2!9! 4
= P( E ) =
11! 55
Answer: (q)

4!8! 4 × 3 × 2 6
(C) P(=
E) = =
11! 11×105 × 93 55
Answer: (r)
9!2! 1
(D) P(=
E) =
11! 55

Answer: (s)
COMPREHENSION TYPE QUESTIONS
1. Passage: A and B are two bags. A contains 4 fair coins and 3 counterfeit coins while B has 5
fair and 7 counterfeit coins. The counterfeit coins have tail on both sides. Two coins are
transferred from bag A to bag B and then a coin is taken from the bag B and tossed. Answer
the following three questions.
(i) If both coins transferred from A to B are counterfeit coins, then probability of
getting a tail is
3 5 1 23
(A) (B) (C) (D)
4 28 2 28
(ii) If tail appears, then the probability of both coins transferred from bag A are
counterfeit is
1 23 23 23
(A) (B) (C) (D)
7 28 153 66
(iii) If head appears, then the probability of both coins transferred from bag A are fair
is
14 21 25 14
(A) (B) (C) (D)
43 43 43 19

Solution:
(i)
Solution Manual for Mathematics For JEE (Main & Advanced), Trigonometry Vector
Algebra Probability, Vol 2,
Copyright©2017 Wiley India Pvt. Ltd. All rights reserved
E1 = transferring of four coins
E2 = transferring of counter fair coin
E = coin drawn form B is counter fair
Now
4
c1 6 2
P ( E1=) 7
= =
c2 21 7
3
c2 3 1
P( E2=) 7
= =
c2 21 7
E 3
c2 1
P =  =
7
 E1  c2 7
 E  4
c2 2
P =  =
7
 E2  c2 7
Now,
E  P( E1 ) ⋅ P( E / E1 )
P 1 =
 E  P( E1 ) ⋅ ( E / E1 ) + P( E2 ) ⋅ P( E / E1 )
2 / 7 ⋅ 1/ 7 1
= =
2 / 7 ⋅ 1/ 7 + 1/ 7 ⋅ 2 / 7 2
Now for getting a tail
Total counterfeit coins = 7 + 2 = 9
9 3
P( E=) =
12 4
Answer: (A)
(ii)
(iii) <explanation of (ii) and (iii) are missing>
2. Passage: In a tennis tournament, there are 12 players S1 , S2 ,  , S12 and they are divided
into 6 pairs at random. From each pair, the winner will be decided on the basis of the game
played between the two players of the pair. Assume that all players are of equal strength so
that the probability of any player beating any other player is 1/2. Answer the following
questions.
(i) The probability that S1 and S2 are not playing against other is
1 10 1 5
(A) (B) (C) (D)
2 11 11 22
(ii) The probability that exactly one of S1 and S2 is among the losers is
1 5 6 5
(A) (B) (C) (D)
2 22 11 11
(iii) The probability that both S1 and S2 are among the winners is
1 5 6 5
(A) (B) (C) (D)
4 11 11 12
<solution missing>
3. Passage: A box contains 6 coins out of which at least one is biased. Let EK (1 ≤ K ≤ 6)
denote the event that exactly K out of 6 coins are biased. Also let P( EK ) be directly
proportional to K(K + 1). Answer the following questions.
(i) The proportionality constant is equal to
1 1 1 1
(A) (B) (C) (D)
74 148 112 504
(ii) If E is the event of selecting a biased coin out of 6 coins, then P( E1 / E ) is

Solution Manual for Mathematics For JEE (Main & Advanced), Trigonometry Vector
Algebra Probability, Vol 2,
Copyright©2017 Wiley India Pvt. Ltd. All rights reserved
1 1 1 2
(A) (B) (C) (D)
266 132 133 133
(iii) If the coin selected is found to be biased, then P( E6 / E ) is equal to
2 3 5 3
(A) (B) (C) (D)
19 38 38 19
<solution missing>
4. Passage: There are n urns numbered from 1 to n. The Kth urn contains K white balls and
(n + 1 − K ) black balls. Let EK denote the event of selecting the Kth urn at random and let W
denote the event that the ball drawn from the selected urn is white. Let P(A) denote the
probability of an event A. Answer the following questions.
(i) If P( EK ) is proportional to K for K = 1, 2, 3, ..., n, then lim P(W ) is
n→∞
1 2 1 3
(A) (B) (C) (D)
3 3 4 4
(ii) If P( EK ) = C , a constant for all K = 1, 2, 3, ..., n, then P( En / W ) is equal to
2 1 n 1
(A) (B) (C) (D)
n +1 n +1 n +1 2
(iii) If P( EK ) = 1/ n for all K = 1, 2, 3, ..., n and E denote the event of choosing an
even numbered urn, then P(W / E ) is equal to
1 n+4 n+2 n +1
(A) (B) (C) (D)
2n 2(n + 1) 2(n + 1) 2n
<solution missing>
ASSERTION–REASONING TYPE QUESTIONS
In the following set of questions, a Statement I is given and a corresponding Statement II is given
just below it. Mark the correct answer as:
(A) Both Statements I and II are true and Statement II is a correct explanation Statement I.
(B) Both Statements I and II are true but Statement II is not a correct explanation for
Statement I.
(C) Statement I is true and Statement II is false.
(D) Statement I is false and Statement II is true.
1. A fair coin is tossed 3 times. Let A be the event that the first toss gives head; B is the event
that the second toss gives head and C be the event that there are exactly two consecutive heads
or exactly two consecutive tails.
Statement I: A, B, C are independent events.
Statement II: A, B, C are pairwise independent.
Solution: Statement I
1 1 1
= P( A) = , P( B) = , P(C )
2 2 2
as
C = (THH ), ( HHT ), ( HTT )(TTH )
= n(C ) 4,= n( S ) 8
4 1
P(C )= =
8 2
Now,
P( A ∩ B ∩ C=) P( A) ⋅ P( B) ⋅ P(C )
1 1 1 1
= ⋅ = True
8 2 2 8
Statement II

Solution Manual for Mathematics For JEE (Main & Advanced), Trigonometry Vector
Algebra Probability, Vol 2,
Copyright©2017 Wiley India Pvt. Ltd. All rights reserved
P( A ∩ B)= P( A) ⋅ P( B)
2 1
(T , HH ), ( HHT )= =
8 4
1 1 1
= = True
4 2 4
Answer: (B)
2. Suppose two persons A and B have n + 1 and n coins. All the coins are fair. They toss their
coins.
Statement I: The probability that A gets more heads than B is 1/2.
Statement II: The probability that A gets more heads than B is the same as A gets more tails
than B.
Solution:
1
P( A) = probability for head = n + 1
(2)
1
P( B) = probability for head by B =
2n
Statement I
Now,
P( A) 1/(2)n +1 1
= =
P( B) 1/ 2n 2
Statement II
1
Here probability for a head = probability for a tail =
2
Answer: (A)
3. In throwing a pair of fair dice, let A1 be the event that the first die turns up odd number, A2
be the event that the second die turns up odd number and A3 be the event that the sum of the
faces turn up is odd.
Statement I: The events A1 , A2 , A3 are independent.
Statement II: A1 , A2 , A3 are pairwise independent.
Solution:
3 1
P( A1=) =
36 12
3 1
P( A2=) =
36 12
18 1
P( A3=) =
36 2
Statement I
P( A1 ∩ A2 ∩ A3=) P( A1 ) ⋅ P( A2 ) ⋅ P( A3 )
1 1 1 1
0= ⋅ ⋅ = False
12 12 2 288

Statement II
P( A1 ∩ A2 )= P( A1 ) ⋅ P ( A2 )
3 1 1
= ⋅ False
36 12 12
9 1
P( A2 ∩ A3 ) = =
36 4
1 1
=
P( A2 ) ⋅ P( A3 ) =⋅ False
12 2

Solution Manual for Mathematics For JEE (Main & Advanced), Trigonometry Vector
Algebra Probability, Vol 2,
Copyright©2017 Wiley India Pvt. Ltd. All rights reserved
9 1
P( A3 ∩ A1 ) = =
36 4
1 1 1
= ⋅ = False
12 2 24

Answer: (D)
4. Let E1 , E2 ,  , En be mutually exclusive and exhaustive events of a random experiment with
P( Ei ) > 0 for i = 1, 2, 3, ..., n. Let E be any event connected to the experiment with 0 < P(E)
< 1.
Statement I: P( Ei / E ) > P( Ei ) P( E / Ei )
n
Statement II: ∑ P( Ei ) = 1
i =1
Solution:
Statement I
 E  P( E1 ∩ E ) P( E1 ∩ E )
=P 1  =
 E P( E ) 1
= P( E1 ∩ E )
As
e 
P(ee
) = 1, P( i ) P  
 ei 
= P( Ei ∩ E ), P( Ei ∩ E )= 0 False
Statement II
n
∑ P( Ei ) = P( E1 ) + P( E2 ) + .... P( En )
i =1
= P( E1 + E2 + .... En )= 1
= P= ( S ) 1 True.
Note: As E1 , E2 ,, En are mutually exclusive, so
E1 ∩ E = 0
Answer: (D)
5. A random number selector can only select one of the numbers from the set {1, 2, 3, …, 9} and
he will make these selections with equal probabilities.
Statement I: After n trials (n > 1), the probability that the product of these selected numbers
is divisible by 5 is
n n n
8 5 4
1−   −   +  
9 9 9
Statement II: For the product to be divisible by 10, the selections must contain at least one 5
and one even number.
Solution:
Statement I
For n = 2
64 25 16 81 − 64 − 25 + 16 8
1− = − + =
81 81 81 81 81
40
And for n = 3, given from =
729
Which is divisible by 5.
Statement II also true and valid explanation.
Answer: (A)
INTEGER TYPE QUESTIONS
Solution Manual for Mathematics For JEE (Main & Advanced), Trigonometry Vector
Algebra Probability, Vol 2,
Copyright©2017 Wiley India Pvt. Ltd. All rights reserved
The answer to each of the questions in this section is a non-negative integer. The appropriate
bubbles below the respective question numbers have to be darkened. For example, as shown in the
figure, if the correct answer to the question number Y is 246, then the bubbles under Y labeled as
2, 4, 6 are to be darkened.
X Y Z W
ⓞ ⓞ ⓞ ⓞ
① ① ① ①
② ② ② ②
③ ③ ③ ③
④ ④ ④ ④
⑤ ⑤ ⑤ ⑤
⑥ ⑥ ⑥ ⑥
⑦ ⑦ ⑦ ⑦
⑧ ⑧ ⑧ ⑧
⑨ ⑨ ⑨ ⑨
<COMP: Shade 2, 4, 6 in column Y>
1. Each of two bags contains both black and white balls. The total number of balls in both bags
is 25. One ball is taken from each bag. The probability of both balls are black is 27/50. Then
the probability of both balls are white is p/50 where p is ______.
Solution:
Let box 1 contains x balls and box 2 containing y balls.
so,
x+ y = 25 (1)

B
p ( E1 ) = probability of selecting black from B =
x
p ( E2 ) = probability of selecting black from
y − w 1− w
= =
y y
It is clear that E1 and E2 are independent event, so, P( E1 ∩ E2 )= P( E1 ) ⋅ P( E2 )
B  1 − w  27
P( E1 ∩ E2 )= P( E1 ) ⋅ P( E2 )=  =
x  y  50

Solution Manual for Mathematics For JEE (Main & Advanced), Trigonometry Vector
Algebra Probability, Vol 2,
Copyright©2017 Wiley India Pvt. Ltd. All rights reserved
 
Again E1 and E2 probability of selection 1 white from each bag
So,
P( E1 ∩ E2 )= P( E1 ) ⋅ P( E2 )
W  x−B 
= * 
Y  x 
W BW P
= − =
Y XY 50
BW P
1 − P ( E1 ) − P ( E2 ) + =
xy 50
 1 1  27 P
− 1 − −  + =
 4 4  50 50
27 1 P
− =
50 2 50
P 27 − 25 2
= = =, P 2
50 50 50
Answer: (2)
2. 10 girls and 4 boys are to sit at a round table. If the arrangement is random, the probability of
no two boys sit next to each other is a !/(b !c !) . Then (b + c) − a is equal to _____.
Solution:

10 girls seated around a circular table, makes total 10 spot to arrange 4 boys, such that no
2 boys are together
10!
so, 4 boys arrange in 10 p4 =
6!
Now, total circular arrangement of (10 + 4)
(10 + 4 −1)! =13!
10!
Total probability =
6!13!
Here
a =10
=b 6,=c 1
So, b + c − a = 13 + 6 −10 = 9
Answer: (9)
3. Let A and B be two sets. A contains 4 elements while B contains 3. The probability of
choosing a subjective map from A to B is m/n (m/n is in lowest terms). Then the value of n –
m is _____.

Solution Manual for Mathematics For JEE (Main & Advanced), Trigonometry Vector
Algebra Probability, Vol 2,
Copyright©2017 Wiley India Pvt. Ltd. All rights reserved
Hint: See Theorem 6.19, Vol. 1.
Solution:
A contains 4 elements and B contains 3 elements
So, number of mapping = 4∗ 4 ∗ 4 = 64
Now, total suspecting = 59
(As according Theorem 6.19, Vol. 1)
so,
59 m
P( E=
) =
64 n
Now,
n−m=5
Answer: (5)
4. A number from the set {1, 2, 3, …, 15} is selected at random. The probability that it is a
proper divisor of 30 is a/b (in lowest terms). Then a + b is equal to ______.
Solution:
Divisor of 30 in {1, 2,,15}
= {1, 2,3,5, 6,10,15}
7c1 7 a
P( E=) = =
15c1 15 b
a + b = 15 + 7 = 22
Answer: (22)
5. Let m/n, in lowest terms, be the probability that a randomly selected divisor of 1019 is an
integer multiple of 1010. Then m + n is ______.
<solution missing>
6. A box contains 3 gold coins and 4 silver coins. Coins are drawn one by one without
replacement until all the gold coins are drawn. If a/b (in lowest terms) is the probability that
the number of draws required is more than 4, then b – a value is ______.
<solution missing>
7. If the integers m and n are chosen from the set {1, 2, 3, …, 100}, then the probability that a
number of the form 7 m + 7 n is divisible by 5 is 1/K where K is equal to ______.
<solution missing>
8. The probability of a bomb hitting a bridge is 1/3 and two direct hits are enough to destroy the
bridge. The least number of bombs required so that the probability of the bridge getting
destroyed is greater than 0.5 is ______.
<solution missing>
<H1>Answers

<H2>Single Correct Choice Type Questions

1. (C) 14. (A)


2. (D) 15. (D)
3. (B) 16. (B)
4. (A) 17. (B)
5. (B) 18. (A)
6. (C) 19. (A)
7. (B) 20. (D)
8. (A) 21. (B)
9. (B) 22. (C)
10. (A) 23. (A)
11. (B) 24. (B)
12. (A) 25. (C)
13. (C) 26. (A)

Solution Manual for Mathematics For JEE (Main & Advanced), Trigonometry Vector
Algebra Probability, Vol 2,
Copyright©2017 Wiley India Pvt. Ltd. All rights reserved
27. (C) 39. (D)
28. (B) 40. (A)
29. (D) 41. (D)
30. (D) 42. (A)
31. (D) 43. (C)
32. (A) 44. (D)
33. (B) 45. (B)
34. (A) 46. (A)
35. (C) 47. (A)
36. (D) 48. (D)
37. (A) 49. (D)
38. (C) 50. (A)

<H2>Multiple Correct Choice Type Questions

1. (A), (B), (C) 6. (A), (B), (C)


2. (A), (B), (C), (D) 7. 7, (A), (D)
3. 3, (A), (B), (C), (D) 8. 8, (A), (B), (C)
4. 4 (A), (C), (D) 9. (B), (C)
5. (A), (B), (C) 10. (A), (B), (C), (D)

<H2>Matrix-Match Type Questions

1. (A) → (r); (B) → (p); (C) → (t); (D) 4. (A) → (s); (B) → (q); (C) → (p); (D)
→ (q) → (r)
2. (A) → (s); (B) → (p); (C) → (q); (D) 5. (A) → (p); (B) → (q); (C) → (r); (D)
→ (q) → (s)
3. (A) → (p), (s); (B) → (p), (s); (C) →
(q), (s); (D) → (q), (s)

<H2> Comprehension Type Questions

1. (i) (D); (ii) (C); (iii) (A) 3. (i) (C); (ii) (A); (iii) (B)
2. (i) (B); (ii) (C); (iii) (D) 4. (i) (B); (ii) (A); (iii) (C)

<H2>Assertion–Reasoning Type Questions

1. (B) 4. (D)
2. (A) 5. (A)
3. (D)

<H2>Integer Answer Type Questions<Here no parentheses required>

1. 2
2. 6. 5
3. 9 7. 4
4. 5 8. 4
5. 3 9. 5

Solution Manual for Mathematics For JEE (Main & Advanced), Trigonometry Vector
Algebra Probability, Vol 2,
Copyright©2017 Wiley India Pvt. Ltd. All rights reserved
Solutions to Exercises
Chapter 8

SUBJECTIVE QUESTIONS
1. If a, b, c are positive real numbers and a + b + c = 1, then prove that
 1  1  1  8
8 ≤  − 1 − 1 − 1 ≤
 a  b  c  27abc
Equality holds when a= b= c= 1/ 3.
Solution
We have
a+b+c= 1
(1 − a) + (1 − b) + (1 − c) =3 −1 =2
A.M. of this
(1 − a) + (1 − b) + (1 − c) 2
= =
3 3
G.M.
=3 (1 − a ) (1 − b) (1 − c)
A.M. ≥ G.M.
2
3 (1 − a) (1 − b)(1 − c) ≤
3
8
(1 − a) (−1 − b) (1 − c) ≤
27
(1 − a) (1 − b) (1 − c) 8

abc 27abc
 1  1  1  8
 −1 −1 −1 ≤
 a  b  c  27abc
For
1
a= b= c=
3
 1  1  1  8
8 ≤  −1 −1 −1 ≤ (Proved)
 a  b  c  27abc
2. If a, b, c are positive real numbers, then prove that
2(a3 + b3 + c3 ) ≥ ab(a + b) + bc(b + c) + ca(c + a)
≥ 6abc
Solution
We have
2(a3 + b3 + c3 ) ≥ ab (a + b) + bc(b + c) + ca(c + a)
2(a3 + b3 + c3 ) + (a3 + b3 + c3 ) ≥ a3 + a 2b + ab 2 + b3 + b 2 c + bc 2 + c3 + c 2 a + ca 2
3(a3 + b3 + c3 ) ≥ (a + b + c) (a 2 + b 2 + c 2 )
As A.M. is greater than G.M., so,
(a3 + b3 + c3 ) ≥ 3(a3 ⋅ b3 ⋅ c3 )1/ 3
3(a3 + b3 + c3 ) ≥ 3[3(a b c)]
3(a3 + b3 + c3 ) ≥ 9abc
So,
(a3 + b3 + c3 ) ≥ 3 abc

Solution Manual for Mathematics For JEE (Main & Advanced), Trigonometry Vector
Algebra Probability, Vol 2,
Copyright©2017 Wiley India Pvt. Ltd. All rights reserved
2 (a3 + b3 + c3 ) ≥ 6abc (Proved)
3. If n < 10 for a positive integer n ≥ 2, then show that (n + 1)4 < 10n +1 .
4 n

Solution
For n = 2,
(2) 4 < (10)2
16 <100
Let this is true for n = r, so,
(r )4 < (10)r
For r= (r + 1)
(r + 1)4 < (10)r +1
Hence
(n + 1)4 < (10)n +1 (Proved)
4. If x, y are positive such that 3x + 4 y = 5, then show that
3
x2 y3 ≤
16
and equality holds if and only= if x 2= / 3, y 3/ 4.
Solution
We have
3x + 4 y = 5
5−4y
3x = 5 − 4 y, x =
3
Now, let
Z = x2 y3
2
 5−4y 
Z =  ( y)
3
 3 
1
= (25 − 40 y 2 + 16 y 2 ) y 3
9
1
Z= (25 y 3 − 40 y 4 + 16 y 5 )
9

Differentiating ‘Z’ with respect to y.


dz 1
= (75 y 2 −160 y 3 + 80 y 4 )
dy 9

On double differentiating
d2z 1
= (150 y − 480 y 2 + 320 y 3 )
dy 2 9
when
dz
=0
dy
5 y2
(16 y 2 − 32 y + 15) =
0
9

Either y = 0, or, 16 y 2 − 32 y + 15 =
0

Solution Manual for Mathematics For JEE (Main & Advanced), Trigonometry Vector
Algebra Probability, Vol 2,
Copyright©2017 Wiley India Pvt. Ltd. All rights reserved
32 ± 1024 − 640
y=
32
32 ± 22
=
22
1 1
y = , 2, Z < 0 for y =
2 2

Hence,
3
x2 y3 ≤
16

5. If x, y, z are positive such that x + y + z =


18, then prove that the maximum value of
x 2 y 3 z 4 is 42 6384 .
Solution
We know that A.M. > G.M.
Now,
First term a=x
Second term b =x+d =y
Third term c =x + 2d =z
x+ y+ z= 18
x + ( x + d ) + ( x + 2d ) =
18
3x += 3d 18, x += d 6
Second term b = 6
First term a = 4 , third term c = 8
then, maximum value of x 2 y 3 z 4
= (4)2 (6)3 (8) 4 (Proved)
6. If a, b, c, d, e, f, g are positive, then show that
 a b c  e f g 
 + +  + +  ≥ 9
 e f g  a b c 
Solution
A.M. ≥ G.M.
1/ 3
 a b c   abc 
 + +  ≥ 3 
 e f g   efg 
1/ 3
 e f c   efg 
and  + +  ≥ 3 
 a b g   abc 
so,
1/3
 a b c  e f g   a b c efg 
 + +  + +  ≥ 3× 3  ⋅ 
 e f g  a b c   efg a b c 
 a b c  e f g 
 + +  + +  ≥ 9 (Proved)
 e f g  a b c 
7. If a, b, c are positive, then prove that
1 1 1
(a + b + c)  + +  ≥ 9
a b c

Solution Manual for Mathematics For JEE (Main & Advanced), Trigonometry Vector
Algebra Probability, Vol 2,
Copyright©2017 Wiley India Pvt. Ltd. All rights reserved
When does the equality occur?
Solution
A.M. ≥ G.M.

1/ 3
 a + b + c  1/ a + 1/ b + 1/ c    1 
   ≥ (abc)  
 3  3    abc  
1 1 1
(a + b + c)  + +  ≥ 9 (Proved)
a b c
Equality holds for a= b= c= 1
8. If a, b are positive, then prove that 2 lies between a/b and (a + 2b) /(a + b) .
Solution
a (a + 2b)
A.M. of and
b a+b
a / b + (a + 2b) / a + b a 2 + 2ab + 2b 2
= =
2 2b(a + b)
( a + b) 2 + b 2
=
2b(a + b)
We know that
A.M.
≥1
G.M.
Hence,
( a + b) 2 + b 2
≥1
2b(a + b)
Hence,
a a + 2b
< 2< (Proved)
b a+b
Let
9 x 2 sin 2 ( x) + 4
y=
x sin( x)
9 x 2 sin 2 ( x) + 4
A.M. =
2
=
G.M. 9 x2 =
sin 2 ( x) ⋅ 4 6 x sin ( x)
Now, A.M. ≥ G.M.
2(6) x sin( x)
9 x 2 sin 2 ( x) + 4 ≥
x sin( x).
Minimum volume by = 12 (Proved)
9. If 0 < x < π / 2, then show that the minimum value of
9 x 2 sin 2 x + 4
x sin x
is 12.
<solution missing>
10. In ∆ABC, prove that the in radius
a 2 + b2 + c2
r<
3(a + b + c)
<solution missing>

Solution Manual for Mathematics For JEE (Main & Advanced), Trigonometry Vector
Algebra Probability, Vol 2,
Copyright©2017 Wiley India Pvt. Ltd. All rights reserved
11. If a, b, c are the sides of ∆ABC, prove that
| a 2 (b − c) + b 2 (c − a) + c 2 (a − b) |< abc
(Hint: Factorize the expression and use that the difference of two sides is less than the
third side.)
Solution
We have
a 2b − a 2 c + b2 c − b2 a + c 2 a − c 2b
ab(a − b) + bc(b − c) + ca(c − a)
as we know that (a − b) < C.
so,
abc + bca + cab
3abc
= = abc
A.M. (Proved)
3
12. If θ1 , θ 2 ,  , θ n are real, then show that
2 2
 n   n 
∑ θ +  ∑ sin θi  ≤ n
2
cos i
= i 1=  i 1 
Solution
For i = 1
cos 2 θ + sin 2 θ ≤ (1) =
1 True
For i = 2
(cos θ1 + cos θ 2 ) 2 + (sin θ1 + sin θ 2 ) 2
2 + 2(cos θ1 cos 2 + sin θ1 sin θ 2 ) ≤ (2) 2 2
2 + 2cos(θ1 − θ 2 ) ≤ 4
As cos(θ1 − θ 2 ) ≤1
so, 2 + 2cos(θ1 − θ 2 ) ≤ 4
Hence, it is true for n term and
2 2
 n   n 
 ∑ cos θ1  +  ∑ sin(θ1 )  ≤ n
2
(Proved)
=  i 1=  i 1 
13. If a1 , a2 ,  , an are positive real numbers such that a1 + a2 +  + an = s then prove that
s s s s n2
+ + + + ≥
s − a1 s − a2 s − a3 s − an n − 1
where n ≥ 2
<solution missing>
14. If A, B, C are independent events with positive probabilities of a random experiment, then
prove that
P( A) + P( B) + P(C ) ≥ 3[ P( A ∩ B ∩ C )]1/ 3
When will the equality hold?
Solution
We know that
A.M. ≥ G.M.
P( A) + P( B) + P(C )
≥ [ P( A) ⋅ P( B) ⋅ P(C )]1/ 3
3
P( A) + P( B) + P(C ) ≥ 3[ P( A) ⋅ P( B ) ⋅ P (C )]1/ 3 (Proved)

Solution Manual for Mathematics For JEE (Main & Advanced), Trigonometry Vector
Algebra Probability, Vol 2,
Copyright©2017 Wiley India Pvt. Ltd. All rights reserved
15. Let A, B, C be mutually exclusive and exhaustive events with non-zero probabilities of a
random experiment, and E is any event of the experiment with P( E ) > 0, then prove that
P( E ) ≥ 3[ P( A) P( B) P(C )]1/ 3
× [ P( E / A) P( E / B) P( E / C )]1/ 3
Solution
As A, B, and C are mutually exclusive
So, P( E ) = P( A) + P( B) + P(C )
Now, A.M. ≥ G.M.
P( A) + P( B) + P(C )
≥ [ P( A) ⋅ P( B) ⋅ P(C )]1/ 3
3
P( E ) ≥ 3[ P( A) ⋅ P( B) ⋅ P(C )]1/ 3 (Proved)
16. If a1 , a2 ,  , an are positive real numbers such that each ai < 1 and
a1 + a2 +  + an = sn ≠ 1, then prove that
1
(1 + a1 ), (1 + a2 ),, (1 + an ) <
1 − sn
Solution
We have
(1 + a1 ) + (1 + a2 ) + (1 + a3 ) +  + (1 + an )
n
n + (a1 + a2 − an ) sn
= = 1 − . <Please check the question and the answer. Answer not matching with
n n
the question.>
17. If n > 1 integer, then prove that n > n n / 2 .
<Solution missing>
18. If a, b, c are positive real numbers, then prove that
(a + b + c)(bc + ca + ab) ≥ 9abc
Solution
We know that
a+b+c
A.M. =
3
bc + ac + ab
and
3
(a + b + c) (bc + ac + ab)
≥ (abc)1/ 3 (bc ⋅ ac ⋅ ab)1/ 3
3 3
(a + b + c)(bc + ac + ab) ≥ a ⋅ abc. (Proved)
19. If n > 1 integer, then prove that n n > 1 ⋅ 3 ⋅ 5> (2n − 1).
<Solution missing>
20. In many books, one finds the following problem:
If a1 , a2 ,  , an are positive and (n − 1) s = a1 + a2 +  + an , then show that the product
a1 ⋅ a2 ⋅ a3  an > (n − 1)n ( s − a1 )( s − a2 ) ( s − an )
This result is wrong if we consider=
a1 1,= a2 2 and a=
3 a=
4 6. Find the fallacy in the
argument (or proof) given by the authors.
<Solution missing>

Solution Manual for Mathematics For JEE (Main & Advanced), Trigonometry Vector
Algebra Probability, Vol 2,
Copyright©2017 Wiley India Pvt. Ltd. All rights reserved

You might also like